You are on page 1of 223

CONTENT

Verbal Reasoning
1. Calender................................................................. 2-8
2. Clock...................................................................... 9-13
3. Direction Test......................................................... 14-20
4. Coding-Decoding.................................................... 21-26
5. Series..................................................................... 27-40
6. Puzzle..................................................................... 41-53
7. Seating Arrangment................................................ 54-59
8. Alphabet Test.......................................................... 60-64
9. Analogy................................................................... 65-73
10. Clasification............................................................ 74-79
11. Blood Relation........................................................ 80-86
12. Sequence & Ranking............................................... 87-91
13. Arithemetical Reasoning......................................... 92-101
14. Missing Term.......................................................... 102-112
15. Arrangment of Word............................................... 113-122
16. Simplification......................................................... 123-128
17. Word Formation..................................................... 129-131

Non Verbal Reasoning


18. Dice........................................................................ 133-139
19. Cube & Cubiod....................................................... 140-146
20. Logical Vernn Diagram........................................... 147-152
21. Counting of Figure.................................................. 153-160
22. Diagram Series....................................................... 161-174
23. Mirror & Water Image............................................. 175-181
24. Paper Cutting & Folding......................................... 182-185

Logical Reasoning

25. Syllogism................................................................ 186-199


26. Statement & Conclusion......................................... 200-205
27. Statement & Arguments......................................... 206-214
28. Statement & Assumption........................................ 215-219
29. Statement & Course of Action................................. 220-223

1
1 2009 fall ?
1. Which year will have the same calendar as
that of 2009? (a) Sunday (b) Monday
(a) 2012 (b) 2016 (c) Tuesday (d) Friday
(c) 2020 (d) 2015 9. How many odd days are there in 527 days ?
2. What day of the week will 1st January, 2020 (a) 5 (b) 2
be, given that 1st January, 2009 is a Sunday ?
(c) 3 (d) 6
(a) Thursday (b) Saturday
10. Which among the following years is a leap
(c) Monday (d) Friday year ?
3. Which year will have the same calendar as (a) 1700 (b) 2300
that of 2012 ?
(c) 3800 (d) 3200
(a) 2020 (b) 2040
11. How many days will there be from 26th
(c) 2025 (d) 2031 January, 2004 to 15th May, 2004 (both days
4. Imagine that in a calendar year, there were 998 included) ?
days and 12 days in a week, then no. of odd
(a) 110 (b) 111
days in that year will be ?
(c) 112 (d) 113
(a) 5 (b) 6
(e) None of these
(c) 3 (d) 2
12. If 30th January, 2003 was Thursday, what was
5. If 18th February, 2005 falls on Friday then
the day on 2nd March, 2003?
what will be the day on 18th February, 2007 ?
(a) Tuesday (b) Thursday
(a) Sunday (b) Wednesday
(c) Saturday (d) Sunday
(c) Tuesday (d) Monday
13. If 21st July, 1999 is a Wednesday, what would
6. If 20th February, 1996, which is your date of
have been the day of the week on 21st July,
birth Tuesday. On what day of the week will
1947?
your birthday fall in the year 1998?
(a) Sunday (b) Thursday
(a) Thursday (b) Tuesday
(c) Monday (d) Saturday
(c) Saturday (d) Friday
14. On which dates of April, 2012 will a Sunday
7. Which two months in a year have the same come?
calendar (a) 5, 12, 19, 26 (b) 1, 8,15, 22, 29
(a) April, December (c) 3, 10, 17, 24 (d) 7, 14, 21, 28
(b) September, December 15. If 6th March, 2009 is on Friday, then what
(c) May, August day of the week is on 6th December, 2009 ?
(d) August, December (a) Saturday (b) Sunday
(c) Tuesday (d) Monday
8. On which day of the week does 28th August,

2
16. Meenu was born on 6th December, 1989. 22. Republic day was celebrated on Friday in
When was she 5 years 8 months and 22 days year 1996. What day will Independence day
old ? celebrated in year 2000?
(a) 28.05.1994 (b) 28.08.1995 (S.S.C-2011)
(a) Tuesday
(c) 15.02.1994 (d) 28.08.1994
(b) Monday
17. An application was received by inward clerk (c) Friday
in the afternoon of a week day. Next day he (d) Saturday
forwarded it to the table of the senior clerk, 23. Hari recalls that his father’s birth day falls
who was on leave that day. The senior clerk between 12th and 16th June whereas her
next day evening put up the application to the sister recalls that father’s birthday falls
desk officer. Desk officer studied the between 14th and 18th June. What can be
application and disposed off the matter on the father’s birthday date which they both
same day, i.e., Friday. Which day was the agree?
application received by the inward clerk ? (S.S.C-2011)
(a) Saturday (b) Sunday (a) 14 June (b) 15 June
(c) Wednesday (d) Tuesday (c) 16 June (d) 17 June
18. In which of the following year february was 24. It 11 September 2000 is Friday then what
th

not a 29 days month? will be the day on 15th September 2001?


(S.S.C-2002)
(S.S.C - 2014)
(a) Friday (b) Saturday
(a) 2000 (b) 2004
(c) Thursday (d) Sunday
(c) 1996 (d) 1966 25. Independence day was celebrated on
19. If 1st January 1999 is Tuesday then what Wednesday in year 1988. What day it will
day is on 1st January 2000? be celebrated in 1989.
(S.S.C - 2011) (S.S.C-2001)
(a) Tuesday (b) Thursday (a) Monday (b) Tuesday
(c) Friday (d) Thursday
(c) Friday (d) Wednesday
26. If 2 day of a month is Sunday then what
nd
20. If 1st jannary 1991 was tuesday, then in the will be the day on the 31st the same month?
same year which month will have tuesday (S.S.C-2002)
as 1st day of mounth? (a) Friday (b) Sunday
(S.S.C-2012) (c) Monday (d) Tuesday
(a) May (b) October 28. If John celebrated his victory day on
(c) September (d) November Tuesday 5th January 1965 then when will
21. In year 1996 republic day was celebrated he celebrate the victory day on the same
on Friday. What day will Independence day?
day be celebrated in year 2000? (S.S.C-2011)
(S.S.C-2012) (a) 5 January, 1970 (b) 5 January, 1971
(a) 4 Saturday and 4 Monday (c) 5 January, 1973 (d) 5 January, 1974
(b) 5 Saturday and 5 Monday 29. 1st march 2008 was Saturday. What was the
(c) 4 Saturday and 5 Monday day on 1st march 2002?
(d) 5 Saturday and 44 Monday (IAS - 2008)

3
(a) Thursday (b) Friday 36. I was born on 11th of August. Arvind is 11
(c) Saturday (d) Monday days Younger to me. Independence day this
30. If in any month 3 day is Monday then what
rd year falls on Monday, then Which day was
Arvind born?
will be the fifth day from 21st day of the
same month? (S.S.C-2011)
(IAS - 2014) (a) Sunday (b) Tuesday
(a) Monday (b) Tuesday (c) Monday (d) Wednesday
(c) Wednesday (d) Friday 37. Which date of June 2099 among the
31. Jamal was born on 29th February 1988. How following is Sunday?
many birth days he celebrated till 29-2- (USPC-2022)
2004? (a) 4 (b) 5
(S.S.C 2013) (c) 6 (d) 7
(a) 4 (b) 5 38. Raman remembers that the examination is
(c) 8 (d) 16 after 15th May but before 18th may, while
32. Joseph visits the club on every 5th day, Deep remembers that the examination is
before 21st May but after 16th May. on
Harsh visits on every 24th day, while Sumit
which date of May is the examination?
visit on every 9th day. If all three of them
met at the club on a Sunday, then on which (SSC- 2017)
day will all three of them meet agin? (a) 17 (b) 18
(UPSC - 2021) (c) 19 (d) 20
(a) Monday (b) Wednesday 39. If today is Sunday, then what will be the
(c) Thursday (d) Sunday 59th day after today?
33. If 1 January is Friday, then for a leap year
st (S.S.C- 2011)
what will be the 1st day of march? (a) Tuesday (b) Monday
(S.S.C 2013) (c) Wednesday (d) Thursday
(a) Wednesday (b) Thusday 40. What day of the week was 29 June, 2010?
(c) Friday (d) Tuesday (S.S.C-2012)
34. If 5th of a month is Tuesday, then what will (a) Tuesday (b) Sunday
be the date on day after third day from third (c) Wednesday (d) Monday
Friday in same mouth ? 41. What will be the next term in the following
(S.S.C-2013) sequence, 4/12/95, 1/1/96, 29/1/96, 26/2/
(a) 22 (b) 19 96..
(c) 18 (d) 17 (IAS-1995)
35. In a particular year 16 of June was Friday,
th
(a) 24/3/96 (b) 25/3/96
then in the same year what will be the date (c) 26/3/96 (d) 27/3/96
on first Friday of July? 42. What day of the week was 5 February,
(S.S.C-2012) 2008?
(a) 7 July (b) 8 July (SSC-2020)
(c) 5 July (c) 6 July (a) Thursday (b) Wednesday
(c) Monday (d) Tuesday

4
43. If 9th of a month fall on previous day to (b) Monday (d) Friday
Sunday, then what day will 1st of the month 49. If day after tomorrow is Sunday then what
fall? will be day after tomorrow for ?
(S.S.C-2000) (S.S.C-2010)
(a) Friday (b) Saturday
(a) Friday (b) Thursday
(c) Sunday (d) M onday
(c) Monday (d) Tuesday
44. A nil arrives a place on Friday, he comes to
50. If two day after tomorrow will be Thursday
know that he has arrives 3days earlier, if
then what was the day previous to 3 days
he arrives on Sunday, then how many days
before yesterday.
earlier or late he would arrive?
(S.S.C-2000) (S.S.C-2002)
(a) 1 day earlier (a) Monday (b) Tuesday
(b) 1 day later (c) Wednesday (d) Thursday
(c) Two days later 51. If first day from yesterday was Wednesday,
(d) Two days earlier then when will Sunday fall?
45. If day before yesterday was Tuesday, then (S.S.C-2013)
what will be the next day to tomorrow? (a) 3 days from today
(S.S.C-2002) (b) Tomorrow
(a) Monday (c) Today
(b) Wednesday
(d) Day after Tomorrow.
(c) Friday
52. Nine months before Mohini went to cinema
(d) Saturday she goes for cinema only on Thursday.
46. If day before yesterday was thursday, then Which day of week is today?
when is Sunday?
(S.S.C-2010)
(S.S.C-2006)
(a) Saturday (b) Thursday
(a) Day after tomorrow
(c) Sunday (d) Monday
(b) Today
53. If in any month 23rd is Sunday, then what is
(c) Tomorrow the day previous to two weeks and four
(d) Two days from today. days?
47. Three days earlier to yesterday was (a) Monday (b) Tuesday
Wednesday, then what day will fall two (c) Wednesday (d) Thursday
days after tomorrow?
54. Calculate on the basis of given situation. If
(S.S.C-2002) 1st of November falls on Monday, then what
(a) Wednesday (b) Monday is the day on November 25th?
(c) Friday (c) Tuesday (S.S.C-2002)
48. If one day after tomorrow is Thursday then (a) Tuesday (b) Thursday
What was the day, day before yesterday? (c) Wednesday (d) Friday
(S.S.C-2002) 55. If third Friday of any month is 16th, then
what will be the date on the fourth Tuesday
(a) Saturday (b) Sunday
of same month?
5
(S.S.C-2002) (c) Wednesday (d) Friday
(a) 20 (b) 22 62. If second and fourth Saturdays and all the
(c) 27 (d) 29 Sundays are taken as only holidays for an
office, what would be the minimum number
56. In any month of date 5th falls 2 days after
of possible working days of any month of
Monday, then what will be the previous day
any year?
to 19th of the same month?
(a) 23 (b) 22
(S.S.C-1999) (c) 21 (d) 20
(a) Wednesday (b) Thursday 63. Which day is 10 October, 2027?
th

(c) Tuesday (d) Monday (2021)


(a) Sunday (b) Monday
57. Day after tomorrow is Kiran is birthday.
(c) Tuesday (d) Saturday
The same day , next week, is ‘Shivratri’. 64. If in particular year 12th January is a sunday,
Today is Monday what will be the next day then which one of the following is correct?
of ‘Shivratri’? (2022)
(S.S.C-2002) (a) 15th july is a sunday if the year is a leap
(a) Wednesday (b) Thursday year
(b) 15th july is a sunday if the year is not a
(c) Friday (d) Saturday
leap year
58. Today is Friday. Last Monday was 29 (c) 12th July is a sunday if the year is a
December 1975, Then what is the date leap year
today? (d) 12th july is not a sunday if the year is a
(S.S.C-2002) leap year
(a) 28 December, 1975 65. Which year has the same calendar as that
2009?
(b) 3 January, 1976 (2022)
(c) 2 January, 1976 (a) 2018 (b) 2017
(c) 2016 (d) 2015
(d) 2 January, 1975
66. Seeta and Geeta go for a swim after a gap
59. What is the number of days between 26 of every 2 days and every 3 days
January 2006 to 23 September 2006 (both respectively. If on 1st January both of them
the days in cluded). went for a swim together, when will they
go together next?
(S.S.C-2002)
(UPSC-2019)
(a) 214 (b) 241 (a) 7th January (b) 8th January
(c) 249 (d) 251 (c) 12th January (d) 13th January
67. If in a particular year 12th January is a
60. If 4th of any month is Saturday then what
sunday, then which one of the following is
will be the day on 27th of the same month? correct?
(S.S.C-2002) (UPSC-2020)
(a) Monday (b) Thursday (a) 15th July is a Sunday, if the year is a
leap year
(c) Friday (d) Saturday (b) 15th July is a Sunday, if the year is not
61. If the 3rd day of a month is Monday, which a leap year
one of the following will be the fifth day (c) 12th July is a Sunday, if the year is a
from 21st of this month? leap year
(a) Modday (b) Tuesday

6
(d) 12th July is not a Sunday, if the year is
a leap year
68. Joseph visits the club on every 5th day,
Harsh visits on every 24th day, while Sumit
visits on every 9th day. If all three of them
met at the club on a Sunday, then on which
day will all three of them meet again?
(UPSC-2021)

(a) Monday
(b) Wednesday
(c) Thursday
(d) Sunday
69. Which date of June 2099 among the
following is Sunday?
(UPSC-2022)
(a) 4
(b) 5
(c) 6
(d) 70
49. If today is Sunday, then which day is it
exactly on 1010 th day?
(UPSC-2023)
(a) Wednesday
(b) Thursday
(c) Friday
(d) Saturday

7
Answer Sheet
1. (d) 2. (b) 3. (b) 4. (d)
5. (a) 6. (d) 7. (b) 8. (d)
9. (b) 10. (d) 11. (b) 12. (d)
13. (c) 14. (b) 15. (b) 16. (b)
17. (c) 18. (d) 19. (d) 20. (b)
21. (b) 22. (a) 23. (b) 24. (b)
25. (d) 26. (a) 27. (d) 28. (b)
29. (b) 30. (c) 31. (b) 32. (d)
33. (d) 34. (b) 35. (a) 36. (c)
37. (d) 38. (a) 39. (d) 40. (a)
41. (b) 42. (b) 43. (a) 44. (a)
45. (d) 46. (c) 47. (a) 48. (b)
49. (b) 50. (d) 51. (d) 52. (a)
53. (c) 54. (b) 55. (c) 56. (a)
57. (b) 58. (c) 59. (b) 60. (a)
61. (c) 62. (b) 63. (a) 64. (c)
65. (d) 66. (d) 67. (c) 68. (b)
69. (d) 70. (b)

8
2 8. The reflex agle between the hands of a
1. When seen through a mirror, a clock shows
8:30. The correct time is clock at 10.25 is :
(a) 2 : 30 (b) 3 : 30
10
(c) 5 : 30 (d) 8 : 30 (a) 180 0
(b) 192
2
2. By looking in a miror, it appears that it is 6
: 30 in the clock. What is the real time? 10
(c) 195 0
(d) 197
(a) 6 : 30 (b) 5 : 30 2
(c) 6 : 00 (d) 5 : 30 9. At what time between 7 and 8 o'clock will
3. A clock seen through a mirror shows the hands of a clock be in the same straight
quarter past three. What is the correct time line but, not together?
shown by the clock?
2
(a) 9 : 45 (b) 9 : 15 (a) 5 min. past 7 (b) 5 min. past
11
(c) 8 : 45 (d) 8 : 15 7
4. At what angle of the hands of a clock are
inclined at 15 minutes past 5? 3 5
(c) 5 min. past 7 (d) 5 min. past 7
10 11 11
(a) 58 (b) 640
2 10. At what time between 9 and 10 o'clock will
1 0
1 0 the hands of a watch be together?
(c) 67 (d) 72
2 2 (a) 45 min. past 9
5. The angle between the minute hand and the
(b) 50 min. past 9
hour hand of a clock when the time is 4:20,
is : 1
(c) 49 min. past 9
(a) 00 (b) 100 11
(c) 50 (d) 200 1
(d) 48 min. past 9
6. An accurate clock shows 8 o'clock in the 11
morning. Through how many degrees will 11. At what time between 4 and 5 O' clock will
the hour hand rotate when the clock shows the hands of a clock be at right angle?
2 o'clock in the afternoon? (a) 30 minute past 4
(a) 1440 (b) 1500 3
(b) 16 minute past 4
(c) 1550 (d) 1800 4
7. The angle between the minute hand and the 2
(c) 38 minute past 4
hour hand of a clock when the time is 8.30, 11
is : (d) 33 minute past 4
(a) 800 (b) 750 12. A watch, which gains uniformly, is 2 minute
slow at noon on Monday, and is 4 minut,
(c) 600 (d) 1050

9
48 sec fast at 2 pm on the following will the watch be fast by more than 60
Monday. At what time it was correct? minutes?
(a) 2 pm on Tuesday (S.S.C -2002)
(b) 2 pm on Wednesday
(a) Five (b) Six
(c) 3 pm on Thursday
(c) Seven (d) Eight
(d) 1 pm on Friday
13. At what time between 5 and 6 are the hands 17. 20 minutes before 8:50 Vivek realized that
of a clock coincident? he has arrived 30 minutes earlier to Aman
(a) 22 minute past 5 40 minutes delay for meeting place. What
(b) 30 minute past 5 is the scheduled lime for meeting?
(S.S.C - 2002)
8
(c) 22 minute past 5 (a) 8.20 (b) 8.10
11
3 (c) 8.05 (d) 8.00
(d) 27 minute past 5 18. A watch from midnight, becomes late by 5
11
14. 14. What time will the hands of the clock minutes in first hour, late by 10 minutes at
superimpose each other between 6 and 7. end of 2nd hour, late by 15 minutes at end
(S.S.C -2011) of 3rd hour and so on it gets late. What time
will watch show at the end of 6 hours?
8
(a) 32 minute past 6 (S.S.C-2002)
11
(a) 6:00 am (b) 5:30 am
8
(b) 34 minute past 6 (c) 6:30 am (d) 5:15 am
11
19. What time between 9 and 10 will the hour
8
(c) 30 minute past 6 and minute hand be in opposite directions?
11
(S.SC -2011)
5
(d) 32 minute past 6 (a) 15 minute past 9
7
(b) 16 minute past 9
15. Samay pal in formed Mr. Chopra “Bus for
Meerut leaves every half an hour. Last bus 4
(c) 16 minute past 9
left five minutes back. Next bus did Samay 11
pal informed this to Mr. Chopra? 3
(S.S.C -2006) (d) 17 minute past 9
11
(a) 1.45pm (b) 1.55pm 20. If we represent 1 to 4 hours of day and night
(c) 2.00pm (d) 2.05pm with English alphabet on a clock’s dial and
16. A watch becomes fast by 1 minute during start it with letter ‘C’, then which letter will
first hour, 2 minutes late by end of 2nd hour, be expressed at 16 in the clock?
4 minutes late by end of 3rd hour, 5 minutes (S.S.C-2010)
late by end of 4th hour. So on the situation (a) W (b) P
continues. At the end of how many hours
(c) S (d) R

10
21. A clock in which dots are marked at 3,6,9 (a) On completion of 7 days
and 12 is hung up side down in-front of a (b) On completion of 30 days
mirror. What time will the image show at (c) On Completion of 120 days
4:15?
(d) On Completion of 180 days.
(a) 2.15 (b) 8.45
26. After interval a school runs from 1:00pm
(c) 8.50 (d) 9.45 till 3:52 pm. Four periods are held during
22. Imagine that there hours after every period.4 minutes are
1. Hour and minute hand of a watch given to move from one class room to an-
moves without stroke. other. Find the duration of each period.

2. Watch shows a time between 8 and 9. (a) 40 minutes (b) 42 minutes

3. Both the hands of watch are over (c) 35 minutes (d) 45 minutes
lapping each other. 27. How many times do the hour-hand and the
minute-hand of a watch become coincident
After how many minutes (nearest integer)
to each other in 24 hours?
will both the hands overlap again?
(a) 22 (b) 24
(a) 60 (b) 62
(c) 44 (d) 48
(c) 65 (d) 67 28. In a precise clock, the minute hand will
23. What is the acute angle between hour hand move in a period of 2 hours 20 minutes.
and minute hand of a wall clock when it (UPSC-1996)
shows 3:25 pm.
(a) 520 0
(b) 3200
1
o
(c) 8400 (d) 1400
(a) 60o
(b) 52
2 29. How many times in a day are the hour hand
and the minute hand of a clock at right
o
1 angles to each other?
(c) 47 (d) 42o
2 (UPSC-1997)
24. Krishna’s friend Ram comes to meet him (a) 44 (b) 48
every Sunday. He visited first at 12:30pm, (c) 24 (d) 12
next line at 1:20 pm, again at 2:30 pm and
30. A precision (Accurate) clock shows the
again at 4pm. What time did Ram visit
time 3:00, the time after the hour hand has
Krishna after that?
moved 1350 will be?
(S.S.C -2012) (UPSC-1997)
(a) 5.20pm (b) 5.30pm (a) 7.30 (b) 6.30
(c) 6.20pm (d) 5.50pm (c) 8.00 (d) 9.30
25. A watch loses 2 minutes pr day (2hr) while 31. A clock shows the time as 8 o'clock in the
another watch gains 2 minutes per day morning. How many degrees will the hour
(24hr). At a particular instant the two hand of the clock move till 2 o'clock in the
watches showed an identical time. When afternoon?
will the two watches show the identical (UPSC-2000)
time gain?
11
(a) 1500 (b) 1440 minutes?
(c) 168 0
(d) 180 0
(a) 6:15 pm (b) 6:18 pm
32. What is the acute angle between the hour (c) 6:36 pm (d) 6:48 pm
hand and the minute hand of a wall clock 37. A class starts at 11:00 a.m. and ends at 2:27
when the time is 3.25 pm? p.m. In this interval there are four periods
(UPSC-2002) of equal duration. After each period,
(a) 600 (b) 52.50 students are given a break of 5 minutes.
(c) 47.50 (d) 420 How long is the exact duration of each
period?
33. A Clock Show the time of nine o’ clock
fourteen minites (a o’clock 14 minutes) The (UPSC-2020)
positions of the hour hand and the minute (a) 48 minutes (b) 50 minutes
hand of a clock are virtually interchanged. (c) 51 minutes (d) 53 minutes
The new time shown by the clock is closest 38. A clock slows down by 2 minutes every 24
to which one of the following? hours, while another clock goes fast by
(UPSC-2006) 2 minutes every 24 hours. At a
(a) twelve minutes to three particular moment both the clocks show
(b) thirteen minutes to three the same time. Which of the following
statements is correct if followed by a
(c) fourteen minutes to three
24-hour clock?
(d) fifteen minutes to three
(a) On completion of 30 days, both the
34. How many times will the hour hand and
clocks again show the same time.
the minute hand of a clock be at right angles
(b) On completion of 90 days, both the
while moving from 1 pm to 10 pm?
clocks again show the same time.
(UPSC-2009)
(c) On completion of 120 days, both the
(a) 9 (b) 10
clocks again show the same time.
(c) 18 (d) 20
(d) None of the above statements is correct.
35. Imagine that-
39. A clock strikes once at 1 o'clock, twice at 2
1. The hour and minute hands of a clock o'clock and three times at 3 o'clock and so
move without jerks. on. If it takes 12 seconds to strike at 5
2. Shows the time between 8 o'clock and o'clock, how long will it take to strike at
9 o'clock. 10 o'clock?
3. Both the hands of the clock are above (UPSC-2021)
each other. (a) 20 seconds
After how many minutes (to the nearest (b) 24 seconds
integer) will the two needles be above each (c) 28 seconds
other once again? (d) 30 seconds
(a) 60 (b) 64 40. A wall clock moves 10 minutes fast in every
(c) 65 (d) 66 24 hours. This clock was corrected to show
36. At what time between 6 p.m. and 7 p.m., the correct time on Monday at 8:00 AM.
the minute hand of a clock is ahead by 3 When this clock shows 6:00 PM on

12
Wednesday, what is the correct time?
(UPSC-2022)
(a) 5:36 pm
(b) 5:30 pm
(c) 5:24 pm
(d) 5:18 pm

Answer Sheet
1. (b) 2. (b) 3. (c) 4. (c)
5. (b) 6. (d) 7. (b) 8. (d)
9. (d) 10. (c) 11. (c) 12. (b)
13. (d) 14. (a) 15. (d) 16. (b)
17. (a) 18. (b) 19. (c) 20. (d)
21. (a) 22. (c) 23. (c) 24. (d)
25. (d) 26. (a) 27. (a) 28. (c)
29. (a) 30. (a) 31. (d) 32. (c)
33. (c) 34. (c) 35. (c) 36. (c)
37. (a) 38. (d) 39. (c) 40. (a)

13
3 again turns left, turns right, again turns right,
TYPE-I once again turns left. In which direction is 'A'
walking now
1. 'A' man starts from a point and walks 2km (a) East (b) South
towards North, turns towards his right and
(c) West (d) South-East
walks 2km, turns right again and walks. What
is the direction now he is facing? TYPE-II
(a) South (b) South-East
(c) North (d) West 7. Mamatha walks 14 metres towards west, then
turns to her right and walks 14 metres and then
2. Arun started from point A and walked 10 kms
turns to her left and walks 10 metres. Again
East to point B, then turned to North and
turning to her left she walks 14 metres. What
walked 3 kms to point C and then turned West
is the shortest distance (in metres) between her
and walked 12 kms to point D, then again
starting point and her present position?
turned South and walked 3 kms to point E. In
whichdirection is he from his starting point (a) 38 m (b) 28 m
(a) East (b) South (c) 24 m (d) 10 m
(c) West (d) North 8. Hari travelled 17 km to the east, he turned left
and went 15 km, he again turned left and went
3. A starts from a point and walks 5 kms north, 17 km. How far is he from the starting point?
then turns left and walks 3 kms. Then again
turns left and walks 5 kms. Point out the (a) 17 km (b) 2 km
direction in which he is going now. (c) 15 km (d) 32 km
(a) North (b) South 9. A man starts from his office and goes 5 km
(c) East (d) West East. Then, he turns to the left and again walks
for 3km. Again he turns left and walks 5km.
4. I started walking down a road in the morning At what distance is he from the starting point?
facing the Sun. After walking for sometime I
turned to my left. Then I turned to my right. In (a) 3 km (b) 4 km
which direction was I going then (c) 6 km (d) 7 km
(a) East (b) West 10. Mohan travelled westward 5 kms., turned left
(c) North (d) South and travelled 3 kms., turned right and travelled
9 kms. He then travelled north 3 kms. How far
5. Roy walks 2 km to East, then turns North-West he is from the starting point?
and walks 3 km. Then he turns South and walks
5 km. Then again he turns West and walks 2km. (a) 5 kms. (b) 3 kms.
Finally he turns North and walks 6km. In which (c) 6 kms. (d) 14 kms.
direction, is he from the starting point
11. Sonia started from her house and travelled 4
(a) South-West (b) South-East
km towards east. Then she turned left and
(c) North-West (d) West-North travelled 6 km. Then she turned right and
6. 'A' starts walking towards North, turns left, travelled 4 km. Now at what distance is she

14
from starting point? direction, then 1350 in anticlockwise direction.
(a) 14 km (b) 8 km What directon am i facing?

(c) 5 km (d) 10 km (a) South (b) North


12. A boat moves from a jetty towards East. After (c) North-West (d) South-West
sailing for 9 nautical miles, she turns towards
17. Sandhya walks straight from point A to B which
right and covers another 12 nautical miles. If
is 2 kms away. She turns left, at 900 and walks
she wants to go back to the jetty what is the
shortest distance now from her present 8 kms to C, Where she turns left again at 900
position? and walks 5 kms to D. At D she turns left at
900 and walks for 8 kms to E. How far is she
(a) 21 nautical miles (b)20 nautical miles
from A to E?
(c) 20 nautical miles (d)15 nautical miles
(a) 2 (b) 3
13. A cyclist rides 40 kms to the east, turns north
and rides 20 kms, again turns left and rides 20 (c) 5 (d) 8
kms. How far is he from the starting point?
18. A man is facing South. He turns 1350 in the
(a) 0 km (b) 10 kms anticlockwise direction and then 1800 in the
(c) 20 kms (d) 30 kms clockwise direction. Which direction is he
14. Ram and Shyam started from a fixed place. facing now?
Ram moves 3 km to the North and turns right , (a) South-East (b) South-East
then walks 4 km. Shyam moves to wards west
and walks 5 km, then turns to right and walks (c) North-East (d) North-West
3 km. How far Ram is from Shyam? 19. At 12.30 the hour hand of a clock faces North
(a) 13 km (b) 16 km and the minute hand face South. At 2.45 the

(c) 9 km (d) 10 km minute hand will be in which direction?

15. My friend and I started simultaneously towards (a) North-West (b) West
each other from two places 100 m apart. After (c) South-East (d) East
walking 30 m, my friend turns left and goes
10m, then he turns right and goes 20m and then 20. If the clock read 6.20 and if the minute hand
turns right again and comes back to the road points North-East, in which direction will the
on which he had started walking. If we walk hour hand point?
with the same speed, what is the distance (a) West (b) South-East
between us at this point of time?
(c) East (d) North-West
(a) 50 m (b) 20 m TYPE-IV
(c) 30 m (d) 40 m 21. A is located to the West of B. C is located at
TYPE - III North in between A and B. D is exactly to the
South of B and also in line with B. In which
16. I am facing West. I turn 90 in clockwise
0
direction of C is D located?

15
(a) South (b) South-East
(c) West (d) South-West ANSWER KEY
22. Four villages A,B, C and D lie in a straight line.
D is 10 kms from B. A is exactly between D TYPE-I TYPE-II TYPE-III
and C from B is 2 kms more than it is from D. 1. (a) 7. (c) 16. (d)
How far is C from B? 2. (c) 8. (c) 17. (b)
(a) 4 kms (b) 6 kms 3. (b) 9. (a) 18. (b)
(c) 8 kms (d) 2 kms 4. (a) 10. (d) 19. (b)
23. A man walks down the backside of his house 5. (c) 11. (d) 20. (c)
straight 25 metres, then turns to the right and 6. (a) 12. (d) TYPE-IV
walks 50 metres again: then he turns towards
13. (d) 21. (b)
left and again walks 25 metres, If his house
faces to the East, what is his direction from the 14. (c) 22. (b)
starting point? 15. (b) 23. (d)
(UPSC-2020) 24. (a)
(a) South-East (b) South-West 25. (c)

(c) North-West (d) North-West


24. Two friends X and Y start running and they
run together for 50 m in the same direction and
reach a point. X turns right and runs 60 m, while
Y turns left and runs 40 m. Then X turns left
and runs 50 m and stops, while Y turns right
and runs 50 m and then stops. How far are the
two friends from each other now?
(UPSC-2022)
(a) 100 m (b) 90 m
(c) 60 m (d) 50 m
25. What is the number of selections of 10
consecutive things out of 12 things in a circle
taken in the clockwise direction?
(UPSC-2023)
(a) 3 (b) 11
(c) 12 (d) 66

16
EXPLANATIONS
4. (a)
TYPE - I
1. (a) The diagram given below is quite helpful
in solving the questions on Distance and
Direction Test. Starting Point

Now I am going towards east.


North
North- North-
West East 5. (c) 1km

West East 3km

South- South- 2km


West East
South
5km

2. (c) D 12 km
C
Now Roy is in North-west direction from
3 km 3 km the starting point.

E B 6.(a)
A 10 km

It is clear from the diagram that Arun is


in west direction from the starting point.

Starting Point
3. (b) 3 km

Now, A is waking towards east.


5 km 5 km
TYPE - II

7. (c)
Starting Point
14m 14m
It is clear from the diagram that A is going
towards South.
10m 14m Starting
Point

Required distance = 10 + 14 = 24 metres

17
12. (d) 9 nautical
A miles
8. (c) Final 17km B
Point

15km 15km 12 nautical


miles

Starting 17km
point C

5km Required distance


9. (a)
3km = A C  ( ) 2  (BC) 2
5km
Office  (9) 2  (12) 2
The Man is 3 km away from his office.
 81  144

E B 5km A
10. (d)  225
3km 3km = 15 nautical miles

D 9km C
13. (d) D 20km C
Required distance = (9 + 5) km = 14 km
20km

C 4km D E
11. (d) B
A 40km
6km 6km

A 4km B E North

Required distance
West East
= AD  (AE) 2  (DE) 2

 (8) 2  (6) 2 South

 64  36  100 Required distance = AD


= 10 km
= (AE)2  (DE) 2

 (20) 2  (20) 2

18
Net movement 450 anticlockwise
 400  400

 800  28.28km D 5 km
17. (b) C
 30 km

E B 4 km 8 km
14. (c) C

3 km 3 km
B
D 5 km A E 3 km A 2 km

The distance between A and E = 3 km


North

18. (b)
West East
130 ACW
180 CW

South

EC = EB + BC = (5 + 4) km = 9 km

15. (b) 20m North


10m NW NE
10m
30m
100km West East

Friend covers a total distance of = (30 +


10 20 + 10) = 70m Distance between SW SE
them = 70- 50 = 20m South

TYPE - III It is clear from the diagram that now the


man is facing South-West direction.
16. (c) Initial Position

Final Position

19
located in the South- East direction of
N C.
19. (b) 12
11 1
10 2
W9 3 E
8 4
7 5
6
S

It is clear the diagram that minute hand


will be in west direction.
20. (c) At 6.20 the minute hand points towards
South-East. It means the clock has been
rotated through 900 anticlockwise and
hence it points North-East. Thus, the hour
hand will point towards East.

TYPE - IV

2km 2km 6km


21. (b) D

CB = 6km

22. (b)
C

A B

North
NW NE

West East

SW SE
South

It is clear from the diagram that D is

20
4
1. If Orange is called lemon, Lemon is called ‘Money makes only part’ is written as ‘ zi la
flower, flower is called fish, fish is called tail ne ki’ ‘demand makes supply economics’ is
and tail is called pen, then what is Rose? written as ‘zi mo ka ta’
(S.S.C-2013) 5. W hat is the code for ‘ money’ in the given code
(a) Pen (b) Lemon language?

(c) Flower (d) Fish (a) ga (b) mo


2. If rain is called water, water is called air, air is
(c) pa (d) la
called cloud, cloud is called sky, Sky is called
sea, sea is called Road, then where do Aero 6. W hat is the code for ‘ supply’ in the given code
plane fly?
language?
(S.S.C-2013)
(a) Only ta (b) Only mo
(a) Water (b) Road
(c) Sea (d) Clod (c) Either pa or mo (d) Either mo or ta

3. If Rat is called Dog, Dog is called Mongoose, 7. W hat may be the possible code for’ demand
Mongoose is called Lion, Lion is called Snake, only more’ in the given code language?
and Snake is called Elephant, then which
category will the pet animal fall? (a) xi ne mo (b) mo zi ne

(S.S.C-2012) (c) ki ne mo (d) mo zi ki


(a) Mongoose (b) Lion
8. W hat may be the possible code for ‘ work and
(c) Rat (d) Dog money’ in the given code language ?
4. If police is called Teacher, Teacher is called
Politician, Politician is called Doctor, Doctor (a) pa ga la (b) pa la lu
is called lawyer and lawyer is called surgeon (c) mo la pa (d) tu la ga
then who will catch the criminals?
9. W hat is the code for ‘ makes’ in the given code
(S.S.C-2012)
language?
(a) Teacher (b) Doctor
(a) mo (b) pa
(c) Police (d) Lawyer (c) ne (d) zi
Directions: Study the information and answer Directions : Study the following in formation
the following questions. to answer the given question.
In a certain code language.
In a certain code, ' always create new ideas' is
‘economics is not money’ is written as ‘ ka la written as 'ba ri sha gi', 'ideas and new thoughts
ho ga’ ' is written as ' fa gi ma ri', 'create thoughts
‘ demand and supply economies’ is written as and insights' is written as 'ma jo ba fa', and
‘mo ta pa ka’ 'new and better solutions' is written as 'ki ri to
fa'.
21
(Bank PO 2011) Matrix-I Matrix-II
0 1 2 3 4 5 6 7 8 9
10. What is the code for 'ideas?
0 A B C D E 5 P Q R S
(a) Sha (b) ba 1 D C B A E Q S P R T
(c) gi (d) ma 2 B A D C E 7 P T R S Q
11. What does 'fa' stand for ? 3 D B C A E 8 Q S P R
4 C D A E B T P S Q R
(a) thoughts (b) insights
(a) 32, 00, 56, 10 (b) 40, 21, 68, 44
(c) new (d) and
(c) 11, 33, 57, 22 (d) 02, 42, 77, 20
12. 'fa lo ba' could be a code for which of the
following? 17. For example ‘F’ can be said to be 14,21etc.
(a) thoughts and action ‘S’ can be represented as 58, 96 etc. Then find
the group to represent ‘TRIP’.
(b) create and innoveat
(S.S.C-2012)
(c) ideas and thoughts
Matrix-I Matrix-II
(d) create new solutions 0 1 2 3 4 5 6 7 8 9
13. What is the code for 'new'? 0 D E F I N 5 O P R S
1 I N D E F S T O P R
(a) ki (b) ri
2 E F I N D 7 P R S T O
(c) to (d) fa 3 N D E F I 8 T O P R
14. Whichof the following may represent 'insights 4 F I N D E R S T O P
always better? (a) 78, 76, 21, 76 (b) 59, 57, 41, 56
(a) jo ki to (b) ki to ri (c) 85, 88, 33, 89 (d) 66, 69, 40, 69
(c) sha jo ri (d) to sha jo
18. For example, ‘F’ can be shown as 14, 21 etc.
15. What is the code for 'thoughts'? and ‘E’ can be shown as 20,32 etc. The
(a) ma (b) fa recognize the group for word FIRE.
(c) ba (d) jo (S.S.C-2012)
Instruction: A word is re presented by a group Matrix-I Matrix-II
of numbers, which is given in any one of the 0 1 2 3 4 5 6 7 8 9
options Number group in options are 0 D E F I N 5 O P R S
represented by two square of letters, as shown 1 I N D E F S T O P R
below in two grids. In grid I the Rows and 2 E F I N D 7 P R S T O
3 N D E F I 8 T O P R
columns are numbered from 0 to 4 and in grid
4 F I N D E R S T O P
II it is numbered from 5 to 9. In these grids a
letter can be represented first by row number (a) 21, 22, 88, 33 (b) 14, 10, 69, 14
then by column number.
(c) 33, 34, 76, 22 (d) 02, 03, 57, 01
16. For example ‘B’ can be represented by 01, 31
19. For example, ‘G’ can be presented by 02, 14,
etc. and ‘P’ can be represented by 67, 75 etc.
etc. and ‘P’ can be represented by 66, 78 etc.
Then is same manner recognize the group for
In the same manner recognize the group that
word ‘CARD’.
represent the word LINE.
(S.S.C-2013)
(S.S.C-2012)

22
Matrix-I Matrix-II 23. If NOTE is written as PQVG, then how TIME
0 1 2 3 4 5 6 7 8 9 will be written?
0 E G H I 5 L M N O
1 H I E F G O P L M N (S.S.C-2008)
2 F G H I E 7 M N O P L
(a) VQOG (b) VKOG
3 I E F G H 8 P L M N
4 G H I E F N O P L M (c) VOKG (d) VGKO
(b) 86, 31, 69, 42 (b) 86, 11, 68, 42 24. If FLOWER is code as EMNXDS the how
(c) 86, 11, 69, 43 (d) 86, 11, 69, 40 will SHOWER be coded?
20. For example, ‘B’ can be represented as 04, (S.S.C-2013)
11, 23 etc. and ‘N’ can be represented as 59, (a) RINXDS (b) SINXDS
66, 78 etc. Find the group for given word
(c) RGNXDS (d) TINXDS
MILK.
25. If RHYTHEMIC is written in from of
(S.S.C-2011)
QGXSGLHB then how is MUSIC written in
same code?
Matrix-I Matrix-II
(S.S.C-2003)
0 1 2 3 4 5 6 7 8 9
0 M F H B 5 I K S U (a) NVRHB (b) LTRHB
1 H B M L F U N I K S
2 L F H B M 7 K S U N I (c) NVTJD (d) LVTHB
3 B M L F H 8 N I K S
4 F H B M L S U N I K
26. If ‘BASKET’ is written as ‘TEKSAB’ then in
same code how is ‘PILLOW’ written?
(a) 12, 67, 32, 99 (b) 31, 86, 33, 87
(S.S.C-2012)
(c) 21, 76, 32, 95 (d) 10, 67, 49, 88
(a) LOWPIL (b) WOLLIP
21. Code for letters in first row, codes are provided
in the second row. (c) LOWLIP (d) WOLPIL

V D A S G K I 27. If JACOB is written in from of QZXLY then


H X O how is KENDY written?
5 1 3 9 0 8 2 (S.S.C-2013)
4 7 6 (a) PVWMA (b) PVMWB
What is decode of IVSHOD (c) PUMWB (d) PYMWA
(a) 258416 (b) 259641 28. IF FIREWOOD is written as ERIFDOOW in
(c) 254961 (d) 259461 a code language, then in same code how is
22. In a code MARCH is written as OCTEJ. How FRACTION written?
will RETORN be written in same code? (S.S.C-2011)
(UPSC-2000) (a) ARFITCNO (b) NOITCARF

(a) TFUVSM (b) QGSTQM (c) CARFNOIT (d) CRAFNOIT


29. IF CAT and BOTE are written XZG and
(c) TGVQTP (d) TGRVSO
YLGV respectively in a code script, then in
same code how is EGG written?
23
(S.S.C-2001) (a) CARFTION (b) FRACNOIT
(a) VSS (b) URR (c) NOTICARF (d) CARFNOIT
(c) VTT (d) UTT 36. In a certain code, 256' means 'red colour chalk',
'589' means 'green colour flower' and 254'
30. IN a sign language MASTER is symbolized
means white colour chalk'. The digit in the
as OCUVGT and RANGE as TCPIG. In same
code that indicates 'white' is
manner how is MANAGERS symbolized?
(UPSC-2020)
(S.S.C-2000)
(a) 2 (b) 4
(a) OCPICGTU (b) OCPCIGTU
(c) 5 (d) 8
(c) OCICPGTV (d) OCPICGTV
37. If LSJXVC is the code for MUMBAI, the
31. IF MARS is written as ARMS, then how is code for DELHI is
ZNEF written is that code language?
(UPSC-2021)
(S.S.C-1999) (a) CCIDD (b) CDKGH
(a) NEZF (b) FENZ (c) CCJFG (d) CCIFE
(c) NFZE (d) MEZF 38. If RAMON is written as 12345 and DINESH
as 675849, then HAMAM will be written as
32. IF word MENTAL is written
LNDFMOSUZBKM, then how the word (UPSC-2020)
TEST written in that code ? (a) 92233 (b) 92323
(c) 93322 (d) 93232
(S.S.C-2000)
39. If $ means 'divided by'; @ means 'multiplied
(a) UVFGTUUV (b) RSCDQRRS by'; # means 'minus', then the value of
(c) SUDFQRSU (d) SUDFRTSU 10#5@1$5 is
33. If in a code Language DELHI is written 79541 (UPSC-2020)
and CALCUTTA is written 82589662 then in (a) 0 (b) 1
that code, how is CALICUT written? (c) 2 (d) 9
(S.S.C-2011) 40. Let A3BC and DE2F be four-digit numbers
where each letter represents a different digit
(a) 5279431 (b) 5978013
greater than 3. If the sum of the numbers is
(c) 8251896 (d) 8543691 15902, then what is the difference between
34. IF GECA means 8642, then what does HFB the values of A and D ?
mean? (UPSC-2022)

(S.S.C-2008) (a) 1 (b) 2


(c) 3 (d) 4
(a) 9735 (b) 7953
41. The letters from A to Z are numbered from 1
(c) 7935 (d) 5379
to 26 respectively. If GHI = 1578 and DEF=
35. A military code writes SYSTEM as SYSMET 912, then what is ABC equal to?
and NEARER as AENRER. Using the same
code, FRACTION can be written as: (UPSC-2022)
(a) 492 (b) 468
(2016)
(c) 262 (d) 246
24
42. In a certain code 'CHAIR' is written as # H (e) None of these
»% and HIDE is written as V» $. How is 48. If 'black' means 'yellow' 'yellow' means 'blue'
'Dear' written in that code? 'blue' means 'green' 'green' means 'white' and
(UPSC-2003) 'white' means 'red', then which one of the fol-
(a) $ % (b) $» % # lowing will represent the colour of 'milk?
(c) $ % » (d) # % (UPSC-2009)
(e) None of these (a) white (b) green
43. In a certain code 'AROMATIC' is written as (c) red (d) black
BQPLBSJB. How is 'BRAIN' written in that (e) None of these
code? 49. In a certian code GEAR is written as and
(UPSC-2003) FIRM is written as 3@ 7. How is FAME writ-
(a) CQBJO (b) CSBJO ten in that code?
(c) CQBHO (d) CSBHO (UPSC-2009)
(d) None of these (a) 397% (b) 3%97
44. In a certain code 'MOTHER' is written as (c) 597% (d) 5397
'OMHURF. How will 'ANSWER' be written (e) None of these
in that code? 50. In a certian language code 'COURT' is writ-
(UPSC-2004) ten as %@ and 'TILE' is written ' ©$ '. How
(a) CNBWRRF (b) MAVSPE is 'CITE' written in that language code?
(c) NBWTRD (d) NBXSSE (UPSC-2009)
(d) None of these (a) 5$84 (b) 5%84
45. If 'blue' means 'green', 'green' means 'white', (3) 5©84 (4) 3©84
'white' means 'yellow', 'yellow' means 'black', (e) None of these
'black' means 'red' and 'red' means 'brown' then 51. In a certain code TRAIN is written as ' V %'
which one of the following will represent the and MEAL is written as ' $V@'. How is ITEM
colour of 'milk'? written in that code?
(UPSC-2004) (UPSC-2009)
(a) green (b) brown (a) 7$34
(c) red (d) yellow (b) 73$4
(e) black (c) 79$4
46. In a certain code language 'ACID' is written (d) 73V4
as 'JDBE' and 'NOSE' is written as 'TPOF'. (e) None of these
How will 'FORD' be written in that code? 52. A person X wants to distribute some pens
(UPSC-2005) among six children A, B, C, D, E and F. Sup-
(a) ESPG (b) SPGE pose A gets twice the number of pens received
(c) SPGC (d) PGES by B, three times that of C, four times that of
(e) None of these D, five times that of E and six times that of F.
47. In a certian code BEND is written as What is the minimum number of pens X
and DIAL is written as . How is LINE should buy so that the number of pens each
written in that code? one gets is an even number?
(UPSC-2009) (UPSC-2020)
(a) 6394 (a) 147
(b) 6943 (b) 150
(c) 6934 (c) 294
(d) 6134 (d) 300

25
53. 10. In a code language 'MATHEMATICS' is (UPSC-2022)
written as 'LBSIDNZUHDR'. How is (a) OGXPMLD
'CHEMISTRY' written in that code language? (b) OGXQMLE
(UPSC-2021) (c) OFXPMLE
(a) DIDLHRSSX (d) OFXPMLD
(b) BIDLHTSSX 55. If 'ZERO' is written as 'CHUR', then how is
(c) BIDLHTSSX 'PLAYER' written?
(d) DGFLIRUQZ (UPSC-2023)
54. If the order of the letters in the English alpha- (a) SOCAGT
bet is reversed and each letter represents the (b) SODBGT
letter whose position it occupies, then which (c) SODBHT
one of the following represents 'LUCKNOW? (d) SODBHU

Answer Sheet
1. (d) 16. (c) 31. (a) 46. (a)
2. (c) 17. (b) 32. (d) 47. (e)
3. (a) 18. (d) 33. (c) 48. (c)
4. (a) 19. (c) 34. (a) 49. (a)
5. (d) 20. (a) 35. (d) 50. (c)
6. (d) 21. (d) 36. (b) 51. (b)
7. (a) 22. (c) 37. (a) 52. (d)
8. (b) 23. (b) 38. (b) 53. (b)
9. (d) 24. (a) 39. (d) 54. (d)
10. (c) 25. (b) 40. (c) 55. (d)
11. (d) 26. (b) 41. (d) ******
12. (b) 27. (b) 42. (e)
13. (b) 28. (c) 43. (c)
14. (d) 29. (c) 44. (e)
15. (a) 30. (b) 45. (d)

26
5
TYPE - I (a) 431 (b) 314
(c) 531 (d) 532
Direction: Find the missing number in the
following series. 11. 8, 24, 12, ? , 18, 54
1. 2460, 3570, 4680, ? (a) 28 (b) 36
(a) 8640 (b) 5670 (c) 46 (d) 38
(c) 5970 (d) 5790 12. 36, 28, 24, 22, ?
2. 3, 15, 35, 63, ? (a) 18 (b) 19
(a) 77 (b) 84 (c) 21 (d) 22
(c) 99 (d) 98 13. 5255, 5306, ___, 5408, 5459
3. 4, 10, 22, 46, ?, 190 ? (a) 5057 (b) 5357
(a) 56 (b) 16 (c) 2257 (d) 5157
(c) 76 (d) 94 14. 12, 36, 108, 324, ?
4. 2, 5, 10, 17, 26, 37, 50, ? (a) 972 (b) 648
(a) 8640 (b) 5670 (c) 1296 (d) 432
(c) 5970 (d) 5790 15. 44, 40, 34, ?, 16, 4
5. 7, 22, 37, ? , 67, 82, (a) 28 (b) 26
(a) 8640 (b) 5670 (c) 21 (d) 19
(c) 5970 (d) 5790 16. 110, 99, 86, ?, 54, 35
6. 15, 31, 64, 131, ? (a) 81 (b) 75
(a) 266 (b) 256 (c) 71 (d) 67
(c) 192 (d) 524 17. 2, 3, 5, 6, ?, 9, ?, 12
7. 33, 28, 24, ? , 19, 18 (a) 9, 11 (b) 11, 8
(a) 21 (b) 22 (c) 8, 11 (d) 8, 10
(c) 20 (d) 23 18. 98, 72, ?, 32, 18, 8
8. 313, 623, 933, 1243, ? (a) 42 (b) 46
(a) 1863 (b) 2173 (c) 50 (d) 54
(c) 1553 (d) 2483 19. 0, 3, 8, 15, ?, 35, 48
9. 44, 56, 69, 83, ? , 144 (a) 26 (b) 24
(a) 90 (b) 98 (c) 22 (d) 20
(c) 100 (d) 110 20. 19, 23, 26, 30, 33, ?
10. 975, 864, 753, 642, ? , (a) 31 (b) 35

27
(c) 37 (d) 39 (c) 60, 11 (d) 62, 9
21. 2, 4, 12, 48, ? 5. 18, 25, 23, 30, ?
(a) 224 (b) 232 (a) 25 (b) 35
(c) 240 (d) 256 (c) 28 (d) 38
22. 6, 17, 39, 72, ? 6. 2, 65, 7, 59, 12, 53, ? , ?
(a) 94 (b) 127 (a) 15, 42 (b) 17, 45
(c) 83 (d) 116 (c) 17, 47 (d) 18, 48
23. 2.1, 5.2, 8.4, 11.7, 15.1, ? 7. 22, 28, 25, 31, 28, ?
(a) 17.1 (b) 18.6 (a) 25 (b) 34
(c) 18.4 (d) 17.3 (c) 36 (d) 39
24. 2, 8, 40, ?, 1680, 13440 8. 87, 90, 84, 88, 81, 34, ?, ?
(a) 80 (b) 120 (a) 85, 93 (b) 86, 98
(c) 160 (d) 240 (c) 86, 78 (d) 86, 68
25. 5, 11, 17, 25, 33, 43, ? 9. 4, 8, 7, 14, 10, 20, ?
(a) 53 (b) 56 (a) 14 (b) 30
(c) 49 (d) 51 (c) 13 (d) 12
26. 3, 5, 8, 13, 21, 34, 55, ? 10. 15, 17, 20, 22, 27, 29, ? , ?
(a) 34 (b) 55 (a) 31, 38 (b) 36, 38
(c) 21 (d) 89 (c) 36, 43 (d) 38, 45
11. 97, 86, 99, 88, 101, ? , ?
TYPE - II (a) 88, 89 (b) 90, 103
Direction : Find the missing number in (c) 121, 108 (d) 114, 103
the following series. 12. 17, 14, 15, 12, 13, ? , ?
1. 1, 0, 3, 2, 5, 6, ? (a) 10, 11 (b) 14, 11
(a) 9 (b) 8 (c) 11, 13 (d) 12, 15
(c) 10 (d) 7 13. 4, 5, 6, 9, 8, 13, 10, ?
2. 18, 24, 21, 27, ?, 30, 27 (a) 14 (b) 15
(a) 33 (b) 30 (c) 11 (d) 17
(c) 24 (d) 21 14. 48, 82, 44, 77, 40, 72, ?
3. 4, 8, 12, 24, 36, ? (a) 76 (b) 70
(a) 72 (b) 48 (c) 36 (d) 40
(c) 60 (d) 144 15. 62, 61, 63, 62, 64, 63, ___?
4. 1, 2, 3, 14, 5, 34, 7, ? , ? (a) 66 (b) 62
(a) 68, 7 (b) 63, 9 (c) 64 (d) 65

28
16. 12, 8, 14, 6, 16, ?
TYPE - IV
(a) 18 (b) 32
(c) 5 (d) 4 Direction : Which of the following
letter groups when sequentially placed
TYPE - III at the gaps in the given letter-series will
complete it?
Direction: Find the missing number in the 1. a, c, f, j, ? , ?
following series.
(a) ou (b) mo
1. 0, 4, 18, 48, ?, 180
(c) 1p (d) rv
(a) 58 (b) 68
2. T, R, P, N, L, ?, ?
(c) 84 (d) 100
(a) J, G (b) K, I
2. 56, 90, 132, 184, 248,?
(c) J, H (d) K, H
(a) 368 (b) 316
3. A, D, H, M, ?, Z
(c) 362 (d) 326
(a) B (b) G
3. 0, 7, ?, 63, 124
(c) S (d) N
(a) 26 (b) 24
4. a, r, c, s, e, t, g,__,__
(c) 23 (d) 22
(a) x, z (b) u, i
4. 0, 6, 24, 60, ?, 210
(c) w, y (d) v, b
(a) 117 (b) 119
5. b, c, f, g, j, k, ?
(c) 120 (d) 153
(a) p (b) o
5. 7, 25, 61, 121, 124 ?
(c) n (d) m
(a) 210 (b) 211
6. D, H, L, R, ?
(c) 212 (d) 209
(a) T (b) X
6. 4, 18, 48, ? , 180
(c) I (d) O
(a) 80 (b) 100
7. D, E, F, H, I, J, M N O, ?
(c) 105 (d) 125
(a) I K J (b) S T U
7. 3, 8, 18, 35, ? ] 98
(c) O P Q (d) X Y Z
(a) 61 (b) 71
8. A, B, D, G, ?
(c) 41 (d) 51
(a) I (b) J
8. 1, 8, 17, 30, ?, 76
(c) K (d) L
(a) 39 (b) 49
9. a, d, c, f, ? h g ? i
(c) 59 (d) 69
(a) j e (b) f j
9. 7, 25, 61, 121, ?
(c) e k (d) e j
(a) 210 (b) 211
10. I, W, K, U, M, S, O, Q, ?
(c) 212 (d) 209
(a) P (b) L
29
(c) Q (d) M 22. AN, BO, CP, DQ, ?
11. B, F, J, N, R, V, Z, D, H, L, ? (a) E G (b) ER
(a) M (b) O (c) EH (d) EF
(c) P (d) M 23. CAT, FDW, IGZ, ?
12. A, D, G, J, M, P, ? (a) KTC (b) KJA
(a) Q (b) R (c) LHD (d) LJC
(c) T (d) S 24. FFI, KMP, ?, YAD
13. A, C, F, J, O ? , B (a) QSV (b) RTW
(a) I (b) H (c) SUX (d) RTV
(c) U (d) D 25. NOA, PQB, RSC, ?
14. A, Z, B, Y, C ? , ? (a) TUD (b) DTU
(a) X D (b) WE (c) ENO (d) FNQ
(c) DX (d) EW 26. KDW, MGT, OJQ, ?
15. A, P, C, Q, E, R, G, __, __ (a) MNQ (b) QNM
(a) S I (b) H I (c) NMQ (d) QMN
(c) I S (d) T J 27. bcde ebcd debc cdeb ?
16. AB, DEF, HIJK, ?, STUVWX (a) dcbe (b) bcde
(a) LMNOP (b) LMNOR (c) cdbe (d) dbce
(c) MNOPQ (d) QRSTU 28. AZBY, CXDW, EVFU, ?
17. AO, BM, CK, __, EG (a) SHTG (b) GXHW
(a) EI (b) DH (c) GTHS (d) LGY
(c) DI (d) EH 29. EAC, GCE, IEG, ?
18. AB, DC, EF, HG, IJ, LK, ? (a) JHI (b) KGI
(a) LM (b) NM (c) JGI (d) KIJ
(c) MN (d) ML 30. BDF, CFI, EIM, ?
19. AC, FH, ?M, (a) AEH (b) HMR
(a) I (b) J (c) KPS (d) RVZ
(c) K (d) L 31. SFJ, RGK, QHL, ?
20. AE, FJ, ? , PT (a) EMD (b) VIJ
(a) X D (b) WE (c) PIM (d) PKL
(c) DX (d) EW 32. BDF, CFI, DHL, ?
21. AD, EH, IL, ? , QT (a) EJO (b) CJM
(a) LM (b) MN (c) EML (d) EMI
(c) MP (d) OM 33. ICE, JDF, KEG, LFH, ?

30
(a) MIG (b) MHG 9. aac – bba – cc – baa – cb –
(c) MGI (d) HHI (a) bacbc (b) abaac
(c) cabcb (d) bcacb
34. The next step in the series YEB, WFD,
10. a _ cdd_ bcd _ abc _ dab _
UHG, SKL, ?
(a) b a d d c (b) a b d d c
(a) QGL (b) TOL (c) b a d c d (d) b d a d c
(c) QNL (d) QOL 11. _ _ dan_ _nda_dand_
35. What is the next term in this series? (a) dnadna (b) ndanda
BMO, EOQ, HQS, ? (c) andana (d) danndn
12. a _ ba_ b _ b _ a _ b
(a) KSU (b) LMN
(a) abaab (b) aabba
(c) SOV (d) None of these (c) bbabb (d) abbab
13. aa_a_b_abaa_
TYPE - V (a) abab (b) baab
(c) baba (d) abba
Direction: Which of the following letter-
14. Which set of letters when sequentially
groups when sequentially placed at the gaps
placed at the gaps in the given letter
in the given letter-series will complete it?
series will complete it?
1. cc–ccbc–accbcc–c–b
a _ ca_ ca _ caa _
(a) a c a c (b) a b a c
(a) caca (b) cacc
(c) a b a b (d) a a b c
(c) caac (d) ccca
2. H_JH_IJHHI_HH_JH
15. AZB _, AZ _ Y, A_ BY, _ ZBY
(a) IHJI (b) HIHI
(a) YBZA (b) BYAZ
(c) IHIJ (d) HJHJ
(c) BZYA (d) AZBY
3. a b-aa-aaa-a-a b-a
(a) a b b a b (b) a b a a a TYPE - VI
(c) a a a b b a (d) a b b a a Direction: Find out the wrong number in
4. _bc c_ a abc _ ba ab _ the given series:
(a) acbc (b) abac
(c) abcc (d) acac 1. 216, 163, 120, 72, 24
5. bca_b_aabc_a_bba (a) 216 (b) 163
(a) c b a b (b) b a c c (c) 72 (d) 24
(c) a c a b (d) b c b b 2. 6, 11, 16, 27, 38
6. a _ b_ b a _ b _ _b a
(a) 6 (b) 38
(a) b b a a b (b) b b a b b
(c) a a b a b (d) a a b b a (c) 16 (d) 27
7. an _ nn_ ana _ na _nan_ a 3. 41, 41, 42, 44, 48, 51, 56
(a) annan (b) aanan (a) 48 (b) 51
(c) nanna (d) aanan (c) 56 (d) 41
8. cab_c_bdca_d_abd
(a) d, a, b, c (b) c, c, a, a 4. 9, 14, 19, 25, 32, 40
(c) b, c, d, a (d) a, b, c, d (a) 14 (b) 25

31
(c) 32 (d) 9 following number sequence which is
5. 5, 11, 23, 47, 96 followed by an even number and
preceded by an odd number?
(a) 47 (b) 23
302503203202704058070
(c) 96 (d) 11
4208
6. 25, 36, 49, 81, 121, 169, 255
(a) 4 (b) 3
(a) 36 (b) 121
(c) 2 (d) 1
(c) 49 (d) 81
6. How many of the following letter series
7. 62, 46, 34, 24, 16, 10 are A which is immediately followed
(a) 62 (b) 46 by B and immediately preceded by R?
(c) 34 (d) 24 A M B Z A N AA B Z A B A Z B A P Z
ABAZAB
TYPE - VII (a) 0 (b) 1
Direction: How many '8's are there before (c) 2 (d) 3
an even number and after an odd number 7. How many 8 are followed by an even
in the given number series? number and preceded by an odd
1. 3 8 2 5 8 3 2 8 3 2 8 2 7 8 4 8 5 6 8 7 8 number?
4286 7, 8, 5, 6, 7, 8, 4, 3, 2, 1, 3, 8, 6, 4, 3, 7,
(a) 5 (b) 4 8, 4, 2, 1, 3, 8, 2, 8, 9
(c) 3 (d) 2 (a) 4 (b) 3
2. 4, 1, 4, 1, 5, 4, 2, 6, 4, 1, 8, 3, 4, 9, 2, 4, (c) 2 (d) 5
8, 3, 4, 8, 2, 8, 4, 5 ,4, 8, 7, 4, 6, 4, 5, 4 8. How many DLF are there in the
(a) 6 (b) 4 following series which have B in the
middle and the letter L on either side?
(c) 3 (d) 5
APF D LC O N LD F PAN T R LD
3. In the following series of letters, how NQRSFDONJKFDLEFGLD
many 'p' is preceded by 't' and followed F CAR F D LWX LDFYZ
by 'p'?
(a) 5 (b) 6
ptpttpptptpppqqptpttpppt
(c) 7 (d) 9
(a) 1 (b) 2
9. C2F, E5H, G12K, 127N, ?
(c) 3 (d) 4
(a) 158P (b) J 58Q
4. Which number appears least frequently
in the following number series? (c) K58Q (d) 157Q
846734378344563464348 10. P 3 C, R 5 F, T8I, V12 L, ?
(a) 8 (b) 7 (a) X17M (b) X17O
(c) 5 (d) 4 (c) Y17O (d) X16O
5. How many such '0' are there in the 11. 4E, 8I, 13N, 19T, ?

32
(a) 25U (b) 26A
(c) 26Z (d) 25Y ANSWER KEY

12. B2D, E3H, 14M, ? TYPE - I


1. (d) 2. (c) 3. (d) 4. (b) 5. (c)
(a) N5R (b) N5T
6. (a) 7. (a) 8. (c) 9. (b) 10. (c)
(c) N5S (d) N5Q 11. (b) 12. (c) 13. (b) 14. (a) 15. (b)
16. (c) 17. (c) 18. (c) 19. (b) 20. (c)
A D H M 21. (c) 22. (d) 23. (b) 24. (d) 25. (a)
13. , , , ,?
4 9 15 22 26. (d)
R S TYPE - II
(a) (b)
30 30 1. (d) 2. (c) 3. (a) 4. (d) 5. (c)
Q Q
(c) (d) 6. (b) 7. (b) 8. (c) 9. (c) 10. (b)
30 30
11. (b) 12. (a) 13. (d) 14. (c) 15. (d)
14. What is the value of ‘X’ in the sequence 16. (d)
2, 7, 22, 67, 202, X, 1822 ? TYPE - III
1. (d) 2. (d) 3. (a) 4. (c) 5. (b)
(a) 603 (b) 605 6. (b) 7. (a) 8. (b) 9. (b)

(c) 607 (d) 608 TYPE - IV


1. (a) 2. (c) 3. (c) 4. (b) 5. (c)
15. What is the value of X in the sequence 6. (b) 7. (b) 8. (b) 9. (b) 10. (c)
2, 12, 36, 80, 150, X ? 11. (a) 12. (b) 13. (b) 14. (d) 15. (c)
16. (c) 17. (d) 18. (d) 19. (b) 20. (d)
(UPSC-2022) 21. (a) 22. (c) 23. (d) 24. (b) 25. (a)
26. (d) 27. (b) 28. (c) 29. (b) 30. (d)
(a) 248
31. (b) 32. (b) 33. (b) 34. (c) 35. (d)
(b) 252
TYPE - V
(c) 258 1. (a) 2. (a) 3. (d) 4. (c) 5. (c)
6. (b) 7. (b) 8. (a) 9. (c) 10. (a)
(d) 262 11. (c) 12. (c) 13. (b) 14. (b) 15. (a)

16. Choose the group which is different TYPE - VI


from the others: 1. (b) 2. (d) 3. (c) 4. (d) 5. (d)
6. (c) 7. (c)
(UPSC-2023)
TYPE - VII
(a) 17, 37, 47, 97
1. (c) 2. (d) 3. (c) 4. (d) 5. (b)
(b) 31, 41, 53, 67 6. (d) 7. (a) 8. (b) 9. (c) 10. (b)
11. (b) 12. (c) 13. (b) 14. (c) 15. (d)
(c) 71, 73, 79, 83 16. (d) ****************************

(d) 83, 89, 91, 97

33
Explanation 8. (c) 313 623 933 1243 1553

TYPE - I 310 310 310 310

1. (d) 2460 3570 4680 5790 9. (b) 44 56 69 83 98 114

+12 +13 +14 +15 +16

+1110 +1110 +1110 10. (c) 975 864 753 642 531

ie ? 5790 will go in its place. +111 +111 +111 +111

2. (c) 3 15 35 63 99 11. (b) 8 24 12 36 18 54

×3 ÷2 ×3 ÷2 ×3
+12 +20 +28 +36
12. (b) 36 28 24 22 21
+8 +8 +8
–8 –4 –2 –1
3. (d) 4  2 + 2 =8+1=7
13. (b) 5255 5306 5357 5408 5459
10  2 + 2 = 20 + 2 = 22
+51 +51 +51 +51
22  2 + 2 = 44 + 2 = 46
14. (a) 12 36 108 324 972
46  2 + 2 = 92 + 2 = 94
×3 ×3 ×3 ×3
94  2 + 2 = 188 + 2 = 190
15. (b) 44, 40, 34, 26, 16, 4,
4. (b)
–3 –6 –8 –10 –12
2 5 10 17 26 37 50 65

+3 +5 +7 +9 +11 +13 +15


16. (c) 110, 99, 86, 71, 54, 35,

–11 –13 –15 –17 –19


Hence : '65' should come in place of '?'.
17. (c) 2, 3, 5, 6, 8, 9, 11, 12
5. (c) 7 22 37 52 67 82
+1 +2 +1 +2 +1 +2 +1
+15 +15 +15 +15 +15
18. (c) 98, 72, 50, 32, 18, 8
6. (a) 15 31 64 131 266
–26 –22 –18 –14 –10
2 + 1 2 + 2 2 + 3 2 + 4
19. (b)
7. (a) –4 –2

33 28 24 ? 19 18
20. (c)
–5 –3 –1

i.e. required number = 24 – 3 = 21


34
2 4 12 48 240 3. (a) +1.5 +1.5
21. (c)
2 3 4 5 4 8 12 24 36 72

Hence, the correct answer would be ×2 ×2 ×2


option (b).
4. (d) +12 +20 +28
22. (d) 6 17 39 72 116
1 2 3 14 5 34 7 62 9
+11 +22 +33 +44
+2 +2 +2 +2

23. (b) 2.1, 5.2, 8.4, 11.7, 15.1, 18.6


5. (c) +5
+3.1 +3.2 +3.3 +3.4 +3.5
18 25 23 30 28

24. (d) 2, 8, 40, 240, 1680, 13440


+5 +5
4 5 6 7 8

–6 –6 –6
25. (a) 5 11 17 25 33 43 53
6. (c)
2 65 7 59 12 53 17 47
+6 +6 +8 +8 +10 +10
+5 +5 +5

26. (d) 3 5 8 13 21 34 55 89
+3 +3
7. (c)
+2 +3 +5 +8 +13 +21 +34
22 28 25 31 28 34
+1 +2 +3 +5 +8 +13
+3 +3
+1 +1 +2 +3 +5
–2 –2
8. (c)
TYPE - II
87 90 84 88 81 86 78
1. (d) Two alternate number series are –3 –3 –3
given:
+6 +6
+2 +4 9. (c)
1 0 3 2 5 6 7 4 8 7 14 10 20 13

+2 +2 +3 +3 +3 +3

9. (b)
2. (c) +3 +3
–5 –7 –9
18 24 21 27 24 30 27
15 17 20 22 27 29 36 38
+3 +3 +3
+5 +7 +9

35
+2 +2 +2 3. (a) 0 7 26 63 124
10. (c)
97 86 99 88 101 90 103 +7 +19 +37 +61
+2 +2 +12 +18 +24
–2 –2
+6 +6

11. (a) 17 14 15 12 13 10 11
4. (c) 0 6 24 60 120 210
–2 –2 –2
+6 +18 +36 +60 +90
12. (d) +2 +2 +2
4 5 6 9 8 13 10 17 +12 +18 +24 +30
+4 +4 +4
+6 +6 +6
-5 -5
13. (c)
48 82 44 77 40 72 36 5. (b) 7 25 61 121 211

–4 –4 –4 +18 +36 +60 +90

+1 +1 +18 +24 +30


14. (d)
62 61 63 62 64 63 65 +6 +6
+1 +1 +1
6. (b) 4 18 48 100 180
+2 +2
+14 +30 +52 +80
12 8 14 6 16 4
15. (d)
+16 +22 +28
–2 –1
+6 +6
TYPE - III
7. (a) 3 8 18 35 61 98
1. (d) 0 4 18 48 100 180
+5 +10 +17 +26 +37
+4 +14 +30 +52 +80
+5 +7 +9 +11
+10 +16 +22 +28
+2 +2 +2
+6 +6 +6

8. (b) 1 18 17 30 49 76
2. (d) 56 90 132 184 248 326
+7 +9 +13 +19 +27
+34 +42 +52 +64 +78
+2 +4 +6 +8
+8 +10 +12 +14
+2 +2 +2
+2 +2 +2

36
9. (b) 7 25 61 121 211 9. (d) –1 –1 –1 –1

+18 +36 +60 +90 a d c f e h g j i

+3 +3 +3 +3
+18 +24 +30
10. (c) Here are two alternating series:
+6 +6
TYPE - IV I 
-2
 K 
+2
 M 
+6
 O 
+2
Q
W 
-2
 U 
-2
 S 
-2
Q
1. (a)
Hence, the required letter will be
2
a  3
 c  4
 f  5
 j  6
 o  u Q.
11. (c) The sequence of the given series is
2.
2 2 2 2
(c) T  R  P  N  as follows
2 2
L   J  H B F J N R V Z D H L P

+4 +4 +4 +4 +4 +4 +4 +4 +4 +4
3. (c) A 
+3
 D 
+4
 H 
+5
 M 
+6

12. (d) A  D  G  J 
+3 +3 +3 +3
S 
-7
Z
M 
+3
 P 
+3
S
4. (b) There are two alternating
categories: 13. (c) A 
+2
 C 
+3
 F 
+4
 J 
+5

O 
+6
 U 
+7
B
a  c  e  g  i
+2 +2 +2 +2

r 
+1
 s 
+1
 t 
+1
u 14. (a) –1 –1

A Z B Y C X D
5. (c) b c f g j k n
+1 +1 +1
+1 +3 +1 +3 +1 +3
15. (a) Here are two alternating series:
6. (b) D H L R X
A 
+2
 C 
+2
 E 
+2
 G 
+2
I
+4 +4 +6 +6
P 
+1
 Q 
+1
 R 
+1
R
7. (b) 16. (c)
A -4
 H 
+5
 M 
+6
 S
E 
-4
 I +5
 N 
+6
 T AB, DEF, HIJK, MNOPQ, STUVWX
F 
-4
 J 
+5
 O 
+6
 U +2 +2 +2

17. (c)
8. (c) A B D G K
+1
A +1 B +1 C D +1 E +1 F
+1 +2 +3 +4
O +1 M –2 K –2 I –2 G –2 E

37
18. (c)
28. (c) A +2 C +2 E +2
+1 G
A +1 B +1 C D +1 E +1 F –2
Z –2 X –2 V T
O +1 M –2 K –2 I –2 G –2 E +2
B +2 D +2 F H
–2 –2 U –3
Y W S
19. (d) A  F  K
+5 +5

C 
+5
 H 
+5
M
+2 G +2 I +2
29. (b) E K
20. (c) A  F  K  P A +2 C +2 E +2 G
+5 +5 +5

E   J   O  T C +2 E +2 G +2 I
+5 +5 +5

A +4 E +4 I +4 M +4 Q
30. (b) B +1 C +2 E +3 H
21. (c) D +2 F +3 I +4 M
D +4 H +
+4 L +4 P +4 T
F +3 I +4 M +5 R
22. (b) A +1 B +1 C +1 D +1 E
–1 –1 Q –1
31. (c) S R P
N +1 O +
+1 P +1 Q +1 +1
F +1 G +1 H I
+1 +1 +1
+3 J K L M
23. (d) C +3 F +3 I L
A +3 D +3 G +3 J +1 +1 D +1
32. (a) B C E
T +3 W +3 Z +3 C D +2 F +2 H
+2
J
+3
F +3 I +3 L O
24. (b) D +7 K +7 R +7 Y

F +7 M +7 T +7 A +1 J +1 K +1 L +1 M
33. (c) I
I +7 P +7 W +7 D C +1 D +1 E +1 F +1 G
E +1 F +1 G +1 H +1 I
+2
25. (a) D +2 P +2 R T
-2 Q
O +2 Q +2 S +2 U 34. (d) Y -2 W -2 U -2 S
A +1 B +1 C +1 D E +1 F +1 H +3 K +4 O
B +2 D +3 G +5 H +0 L
+2 M +2 O +2 Q
26. (d) K
+3 M 35. (a) B +3 E +3 H +3 K
D +3 G +3 J
W –3 T –3 Q –3 D M +2 O +2 Q +2
+2
O +2 Q +2 S U
27. (b) In each subsequent term, the last
letter becomes the first letter. TYPE - V

1. (a) H I JH / H IJ H / H I J H / H I JH

38
2. (a) c c a c c b / c c a c c b / cc a c c b 3. (a)
41 41 42 44 47 51 56
 acac
+0 +1 +2 +3 +4 +4
3. (d) a b a a / a b aa / a b a a / ab a a
Hence the number 48 from the
4. (d) a bcc b a / abc c ba / ab c
series is wrong.
5. (c) bca a / b c aa / bc a a / b caa
4. (d) 10 14 19 25 32 40
6. (b) a b b b ba / a b b b ba +4 +5 +6 +7 +8

7. (b) an a / nn a / ana / n na / a na / n n a Hence, the number '9' from the


series is wrong.
8. (a) c a b d / c a b d / c a b d / c a b d 
5. (c) 5  2 + 1 = 111
dadc
11  2 + 1 = 23
9. (c)
a a c/ c b b / a a c / c b b / a a c / c b b 23  2 + 1 = 47

47  2 + 1 = 95
10. (a) a b c d d / a b c d d / a b c d d / a b c
Hence, the number 96 is the wrong
11. (c) an d / an d / a nd/ a n d / and / a number.
12. (c) a b b / a b b / a b b / a b b 6. (a) Except the number 36, all other odd
numbers are perfect square.
13. (b) aa b / a a b / a ab / aa b
25 = 5  5; 49 = 7  7;
14. (b) a c c / a a c / a c c / a a c 81 = 9  9; 141 = 11  11;
1;

15. (a) A Z B 169 = 13  13;


Y / A Z B Y / A Z BY/
A Z B Y 225 = 15  15

but,
TYPE - VI
36 = 6  6
1. (b) 216, 168, 120, 72, 24
7. (a) 60 – 14 = 46

–48 –48 –48 –48 46 – 12 = 34


34 – 10 = 24
2. (c) 3 6 11 18 27 38
24 – 8 = 16
+3 +5 +7 +9 +11
16 – 6 =10
+2 +2 +2 +2
Thus, the number 62 is wrong in
the series.
39
8. (b) A P F D L C O N L D F P A N T R L D
TYPE - VII
NQRSFDOMJKF D LEFGL D
1. (c) From the given number series, F CAR F D LW X L D F Y Z

3 8 25 832832827 8 485687 8 4
9. (c) C +2 E +2 G +2 I
+2
K
286
+15 +31
2 +3 5 +7 12 27 58
Hence, the number of required 8 is +3 +3
E +3 H +3 K N Q
three.
2. (c) From the given series, +2 R +2 T +2 +2
10. (b) P V X
+4 +5
414154264183 4 92 4 83 4 828 3 +2 5 +3 8 12 17
+3 +3
45 4 87 4 6454 C +3 F +3 I L O

Hence, the required number of


11. (b) 4 +4 5 +5 13 +6 19 +7 26
digits of 4 is 5.
+6 +7
E +4 I +5 N T A
3. (c) There are only three letters in the
question.
12. (b) B +3 E +4 I +5 N
4. (d) 3  5 Time, 4  7 Time, 2 +1 3 +1 4
+1
5
+6 S
6  3 Time, 7  2 Time, D +4 H +5
M

5  1 Time, 8  3 Time, +5
13. (b) A +3 D +4 M +6 S
5. (b) 302 503203202 704 0580 704 4 5 8 12 30
+5 +6 +7 +8
208
6. (d) Z A B

is such a combination

AMBZA NAAB ZAB AZBA


P ZAB A ZAB

7. (a)

784 386 784

382

40
6
 Read the following information carefully and (a) R
answer the questions that follow : (b) S
Five friends P, Q, R, S and T traveled to five (c) T
different cities of Chennai, Kolkatta, Delhi, (d) Data inadequate
Banglore and Hyderabad by different modes (e) None of these
of transport of Bus, Train, Aeroplane, Car and 5. Which of the following combination of place
Boat from Mumbai. and mode is not correct ?
I. The person who traveled to Delhi did not (a) Delhi – Bus
travel by boat.
(b) Kolkatta – Aeroplane
II. R went to Banglore by car and Q went to
(c) Banglore – Car
Kolkatta by aeroplane.
(d) Chennai – Boat
III. S travelled by boat whereas T traveling
(e) Hyderabad – Bus
by train.
 Read the following information carefully and
IV. Mumbai is not connected by bus to Delhi
answer the questions that follow :
and Chennai.
There are six persons A, B, C, D, E and F in a
1. Which of the following combinations of
school. Each of the teachers teaches two
person and mode is not correct ?
subjects, one compulsory subject and the other
(a) P – Bus
optional subject.
(b) Q – Aeroplane
I. D’s optional subject was History while
(c) R – Car three others have it as compulsory
(d) S – Boat subject.
(e) T –Aeroplane II. E and F have physics as one of their
2. Which of following combinations is true for subjects.
S? III. F’s compulsory subject is Mathematics
(a) Delhi – Bus which is an optional subject of both C
(b) Chennai – Bus and E.
(c) Chennai – Boat IV. History and English are A’s subject but
(d) Data inadequate in terms of compulsory and optional
(e) None of these subjects. They are just reverse of those
3. The person traveling to Delhi went by which of D’s.
of the following modes ? V. Chemistry is an optional subject of only
(a) Bus one of them.
(b) Train VI. The only female teacher in the School has
(c) Aeroplane English as her compulsory subject.
(d) Car 6. What is C’s compulsory subject?
(e) Boat (a) History
4. Who among the following traveled to Delhi. (b) Physics

41
(c) Chemistry 5. A is married to D, and E is not one of the
(d) English two non-working housewives.
(e) Mathematics Based on the above, answer the next THREE
7. Which of the following has same compulsory items:
and optional subject as those of F’s ? 10. How many members of the groups are males?
(a) D (a) 2
(b) B (b) 3
(c) A (c) 4
(d) C (d) Connot be determined
(e) None of these 11. What is E working as?
8. D is regarding which is the compulsory and (a) Lawyer
which is the optional subject, who has the (b) Architect
same to subject combination as F ? (c) Lecturer
(a) A (d) Accountant
(b) B 12. Which of the following is a married couple?
(c) E (a) A-B
(d) D (b) A-F
(e) None of these (c) C-E
9. Which of the following groups has History as (d) None of the obove
the compulsory subject ? 13. Who is a female member in the group?
(a) A, C, D (a) A
(b) B, C, D (b) B
(c) C, D (c) C
(d) A, B, C (d) D
(e) A, D (e) E
For the next THREE items  Read the following information carefully and
In a group of six men and women- answer the questions given below :
A,B,C,D,E and F there are two housewives I. Ravi and Kapil are good in Hockey and
who have not taken up any job, one lecturer, Vollyball,
one architect, one accountant and one II. Sachin and Ravi are good in Hockey and
lawyere. Further Baseball.
1. There are two married couples in the III. Gaurav and Kapil are good in Cricket and
group. volleyball.
2. The lawyer is married to D who is IV. Sachin, Gaurav and Micheal are good in
housewife and she has not taken up any Football and Baseball.
job. 14. Who is good in Hockey, Cricket and
3. No lady in the group is either an Volleyball?
accountant or an architect. (a) Sachin
4. C who is an accountant is married to F (b) Kapil
who is a lecturer.
(c) Ravi
(d) Gaurav

42
15. Who is good in Baseball, Cricket Vollyball persons taller and two persons shorter than him
and Football ? / her ?
(a) Sachin (a) A
(b) Kapil (b) B
(c) Gaurav (c) C
(d) Ravi (d) D
16. Who is good in Baseball, Vollyball and  Five children were administered psychological
Hockey ? tests to know their intellectual levels. In the
(a) Sachin report, psychological pointed out than the
(b) Kapil I. Child A is less intelligent than the child
(c) Ravi B.
(d) Gaurav II. The child C is less intelligent than the
 Read the information given below to answer child D.
these questions : III. The child B is less intelligent than the
I. Aarti is older than Sanya. child C and
II. Muskan is elder than Aarti but younger IV. child A is more intelligent than the child
than Kashish. E.
III. Kashish is elder than sanya. 21. Which child is the most intelligent?
IV. Sanya is younger than Muskan. (a) A
V. Gargi is the eldest. (b) B
17. Who is the youngest ? (c) D
(a) Kashish (d) E
(b) Aarti (e) None of these
(c) Muskan 22. In the shop, there were 4 dolls of different
(d) Sanya heights A, B, C and D. D is neither as tall as A
nor as short as C. B is shorter than D but taller
18. Agewise, who is in the middle ?
than C. If we wants to purchase the tallest doll,
(a) Kashish
which one should she purchase ?
(b) Aarti
(a) Only A
(c) Muskan
(b) Only D
(d) Sanya
(c) Either A or D
19. Which of the given statements is superfluous
(d) Either B or D
and can be dispensed with while answering
(e) None of these
the above questions ?
23. If (i) P is taller than Q, (ii) R is shorter than P,
(a) Either (i) or (iii)
(iii) S is taller than T but shorter than Q. then
(b) Only (iv)
who among them is the tallest ?
(c) Either (i) or (iv)
(a) P
(d) Both (iii) and (iv)
(b) Q
20. A, B, C, D and E are five friends. A is shorter
(c) S
than B but taller than E. C is the tallest. D is
(d) T
shorter than B and taller than A. Who has two

43
24. In the examination, Raj got more marks than IV. Ashok is taller than Jayesh.
Mukesh but not as many as Priya. Priya got 28. Who among them is the tallest ?
more marks than Gaurav and Kavita. Gaurav (a) Gopal
got less marks than Mukesh but his marks are (b) Ashok
not the lowest in the group. Who is second in (c) Kunal
the descending order of marks ?
(d) Navin
(a) Priya
(e) Jayesh
(b) Kavita
29. Which of the given information is not
(c) Raj sufficient to answer the above question ?
(d) Cannot be determined (a) A
(e) None of these (b) B
25. Daksh is taller than manick but not as tall as (c) C
Rohan. Somesh is shorter than Daksh but taller
(d) D
than Farhan. Who among them is the shortest
(e) None of these
?
30. Five boys took part in a race. Raj finished
(a) Daksh
before Mohit but behind Gaurav. Ashish
(b) Manick
finished before Sanchit but behind mohit. Who
(c) Farhan won the race ?
(d) Cannot be determined (a) Raj
26. Garima is taller than Sarita but not taller than (b) Gaurav
Reena. Reena and Tanya are of the same
(c) Mohit
height. Garima is shorter than Anu. Amongst
(d) Ashish
all the girls, who is the shortest?
These question are based on the following
(a) Anu
information :
(b) Reena and Tanya
Five men A, B, C, D and E read a newspaper.
(c) Garima
The one who read first gives it to C. The one
(d) Sarita
who reads last had taken from A. E was not
27. D is taller than C and E. A is not as tall as E. C
the first or last to read. There were two readers
is taller than A. D is not as tall as B. Who
between B and A.
among them is next to the tallest one ?
31. B Passed the newspaper to whom ?
(a) A
(a) A
(b) D
(b) C
(c) B & D
(c) D
(d) Data inadequate
(d) E
(e) None of these
(e) None of these
 Read the following information carefully and
32. Who read the newspaper last ?
answer the questions given below it:
(a) A
I. Gopal is shorter than Ashok but taller than
Kunal. (b) B
II. Navin is shorter than Kunal. (c) C
III. Jayesh is taller than Navin. (d) D
(e) None of these

44
 Study the following information carefully (d) B, C, P, Q, R, T
and answer the question given below it. Directions for the following 3 (three) items:
From amongst six boys A, B, C, D, E and F Read the following passage and answer the
and five girls P, Q, R, S and T a team of six is three items that follow:
to be selected under the following conditions: A tennis coach is trying to put together a
I. A and D have to be together. team ,of our players for the forthcoming
II. C cannot go with S. tournament. For this 7 players are available
III. S and T have to be together. : males A, B and C; and females W, X, Y
IV. B cannot be teamed with E. and Z. All players have equal capability and
V. D cannot go with P. at least 2 males will be there in the team.
VI. B and R have to be together. For a team of four, all players must be able
to play with each other. But, B cannot play
VII.C and Q have to be together.
with W, C cannot play with Z and W cannot
33. If there are five boys in the team, the alon girl
play with Y.
member is
(UPSC-2013)
(a) P
38. If Y is selected and B is rejected, the team
(b) Q
will consist of which one of the following
(c) R groups?
(d) S (a) A, C , W and Y
34. If including P, The team has three girls, the (b) A, C , X and Y
member are
(c) A, C , Y and Z
(a) B, C, F, Q, R
(d) A, W , Y and Z
(b) A, D, E, S, T
39. If B selected and Y is rejected, the team will
(c) A, D, B, S, T consist of wich one of the following groups?
(d) B, F, R, S, T (a) A, B , C and W
35. If the team including C consists of four boys, (b) A, B , C and Z
the member of the team other than C are
(c) A, B , C and X
(a) A, D, E, P, Q
(d) A, W , Y and Z
(b) A, B, D, Q, R
40. If all the three males are selected, then how
(c) D, E, F, A, Q many combinations of four member teams are
(d) B, E, F, R, Q possible?
36. If four members including E have to be boys, (a) 1
the members other than E are (b) 2
(a) A, B, C, Q, R (c) 3
(b) A, D, F, S, T (d) 4
(c) B, C, F, Q, R 41. A,B,C,D, and E belong to five different cities
(d) A, C, D, F, Q P,Q,R,S and T (not necessarily in that order).
37. If four members have to be girls, the members Each one of them comes from a different city.
of the team are Further it is given:
(a) B, C, p, Q, R, S (CPF-2004)
(b) B, F, P, R, S, T 1. B and C do not belong to Q.
(c) B, C, Q, R, S, T 2. B and E do not belong to P and R.
45
3. A and C do not belong to R, S and T. (UPSC-2012)
4. D and E do not belong to Q and T. (a) Sita is not as pretty as Gita.
Which one of the following statements is not (b) Sita is prettier than Rita.
correct? (c) Rita is not as pretty as Gita.
(a) C belongs to P (d) Gita is prettier than Rita.
(b) D belongs to R 45. Given that ,
(c) A belongs to Q (UPSC-2012)
(d) B belongs to S 1. A is the brother of B.
42. Three persons A, B and C wore shirts of black, 2. C is the father of A.
blue and orange colours (not necessarily in that 3. D is the brother of E.
order) and pants of green, yellow and orange 4. E is the daughter of B.
colour (not necessarily in that order). No Then, the uncle of D is
person wore shirt and pant of the same colour.
(a) A
Further, it is given that
(b) B
(CPF-2005)
(c) C
1. A did not wear shirt of black colour.
(d) E
2. B did not wear shirt of blue colour.
46. Examine the following statements:
3. C did not wear shirt of orage colour.
(UPSC-2012)
4. A did not wear pant of green colour.
1. Rama scored more than Rani.
5. B wore pant of orange colour.
2. Rani scored less than Ratna.
What were the colours of the pant and shirt
3. Ratna scored more than Rama.
won by C respectively?
4. Padma scored more than Rama but less
(a) Orange and black
than Ratna.
(b) Green and blue
Who scored the highest?
(c) Yellow and blue (a) Rama
(d) Yellow and black (b) Padma
43. Examine the following statements: (c) Rani
(UPSC-2012) (d) Ratna
1. Either A and B are of the same age or A 47. Only six roads A, B, C, P, Q and R connect a
is older than B. military camp to the rest of the country. Only
2. Either C and D are of the same age or D one out of A, P and R is open at any one time.
is older than C. If B is closed, so is Q. Only one of A and B is
3. B is older than C. open during storms. P is closed during floods.
Which one of the following conclusion can In this context, Which one of the following
be drawn from the above statements? statements is correct?
(a) A is older than B (UPSC-2012)
(b) B and D are of the same age (a) Under normal conditions only three roads
(c) D is older than C are open.
(d) A is older than C (b) During storms at least one road is open.
44. Gita is prettier than Sita but not as pretty as (c) During floods only three roads are open.
Rita, Then, (d) During calamities all roads are closed.

46
48. Four political parties W,X,Y and Z decided to 50. Which lecture is scheduled between Statistics
set up a joint candidate for the coming and English?
parliamentary elections. The formula agreed (a) Economics
by them was the acceptance of a candidate by (b) History
most of the parties. Four aspiring candidates, (c) Mathematics
A, B, C and D approached the parties for their (d) No lecture
tickets.
51. Which lecture is the last one the week ?
(UPSC-2012)
(a) History
1. A was acceptable to W but not to Z.
(b) English
2. B was acceptable to Y but not to X.
(c) Mathematics
3. C was acceptable to W and Y.
(d) Economics
4. D was acceptable to W and X.
52. Which lecture is scheduled on Wednesday?
When candidate B was preferred by W and Z,
(a) Statistics
candidate C was preferred by X and Z, and
(b) Economics
candidate A was acceptable to X but not to Y,
who got the ticket ? (c) English
(a) A (d) History
(b) B 53. Which lecture is scheduled before the
Mathematics lecture?
(c) C
(a) Economics
(d) D
(b) History
Directions for the following 5 (five) items:
(c) Statistics
Examine the information given in the
following paragraph and answer the items (d) English
that follow: 54. Examine the following statements:
Guest lectures on five subjects viz., (UPSC-2014)
Economics, History, Statistics, English and 1. George attends Music classes on Monday.
Mathematics have to be arranged in a week 2. He attends Mathematics classes on
from Monday to Friday. Only one lecture can Wednesday.
be arranged on each day. Economics cannot 3. His Literature classes are not on Friday.
be scheduled on Tuesday. Guest faculty for 4. He attends History classes on the day
History is available only on Tuesday. following the day of his Mathematics
Mathematics lecture has to be scheduled classes.
immediately after the day of Economics 5. On Tuesday, he attends his Sports classes.
lecture. English lecture has to be scheduled If he attends just one subject in a day and his
immediately before the day of Economics Sunday is free, then he is also free on
lecture. (a) Monday
(UPSC-2012) (b) Thursday
49. Which lecture is scheduled on Monday ? (c) Saturday
(a) History (d) Friday
(b) Economics Directions for the following 4 (four) items:
(c) Mathematics Read the following statements and answer the
(d) Statistics four items the follow:

47
(UPSC-2013) Rohit, Tanya, Shobha, Kaushal, Kunal,
Five cities P, Q, R, S and T are connected by Mukesh and Jaswant.
different modes of transport as follows: Rohit and Tanya will not work together. Kunal
P and Q are connected by boat as well as rail. and Shobha will not work together. Mukesh
S and R are connected by boat as well as rail. and Kunal want to work together.
Q and T are connected by air only Which of the following is the most acceptable
P and R are connected by boat only group of people that can be selected by the
T and R are connected by rail and bus. music director?
55. Which mode of transport would help one to (UPSC-2013)
reach R starting from Q, but without changing (a) Rohit, Shobha, Kunal and Kaushal
the mode of transport? (b) Tanya, Kaushal, Shobha and Rohit
(a) Boat (c) Tanya, Mukesh, Kunal and Jaswant
(b) Rail (d) Shobha, Tanya , Rohit and Mukesh
(c) Bus Directions for the following 3 (three) items:
(d) Air Examine carefully the following statements
56. If a person visits each of the places starting and answer the three items that follow:
form P and gets back to P, Which of the (UPSC-2013)
following places must he visit twice? Out of four friends A, B, C and D.
(a) Q A and B play football and cricket.
(b) R B and C play cricket and hockey,
(c) S A and D play basketball and football,
(d) T C and D play hockey and basketball,
57. Which one of the following pairs of cities is 60. Who does not play hockey?
connected by any of the routes directly without (a) D
going to any other city? (b) C
(a) P and T (c) B
(b) T and S (d) A
(c) Q and R 61. Who plays football, basketball and hockey?
(d) None of these (a) D
58. Between which two cities among the pairs of (b) C
cities given below are there maximum travel (c) B
options available? (d) A
(a) Q and S 62. Which game do B, C and D play?
(b) P and R (a) Basketball
(c) P and T (b) Hockey
(d) Q and R (c) Cricket
59. The music director of a film wants to select (d) Football
four persons to work on different aspects of 63. Geeta is older than her cousin Meena. Meena’s
the composition of a piece of music. Seven brother Bipin is older than Geeta. When meena
persons are available for this work; they are and Bipin visit Geeta, they like to play chess.
Meena wins the game more often than Geeta.

48
Based on the above information, four If they are made to sit in the above order of
conclusions, as given below, have been made. their height, who will occupy the mid
Which one of these logically follows from the position?
information given above? (a) A
(UPSC-2013) (b) B
(a) While playing chess with Geeta and (c) C
Meena, Bipin often loses. (d) D
(b) Geeta is the oldest among the three. Directions for the following 3 (three) items:
(c) Geeta hates to lose the game. Read the passage given below and answer the
(d) Meena is the youngest of the three. items that follow.
64. Seven men, A, B, C, D, E, F and G are standing A,B,C,D,E,F are members of a family They
in a queue in that order. Each one is wearing a are engineer, stenographer, doctor,
cap of a different colour like violet, indigo, draughtsman, lawyer and judge (not in order).
blue, green, yellow, orange and red. D is able A, the engineer is married to the lady
to see in front of him green and blue, but not stenographer. The judge is married to the
violet. E can see violet and yellow, but not lawyer. F, the draughtsman is the son of B and
red. G can see caps of all colours other than brother of E. C, the lawyer is the daughter-in-
orange. If E is wearing an indigo coloured cap, law of D. E is the unmarried doctor. D is the
then the colour of the cap worn by F is grandmother of F. There are two married
(UPSC-2013) couples in the family.
(a) Blue (UPSC-2014)
(b) Violet 67. What is the profession of B?
(c) Red (a) Judge
(d) Orange (b) Lawyer
65. There are seven persons up on a ladder, (c) Draughtsman
A,B,C,D,E, F and G (not in that order). A is (d) Cannot be determined
further up than E but is lower than C. B is in 68. Which of the following is/are a couple/
the middle. G is between A and B. E is between couples?
B and F. If F is between E and D, the person (a) AD only
on the bottom step of the ladder will be (b) BC only
(UPSC-2014) (c) Both AD and BC
(a) B (d) Both AC and BD
(b) F 69. What is the profession of D?
(c) D (a) Judge
(d) E (b) Stenographer
66. Consider that: (c) Doctor
(UPSC-2014) (d) Cannot be determined
1. A is taller than B. 70. Two men, Anil and David, and two women
2. C is taller than A. Shabnam and Rekha are in a sales group. Only
3. D is taller than C. two speak Tamil. The other two speak Marathi.
4. E is the tallest of all. Only one man and one woman can drive a car.

49
Shabnam speaks Marathi. Anil speaks Tamil. contains volleyball. The box containing the
Both Rekha and David can drive. hockey ball is between the boxes containing
Which of the following Statements is true? golf ball and volleyball.
(UPSC-2015) 72. Which one of the following boxes contains
(a) Both the Tamil speakers can drive a car. the golf ball?
(b) Both the Marathi speakers can drive a car. (a) F
(c) Both of those who can drive a car speak (b) E
Marathi. (c) D
(d) One of those who can drive a car speaks (d) None of the above
Tamil. 73. Which of the following statements is/are
71. A ate grapes and pineapple; B ate grapes and correct?
oranges; C ate oranges, pineapple and apple; (a) D is painted yellow
D ate grapes, apple and pineapple. After taking (b) F is painted indigo
fruits, B and C fell sick, In the light of the (c) B is painted blue
above facts, it can be said that the cause of (d) All of the above
sickness was: 74. The football is in the box of which colour?
(UPSC-2016) (a) Yellow
(a) Apple (b) Indigo
(b) Pineapple (c) Connot be determined as data are
(c) Grapes inadequate
(d) Oranges (d) Blue
Direction for the following 3 (three) items: Direction for the following 3 (three) items :
Consider the given information and answer Consider the given information and answer
the three items that follow. the three items that follow.
(UPSC-2016) (UPSC-2016)
Six boxes A, B, C, D, E and F have been When three friends A, B met, it was found
painted with six dirfferent colours viz., violet, that each of them wore an outer garment of a
indigo, blue, green, yellow and orange and different colour. In random order, the garments
arranged from left to right (not necessarily are: jacket, sweater and tie; and the colours
either kept or painted with the colours in the are: blue white and black. Their surnames in
same order). Each box contains a ball of any random order are: Ribeiro, Kumar and Singh.
one of the following six games: cricket, Further, we know that:
hockey, tennis, golf, football and volleyball 1. Neither B nor Ribeiro wore a white
(not necessarily in the same order). The golf sweater
ball is in violet box and is not in the box D.
2. C wore a tie
The box A which contains tennis ball is orange
3. Singh’s garment was not white
in colour and is the extreme right. The hockey
ball in neither in box D nor in box E. The box 4. Kumar does not wear a jacket
C having cricket ball is painted green. The 5. Ribeiro does not like to wear the black
hockey ball is neither in the box painted blue colour
nor in the box painted yellow. the box C is 6. Each of the friends wore only one outer
fifth from right and next to box B. The B garment of only one colour

50
75. What is C’s surname? (a) B
(a) Ribeiro (b) D
(b) Kumar (c) E
(c) Singh (d) Cannot be determined as data are
(d) Cannot be determined inadequate
76. What is the colour of the tie? 79. To which city does the Lecturer specialized
(a) Black in Economics belong?
(b) Blue (a) Hyderabad
(c) White (b) Mumbai
(d) Cannot be determined (c) Neither Hyderabad nor Mumbai
77. Who wore the sweater? (d) Cannot be determined as data are
(a) A inadequate
(b) B 80. Who of the following belongs to Hyderabad?
(c) C (a) B
(d) Cannot be determind (b) E
Directions for the following 3 (three) items: (c) Neither B nor E
Consider the given information and answer (d) Cannot be determined as data are
the three items that follow. inadequate
(UPSC-2017) Direction for the following 6 (six) items:
A, B, C, D, E, F and G are Lecturers from Read the information given below and answer
different cities- Hyderabad, Delhi, Shillong, the six items that follow.
Kanpur Chennai, Mumbai and Srinagar (not (UPSC-2018)
necessarily in the same order) who participated A, B, C and D are students. They are studying
in a conference. Each one of them is in four different cities, viz., P, Q, R and S (not
specialized in a different subject, viz., necessarily in that order). They are studying
Economics, Mathematics and Statistics (not in Science college, Arts college, Commerce
necessarily in the same order). Further college and Engineering college (not
1. Lecturer from Kanpur is specialized in necessarily in that order), which are situated
Geography in four different States, viz., Gujarat,
2. Lecturer D is from Shillong Rajasthan, Assam and Kerala (not necessarily
3. Lecturer C from Delhi is specialized in in that order). Further, it is given that
Sociology 1. D is studying in Assam
4. Lecturer B is specialized in neither 2. Arts college is located in city S which is
History nor mathematics in Rajasthan
5. Lecturer A who is specialized in 3. A is studying in Commerce college
Economics does not belong to Hyderabad 4. B is studying in city Q
6. Lecturer F who is specialized in 5. Science college is located in Kerala
commerce belongs to Srinagar 81. A is studying in
7. Lecturer G who is specialized in Statistics (a) Rajasthan
belongs to Chennai (b) Gujarat
78. Who is specialized in Geography? (c) city Q

51
(d) Kerala higher than A.
82. Science college is located in 87. If F stands second in the ranking, then the
(a) city Q position of B is
(b) city S (a) Third
(c) city R (b) Fourth
(d) city P (c) Fifth
83. C is studying in (d) Sixth
(a) Science college 88. If B scores the least, the rank of C will be
(b) Rajasthan (a) Second
(c) Gujarat (b) Third
(d) city Q (c) Fourth
84. Which one of the following statements is (d) Second or third
correct? 89. If E is ranked third, then which one of the
(a) D is not studying in city S. following is correct ?
(b) A is studying in Science college. (a) E gets more marks than C
(c) A is studying in Kerala. (b) C gets more marks than E
(d) Engineering college is located in Gujarat. (c) A is ranked fourth
85. Which one of the following statements is (d) D is ranked fifth
correct regarding Engineering college? 90. Half of the villagers of a certain village have
(a) C is studying there. their own houses. One-fifth of the villagers
(b) B is studying there. cultivate paddy. One-third of villagers are
(c) It is located in Gujarat. literate. Four-fifth of the villagers are below
(d) D is studying there. twenty five. Then, which of the following is
certainly true?
86. Which one of the following statements is
correct? (UPSC-2021)
(a) Engineering college is located in Assam. (a) All the villagers who have their own
houses are literate.
(b) City Q is situated in Assam
(b) Some villagers under 25 years of age are
(c) C is studying in Kerala.
literate.
(d) B is studying in Gujarat.
(c) Only half of villagers who cultivate paddy
Direction for the following 3 (three) items:
are literate.
Read the information given below and answer
(d) No villager under 25 years of age has his
the three items that follow.
own house.
(UPSC-2019)
91. Eight students A, B, C, D, E, F, G and H sit
Six students A, B, C, D, E and F appeared in around a circular table, equidistant from each
several tests. Either C or F scores the highest. other, facing the centre of the table, not
Whenever C scores the highest, then E scores necessarily in the same order. B and D sit
the least. Whenever F scores the highest, B neither adjacent to C nor opposite to C. A sits
scores the least. in between E and D, and F sits in between B
In all the tests they got different marks; and H. Which one of the following is definitely
D scores higher than A, but they are close correct?
competitors; A scores higher than B; C scores (UPSC-2022)
52
(a) B sits in between A and G
Answer Sheet
(b) C sits opposite to G
(c) E sits opposite to F 1. (e) 39. (c) 77. (d)
(d) None of the above 2. (c) 40. (a) 78. (b)
92. A person X wants to distribute some pens 3. (b) 41. (b) 79. (c)
among six children A, B, C, D, E and F. 4. (c) 42. (d) 80. (d)
Suppose A gets twice the number of pens 5. (a) 43. (c) 81. (b)
received by B, three times that of C, four times 6. (b) 44. (a) 82. (c)
that of D, five times that of E and six times 7. (a) 45. (a) 83. (c)
that of F. What is the minimum number of pens 8. (a) 46. (c) 84. (a)
X should buy so that the number of pens each 9. (d) 47. (d) 85. (d)
one gets is an even number? 10. (a) 48. (d) 86. (a)
(UPSC-2022) 11. (b) 49. (c) 87. (b)
(a) 147 (b) 150 12. (b) 50. (d) 88. (d)
(c) 294 (d) 300 13. (b) 51. (d) 89. (a)
93. How many distinct 8-digit numbers can be 14. (d) 52. (d) 90. (c)
formed by rearranging the digit of the number 15. (a) 53. (e) 91. (d)
11223344 such that odd digits occupy odd
16. (d) 54. (a) 92. (a)
positions and even digits occupy even
17. (e) 55. (e) 93. (c)
positions?
18. (c) 56. (d) 94. (c)
(UPSC-2023)
19. (d) 57. (d) 95. (c)
(a) 12 (b) 18
20. (d) 58. (d) ********
(c) 36 (d) 72
21. (c) 59. (b)
94. ABCD is a square. One point on each of AB
22. (c) 60. (c)
and CD; and two distinct points on each of
23. (b) 61. (c)
BC and DA are chosen. How many distinct
triangles can be drawn using any three points 24. (d) 62. (b)
as vertices out of these six points? 25. (b) 63. (d)
(UPSC-2023) 26. (a) 64. (b)
(a) 16 (b) 18 27. (b) 65. (a)
(c) 20 (d) 24 28. (e) 66. (b)
95. The letters of the word 29. (a) 67. (b)
“INCOMPREHENSIBILITIES” are arranged 30. (c) 68. (b)
alphabetically in reverse order. How many 31. (c) 69. (b)
positions of the letter/letters will remain 32. (a) 70. (c)
unchanged? 33. (a) 71. (b)
(UPSC-2023) 34. (c) 72. (d)
(a) None (b) One 35. (d) 73. (b)
(c) Two (d) Three 36. (d) 74. (d)
37. (a) 75. (a)
38. (b) 76. (a)

53
1.
7
Five persons are standing in a line. One of the right end. 12. How many persons are there to
two persons at the extreme ends is a professor the right of D?
and the other a businessman. An advocate is
(a) One (b) Two
standing to the right of a student. An author is
to the left of the businessman. The student is (c) Three (d) Four
standing between the professor and the 6. Which of the following pairs is sitting to one
advocate. Counting from the left. the advocate side of D?
is at which place? (a) FB
(a) 1st (b) 2nd (b) FC
(c) 3rd (d) 5th (c) FE
Directions (Questions 2 to 4) : Read the (d) Cannot be determined
following information to answer the given
Seven children A, B, C, D, E, F and G are (e) None of these
standing in a line. G is to the right of D and to 7. Who is immediate left of C?
the left of B. A is on the right of C. A and D (a) A
have one child between them. E and B have
(b) E
two children between them. D and F have two
children between them. (c) Either E or A
2. Who is on the extreme right? (d) Cannot be determinded
(a) B (b) E (e) None of these
(c) F (d) G 8. Who is at the right end?
3. Who is exactly in the middle? (a) A
(a) A (b) C (b) B
(c) D (d) E (c) E
4. Who is on the extreme left? (d) Cannot be determined
(a) A (b) B (e) None of these
(c) C (d) D Directions (Questions 9 to 11): Study the
following information to answer the given
A Directions (Questions 5 to 8) : Study the
questions:
following information carefully and answer
the questions given below: P, Q, R, S, T, U, V and W are eight friends
Six friends A, B, C, D, E and F are sitting sitting around a circle facing towards the
in a row facing towards North. C is sitting centre
between A and E. D is not at the end. B is (i) W is on the immediate left of P but is not
sitting immediate right to E., F is not at the the neighbour of T or S.

54
(ii) U is on the immediate right of Q and V is (c) FE
the neighbour of T. (d) Cannot be determined
(iii) R is between T and U. (e) None of these
14. Who is immediate left of C ?
9. Which of the following statements is true?
(a) A
(a) T is between U and Q.
(b) E
(b) U is the neighbour of V.
(c) Either E or A
(c) V is between W and T.
(d) Cannot be determined
(d) W is between P and S.
(e) None of these
(e) None of these
15. Who is at the right end ?
10. What is the position of S? (a) A
(a) On the immediate left of Q (b) B
(b) Second to the right of U (c) E
(c) Between Q and U (d) Cannot be determined
(d) On the immediate left of P (e) None of these
(e) None of these  Study the following information to answer the
given questions :
11. What is the position of V?
P, Q, R, S, T, U, V and W are eight friends
(a) Second to the left of S
sitting around a circle facing towards the centre.
(b) Third of the right of U I. W is on the immediate left of P but is not
(c) On the immediate right of W the neighbour of T or S.
(d) Between T and R II. U is on the immediate right of Q and V is
the neighbour of T.
(e) None of these
 Study the following information carefully and III. R is between T and U.
answer the questions given below: 16. Which of the following statements is true?
Six friends A, B, C, D, E and F are sitting in a (a) T is between U and Q
row facing towards North. C is sitting between
A and E. D is not at the end. B is sitting (b) U is the neighbor of V
immediate right to E. F is not at the right end. (c) V is between W and T.
12. How many persons are there to the right of D?
(d) W is between P and S
(a) One
(b) Two (e) None of these
(c) Three
17. What is the position of S ?
(d) Four
(e) None of these (a) On the immediate left of Q
13. Which of the following pairs is sitting to one (b) Second of the right of U
side of D ?
(a) FB (c) Between Q and U
(b) FC (d) On the immediate left of P

55
(e) None of these (b) A is at the Second place.
18. What is the position of V ? (c) A is at the third place.
(a) Second to the Left of S (d) A is at the fourth place.
(b) Third of the right of U 23. If A is not at the third place, then C has which
of the following options ?
(c) On the immediate right of W
(a) The first place only
(d) Between T and R
(b) The third place only
(e) None of these
(c) The first or third place only
 Read the following information carefully to
answer the given questions : (d) Any of the Places

Fifty books belonging to different subjects, 24. If A and B are together, then which of the
following must be necessarily true ?
History (8), Geography (7), Literature (13),
Psychology (8) and science (14), are placed on (a) C is at the first place
a shelf. They are arranged in an alphabetical (b) D is at the first place.
order subject to the condition that no two books
(c) C is not at the first place.
of the same subject are placed together so long
as books of others subjects are available unless (d) A is at the third place.
otherwise mentioned. All counting is done from
26. There are five houses A, B, C, D, O in a row.
the left.
A is right side of B and left side of C. O is in
19. Which subject does the 40th books belong to ? the irght side of A. B is right of D. Which
(a) Science (b) Psychology house is in the middle?

(c) History (d) Literature (a) O (b) A

20. What is the position of the last book in (c) B (d) D


Psychology? 27. Six friends A, B, C, D, E and F are sitting in a
(a) 36th (b) 37th row facing East, 'C' is between 'A' and 'E', 'B'
is just to the right of 'E' but left of 'D', 'F' is
(c) 38th (d) 39th
not at the right end. Who is between 'B' and
21. Counting from the right to which subject does 'C'?
the 39th books belong ?
(a) A (b) D
(a) History (b) Psychology
(c) E (d) F
(c) Geography (d) Science
27. Six friends A, B, C, D, E and F are sitting in a
 Read the following information to answer the row facing East. C is between A and E. B is
given questions :
just to the right of E but left of D. F is not at
A, B, C and D are seated in a row. But C and D the right end. Who is to the left of A?
cannot be together. Also, B cannot be at the
(a) E (b) C
third place.
(c) D (d) F
22. Which of the following must be false ?
28. Five friends are sitting on a bench. A is to the
(a) A is at the first place.
left of B but on the right of C. D is to the right
56
of B but on the left of E. Who are at the (b) R6 is sitting opposite to R6.
extreames? (c) R2 is sitting opposite to R6.
(a) AB (b) AD (d) R5 is to the immediate right of R6.
(c) BD (d) DE 33. Eight boys P, Q, R, S, T, U, V and W are sitting
29. 6 boys A, B, C, D, E, F are sitting in a row around a circular table facing the centre (not
facing West. D is between A and C. B is just necessarily in the same order).
right of C but left of F. E is not at the right P and Q are sitting opposite to each order.
end. Who is at the right end?
S is sitting second to the right of P and V is
(a) C (b) F sitting second to the right of q.
(c) D (d) B T and W are sitting opposite to each other.
30. Six teachers A, D, G, J, M and P are sitting R is sitting between P and V.
around a circular table facing the centre (not
necessarily in the same order). M is third to How many boys are sitting between U and V,
the left of G. D is to the immediate left of J. A when counted from the right of U?
is to the immediate left of M. (a) Five (b) Two
If we count from P in clockwise direction, how (c) Three (d) Four
many teachers are sitting between P and J? 34. Eight friends A, B, C, D, E, F, G and H are
(a) 4 (b) 2 standing in two rows, four in each row, during
(c) 1 (d) 3 prayer. H is after D. C is just before A. E is
after B. C and G are the first in both the rows.
31. Six shopkeepers K, P, R, X, A and C are sitting F is between G and B, the two who are
around a circular table facing towards the standing at the third position in both the rows,
centre (not necessarily in the same order). C are :
is second to the left of A, P is to the immediate
(a) B and H (b) D and F
right of K. X is second to the right of R. A is
to the immediate left of X, Which of the (c) B and F (d) D and B
following pair of shopkeepers represents the 35. Seven professionals are standing in a row. A
immediate neighbours of X? journalist is to the immediate right of a
(a) A and K (b) A and C pharmacist and to the immediate left of a
(c) K and P (d) A and R teacher. An advocate is on the immediate right
of an accountant. The advocate and Pharmacist
32. Six employees R1, R2, R3, R4, R5 and R6 have one person bwtween them. The teacher
are sitting around a circular table facing and a doctor have two persons between them.
towards the centre (not necessarily in the same An architect and the pharmacist have two
order). R5 is third to the left of R3. R2 is to persons between them. Who is in the middle?
the immediate left of R4. R1 is to the
immediate left of R5. (a) pharmacist

Which of the following statement is incorrect (b) Journalist


about R6? (c) Doctor
(a) R4 is second to the left of R6. (d) Teacher
57
36. Amit, Gaurav, Hatim, Varu, Yukti and Zaid row facing North. B is not neighbour of G. F
are sitting in a straight line, all facing the north. is to the immediate right of G and neighbour
Gaurav is fourth to the left of Amit. Yukti is of E. G is not at the extreme end. A is sixth to
sitting at one corner. Hatim is fourth to the the left of E. H is sixth to the right of C.
left of Yukti. Zaid is third to the right of Which one of the following is correct in
Gaurav. Who is sitting at the second place to respect of the above?
the left of Zaid?
(UPSC-2017)
(a) Varun
(a) C is to the immediate left of A.
(b) Yukti
(b) D is immediate neighbour of B and F.
(c) Hatim
(c) G is to the immediate right of D.
(d) Amit
(d) A and E are at the extreme ends.
37. Eight north-facing restaurants named P, Q,
R, S, T, U, V and W are located in a straight 40. Eight students A, B, C, D, E, F, G and H sit
line. S is second to the left of T. W is third around a circular table, equidistant from each
to the left of P, T is between P and V. S is other, facing the centre of the table, not
third to the left of V. W is to the imediate necessarily in the same order. B and D sit
right of U. R is third to right of P. Which neither adjacent to C nor opposite to C. A sits
restaurant is fourth to the right of S? in between E and D, and F sits in between B
and H. Which one of the following is definitely
(a) U correct?
(b) W (UPSC-2022)
(c) R (a) B sits in between A and G
(d) V (b) C sits opposite to G
38. Eight friends, A, B, C, D, E, F, G and H, are (c) E sits opposite to F
sitting in two lines of four persons each. Both
the lines are facing each other. A is facing D. (d) None of the above
E is to the immediate left of G. H is to the
immediate left of G. H is to the immediate right
of B. C is facing G. F is facing H. B is to the
immediate right fo the person who is facing
G. D is to the immediate left of F Which four
persons are sitting in the same line?
(a) A, B, C, H
(b) B, C, D, H
(c) A, C, F, G
(d) C, D, E, F
39. Consider the following
A, B, C, D, E, F, G and H are standing in a

58
Answer Sheet
1. (c) 21. (a)
2. (c) 22. (a)
3. (c) 23. (c)
4. (c) 24. (d)
5. (c) 25. (b)
6. (e) 26. (c)
7. (a) 27. (d)
8. (b) 28. (d)
9. (c) 29. (b)
10. (a) 30. (c)
11. (b) 31. (a)
12. (d) 32. (d)
13. (d) 33. (b)
14. (a) 34. (d)
15. (b) 35. (a)
16. (c) 36. (c)
17. (a) 37. (c)
18. (b) 38. (a)
19. (d) 39. (c)
20. (c) 40. (d)

59
1.
8
How many such pairs of letters are there in the alphabet?
word 'ENTHUSIASTIC' each of which has as (a) None (b) One
many letters between them in the word as there (c) Two (d) Three
are between them in the English alphabet? (e) More than three
(a) Two (b) Three (UPSC-2009)
(c) Four (d) More than four 6. How many such pairs of letters are there in the
(e) None of these word STAPLER each of which has as many letters
(UPSC-2004) between them in the word as in the English
2. If R is denoted by N, D is denoted by T, I by U, O alphabet?
by I, E by R, I by O, U by D, N by C and C by E; (a) None (b) One
then how will the word 'INTRODUCE' be written? (c) Two (d) Three
(a) UCOINTDER (e) More than three
(b) UCONTIDER (UPSC-2009)
(c) UCONITDER 7. How many such pairs of letters are there in the
(d) UCONDTER word CONFIRM each of which has as many
(e) buesa ls dksbZ ugha letters between them in the word as in the English
(UPSC-2004) alphabet?
3. If all the vowels of the word ENIGMATIC are (a) None (b) One
substituted with the next letter of the English (c) Two (d) Three
alphabetical series, and each consonant is (e) More than three
substituted with the letter preceding it. Which of (UPSC-2009)
the following would be the sixth letter from the 8. How many such pairs of letters are there in the
left? word CHRONICLE each of which has as many
(a) F (b) B letters between them in the word as in the English
(c) S (d) L alphabet?
(e) None of these (a) None (b) One
(UPSC-2008) (c) Two (d) Three
(e) More than three
4. If it is possible to make only one meaningful word
with the fourth, seventh, ninth and tenth letters (UPSC-2010)
of the word RECREATION, which of the 9. How many such pairs of letters are there in the
word PERFORRATE each of which has as many
following will be the third letter of that word? If
letters between them in the word as in the English
no such word can be made, give 'X' as your answer
alphabet?
and if more than one such word can be formed,
(a) None (b) One
give 'Y' as the answer.
(c) Two (d) Three
(a) X (b) Y
(e) More than three
(c) T (d) N
(UPSC-2010)
(e) R 10. How many such pairs of letters are there in the
(UPSC-2008) word ADJUSITING each of which has as many
5. How many such pairs of letters are there in the letters between them in the word as in the English
word JUMPING each of which has as many letters alphabet?
between them in the word as in the English (a) None (b) One

60
(c) Two (d) Three thus formed?
(e) More than three (a) F (b) R
(UPSC-2010) (c) V (d) S
11. How many such pairs of letters are there in the (e) K
word CONTRAST each of which has as many (UPSC-2010)
letters between them in the word as in the English
16. How many such pairs of letters are there in the
alphabet?
word SAMPLER, each of which has as many
(a) None (b) One
letters between them in the word (in both forward
(c) Two (d) Three
and backward directions) as they have between
(e) More than three them in the English alphabetical series?
(UPSC-2010)
(a) None (b) One
12. How many such pairs of letters are there in the
(c) Two (d) Three
word POSITIVE each of which has as many
letters between them in the words in the English (e) More than three
alphabet? (UPSC-2010)
(a) None (b) One 17. If it is possible to make only one meaningful word
(c) Two (d) Three with the second, fifth, eight and the eleventh
(e) More than three letters (when counted from left hand side) of the
(UPSC-2010) word 'DEVELOPMENT' which
would be the second letter of the word from the
13. If each alphabet in the word FRACTION is
arranged in alphabetical order (from left to right). left? If more than one such word can be formed,
and then each vowel is changed to next letter in give X as the answer. If no such word can be
the English alphabetical series and each consonant formed, give Z as your answer.
is changed to previous letter (a) E (b) L
in English alphabetical series, which of the (c) T (d) X
following will be fourth from the right side of (e) Z
the new arrangement thus formed? (UPSC-2010)
(a) M (b) J 18. If it is possible to make only one meaningful word
(c) P (d) E with the first, third, fourth and the seventh letters
(e) Q of the word 'EXHAUSTION' which would be the
(UPSC-2010) third letter of the word from the right? If more
14. How many such pairs of letters are there in the than one such word can be formed, give 'P' as the
word RETURS, each of which has as many letters answer. If no such word can be formed, give 'Q'
between them in the word (in both forward and as your answer.
backward directions) as they have between them (a) P (b) A
in the English alphabetical series? (c) H (d) T
(a) None (b) One (e) Q
(c) Two (d) Three (UPSC-2010)
(e) More than three 19. How many such pairs of letters are there in the
(UPSC-2010) word REGIONAL, each of which has as many
15. If each alphabet in the word ABSOLUTE is letters between them in the word (in both forward
arranged in alphabetical order (from left to right), and backward directions) as they have between
and then each vowel is changed to next letter in them in the English alphabetical series?
the English alphabetical series and wach (a) None (b) One
consonant is changed to previous letter in English (c) Two (d) Three
alphabetical series, which of the following will (e) More than three
be sixth from the left side of the new arrangement (UPSC-2010)

61
20. It is possible to make only one meaningful word (e) Y
with the first, fifth and the tenth letters of the word 25. If it is possible to make only one meaningful
'AUTOMOBILE', which would be the second English word with the third, fouth, seventh and
letter of the word from the right? If more than the eight letters of the word 'VIDEOGRAPHY',
one such word can be formed, give 'Y' as the which would be the second letter of that word
answer. If no such word can be formed, give 'Z'
from the right end? If more than one such word
as your answer.
can be formed, give 'X' as the answer. If no such
(a) Y (b) L
word can be formed, giveZ as your answer.
(c) A (d) E
(a) D (b) A
(e) Z
(c) R (d) X
(UPSC-2010)
(e) Z
21. How many such pairs of letters are there in the
word 'SUBSTANCE' each of which has as many 26. If only the consonant in the word MEAT are
letters between them in the word (in both forward changed in such a way that each of them be comes
and backward directions) as in the English the next letter in the the English alphabetical and
alphabet? the remaining letters are kept unchanged, then
(a) None (b) One how many meanigful words can be formed with
(c) Two (d) Three the new set of letters using each letter only once
(e) More than three in a word?
(UPSC-2012) (a) One (b) Three
22. How many such pairs of letters are there in the (c) Two (d) None
word ASJUSTINGT each of which has as many (e) None of these
letters between them in the word as in the English (UPSC-2001)
alphabet? 27. If it is possible to make a meanigful English word
(a) None (b) One with the seventh, tenth and the fourth letters of
(c) Two (d) Three the word 'ARISTOPATHOLOGY' and the only
(e) More than three vowel from the word 'TOP', the third letter of that
(UPSC-2014) word is your answer. If more than one English
23. If it is possible to make only one meaningful word can be formed, 'M' is your answer. If no such
English word with the first, third, eight and ninth word can be formed, your answer is 'N'.
letters of the word TRANSPORMS, (a) O (b) P
which of the following will be the second letter
(c) S (d) M
of that word? If no such word can be formed, give
'X' as the answer and if more than one such word (e) N
can ber formed, give 'Y' as the answer. (UPSC-2002)
(a) T (b) R 28. If it is possible to make only one meanigful word
(c) M (d) X with the first, the third, the fifth and the eighth
(e) Y letters of the word SHAREHOLDING, which of
(UPSC-2014) the following will be the second letter of that
24. If all the vowels in the word 'SAINTLY' are word? If no such word can be made, give 'X' as
rearranged in the alphabetical order from left to the answer and if more than one such word can
right, followed by the consonants in the be made, give 'Y' as the answer/
alphabetical order from left to right, the position (a) L (b) E
of which of the following five alphabets will (c) S (d) X
remain unchanged? (e) Y
(a) S (b) T
(UPSC-2008)
(c) L (d) A
29. How many such pairs of letters are there in the

62
word GLIMPSE each of which has as many letters are written in reversed order and so on, and at the
between them in the word as in the English end Y is interchanged by Z, then which letter is
alphabet? fourth letter to the right of 13th letter from the
(a) None (b) One left?
(c) Two (c) Three (a) M (b) N
(c) Q (d) P
(e) More than three
(UPSC-2009)
30. The positions of the first and the fourth letters of
35. If the English Alphabet series is written in the
the word PLANET are interchanged, similarly,
reverse order and every alternet letter starting
the positions of the second and fifth letters and
from Y is dropped, which letter will be exactly in
third and sixth letters are inter changed. In the
the middle of the remaining letters of the Alphabet
new arrangement thus formed, how many letters
series?
are there between the letter which is second from
(a) L (b) O
the right and the letter which is fourth from the
(c) M (d) N
left according to the English alphabetical order?
36. If the letters in each of the following five groups
(a) None (b) One
of letters are first rearranged in the alphabetical
(c) Two (d) Three order and then the groups of letters so formed are
(e) Four rearranged as in a dictionary, which letter group
(UPSC-2011) would have its group of letters in the MIDDLE
31. If it is possible to make only one meanigful word among the five letter-groups?
with the second, third, eight and ninth letters of MEET, DEAF, ROAD, CODE, LACK
the word, CONFLICTED', which would be the (a) LACK (b) MEET
second letter of the word from the left? If more (c) ROAD (d) DEAF
than one such word can be formed, give 'A' as the (UPSC-2009)
answer. If no such word can be formed, give 'Z' 37. It is possible to make only one meaningful word
as your answer. with the sixth, eighth, ninth and tenth letters of
(a) A (b) T the word 'SOCUMENTARY', which would be the
(c) O (d) N third letter of that word from left end? If more
(e) Z than one such words can be formed, give 'X' as
(UPSC-2011) the answer. If no such words can be formed, give
32. How many such pairs of letters are there in the Z as your answer.
word PREMILINARY, each of which has as many (a) X
letters between them in the word (in both forward (b) R
and backward directions) as they have in the (c) Z
English alphabetical series? (d) A
(a) One (b) None (e) T
(c) Three (d) More than three 38. How many such pairs of letters are there in the
(e) Two word 'PLATFORMS' each of which as many
(UPSC-2015) letters between them in the word (in both forwad
33. Which letter in the word and backward directions), as they have between
SELFRIGHTEOUSNESS does not change its them in the english alphabetical series?
position when the letters are reversed? (a) Three
(a) E (b) G (b) None
(c) H (d) T (c) One
34. If the first 6 letters of the English Alphabet series (d) One
are written in reversed order, then the next 6 letters (e) More than three

63
39. In English alphabet, the first 4 letters are written 40. What is the middle term of the sequence
in opposite order and the next 4 letters are written Z, Z, Y, Y, Y, X, X, X, X, W, W, W, W, W,..., A?
in opposite order and so on and at the end Y and (UPSC-2023)
Z are interchanged. Which will be the fourth letter (a) H
to the right of the 13th letter from the left?
(b) I
(UPSC-2021)
(c) J
(d) M
(a) N (b) T
(c) H (d) I

ANSWER SHEET
1. (a) 2. (c) 3. (b) 4. (e) 5. (c)
6. (c) 7. (c) 8. (d) 9. (c) 10. (d)
11. (d) 12. (e) 13. (a) 14. (d) 15. (b)
16. (c) 17. (a) 18. (a) 19. (e) 20. (a)
21. (d) 22. (d) 23. (e) 24. (e) 25. (d)
26. (d) 27. (a) 28. (e) 29. (c) 30. (d)
31. (a) 32. (e) 33. (d) 34. (b) 35. (d)
36. (c) 37. (a) 38. (d) 39. (b) 40. (c)

64
9
(c) Leaf (d) Ground
TYPE - I
11. Book : Publisher : : Film : ?
Directions: In each of the following questions, (a) Writer (b) Editor
select the related word from the given alternatives.
(c) Director (d) Producer
1. Touch : Feel :: Greet : ?
12. Menu : Food : : Catalogue : ?
(a) Smile (b) Acknowlege (a) Books (b) Library
(c) Success (d) Manners (c) News paper (d) Rack
2. House : Room :: World : ? 13. January : November : : Sunday : ?
(a) Land (b) Sun (a) Tuesday (b) Monday
(c) Air (d) Nation (c) Friday (d) Saturday
3. Good : bad : : Roof : ? 14. Stethoscope : Heartbeat : : ? : Temperature
(a) Walls (b) Pillars (a) Heat (b) Mercury
(c) Floor (d) Window (c) Scale (d) Thermometer
4. Detective : Informer : : Repoter : ? 15. Mirage : Desert : : ?
(a) Source (b) Editorial (a) Sky : Illusion (b) Rainbow : Sky
(c) News (d) Essay (c) Rain : Rainbow(d) Image : Mirror
5. Thunder : Rain : : ? : Night 16. Writer : Pen : : ?
(a) day (b) Dusk (a) Needle : Tailor (b) Artist : Brush
(c) Dark (d) Evening (c) Painter : Canvas (d)Teacher : Class

6. Botany : Plants : : Entomology : ? 17. Fish : Gills : : Man : ?


(a) Ear (b) Eyey
(a) Birds (b) Plants
(c) Lung (d) Nose
(c) Insects (d) Snakes
18. MAN : BIOGRAPHY : : NATION : ?
7. Parliament : Great Britain : Congress : ?
(a) PEOPLE (b) POPULATION
(a) Japan (b) India
(c) GEOGRAPHY (d) HISTORY
(c) USA (d) Netherlands
19. Good conductor : Copper : : Bad Conductor :
8. Psychology : Mind : : Ornithology : ? ?
(a) Sanskrit (b) Coin (a) Iron (b) Aluminium
(c) Mammal (d) Bird (c) Wood (d) Coal
9. Mother : Child :: Cloud? 20. Horse : Hoof : : ?
(a) Shine (b) Water (a) Man : Foot (b) Dog : Black
(c) Rain (d) Weather (c) Palse: Rupee (d) Pen : Pencil
10. Mountain : Hill : : Tree : ? 21. Mitochondria : Energy : : DNA : ?
(a) Forest (b) Shrub (a) Inheritance (b) Reproduction

65
(c) Locomation (d) Immunity (a) 2, 7, 12, 32 (b) 3, 8, 18, 33
22. Moon : Chandrayan : : Mars : ? (c) 4, 8, 19, 34 (d) 8, 13, 22, 38
(a) Apple (b) Aryabhata
(c) Mangalyan (d) Bhaskara TYPE - III
TYPE - II Directions: In each of the following question,
select the related number from the given
Directions: From among the given alternatives alternatives.
select the one in which the set of numbers is most 1. 5 : 36 : : 6 : ?
like the set of numbers given in the question. (a) 48 (b) 50
1. Given set : (3, 7, 15)
(c) 49 (d) 56
(a) 2, 6, 10 (b) 4, 8, 18
2. 18 : 30 : : 36 : ?
(c) 5, 9, 17 (d) 7, 12, 19
(a) 64 (b) 66
2. Given set : (3, 18, 36)
(c) 54 (d) 62
(a) (2, 10, 16) (b) (12, 72, 96)
3. 6 : 18 : : 4 : ?
(c) (4, 24, 48) (d) (6, 42, 48)
(a) 2 (b) 6
3. Given set : (6, 36, 63)
(c) 9 (d) 16
(a) 7, 49, 98 (b) 8, 64, 46
4. 42 : 56 : : 72 : ?
(c) 9, 84, 45 (d) 11, 111, 84
(a) 81 (b) 90
4. Given set : (32, 24, 8)
(a) (42, 34, 16) (b) (24, 16, 0) (c) 92 (d) 100

(c) (34, 24, 14) (d) (26, 32, 42) 5. 6 : 222 : : 7 : ?


5. Given set : (3, 4, 5) (a) 350 (b) 343
(a) (2, 6, 10) (b) (9, 12, 15) (c) 336 (d) 210
(c) (6, 7, 8) (d) (12, 16, 20) 6. 122 : 145 : : 226 : ?
6. Given set : (6, 14, 30) (a) 250 (b) 256
(a) 4, 16, 28 (b) 7, 12, 22 (c) 257 (d) 290
(c) 6, 12, 22 (d) 5, 12, 20 7. 8 : 20 : : 14 : ?
7. Given set : (12, 20, 28) (a) 35 (b) 30
(a) 3, 15, 18 (b) 18, 27, 72 (c) 28 (d) 20
(c) 18, 30, 42 (d) 7, 14, 28 8. 6415 : 5304 : : 7896 : ?
8. Given set : (12, 18, 24) (a) 6705 (b) 6905
(a) (6, 12, 30) (b) (5, 15, 10) (c) 6907 (d) 6785
(c) (10, 16, 18) (d) (8, 12, 16 9. 9536 : 6203 : : 5873 : ?
9. Given set : (64, 216, 125) (a) 2540 (b) 2343
(a) 26, 144, 163 (b) 55, 126, 80 (c) 2353 (d) 2531
(c) 8, 27, 64 (d) 45, 134, 154
10. Given set : (7, 12, 22, 37) TYPE - IV

66
(c) USJHQO (d) SUJHOQ
Direction: In each of the following questions,
12. MONKY : YENOM : : RIGHT : ?
select the related letter from the given alternatives.
1. A D H M : Z W S N : CFJO : ? (a) HTIRG (b) THGIR
(a) YVRM (b) WTPK (c) GIRHT (d) IRGHT
(c) XWTP (d) ZXVT 13. KMNP : ACDF : : PRSU : ?
2. BVSC : YEHX : : MRCP : ? (a) STVW (b) TVWY
(a) NJXK (b) LKXM (c) VXYZ (d) LNPR
(c) NIXK (d) OIYM 14. ACE : KIG : : MOQ : ?
3. EGIK : FILO : : FHJL : ? (a) WUS (b) WVU
(a) GJMP (b) GMJP (c) WVT (d) WUT
(c) JGMP (d) JGPM
15. FLRX : EJOT : : CKTY : ?
4. RED : EFS : : BLUE : ?
(a) BIQV (b) DIPQ
(a) FVMC (b) DTKA
(c) DHQU (d) BIQU
(c) FUNC (d) GVND
5. FILM : ADGH : : MILK : ?
TYPE - V
(a) ADGF (b) HDGE
(c) HDGF (d) HEGF
6. BJNT : CIOS : : DHPV : ? Directions: Select the related letter/number from
the given altenatives.
(a) EGQU (b) EIQW
1. C : 16 : : F : ?
(c) ELPV (d) EIOU
(a) 30 (b) 49
7. MASTER : OCUVGT : : LABOUR : ?
(a) NCDQWT (b) NDERWT (c) 40 (d) 50

(c) NBCRWT (d) NEDRWT 2. CE : 70 : : DE : ?


8. FIGUR : FGIRU : : STRES : ? (a) 90 (b) 60
(a) STERS (b) TRSES (c) 120 (d) 210
(c) SRSET (d) SRTSE T X
3. :2:: :?
9. LOCKER : KMNPBDJLDFQS : : LEFT : ? J H
(a) KNCDSGSU (b) KMDFEGSU (a) 2 (b) 3
(c) KMDFEGUS (d) KMDEFGUS 23
(c) (d) 4
10. DumB : BonD : : RusT : ? 7
(a) MOst (b) TeNt 4. CD : 34 : : AB : ?
(c) PaTH (d) WorK (a) 25 (b) 68
11. CUT : BDTVSU : : TIP : ? (c) 12 (d) 21
(a) UVHJOQ (b) SUHJOQ

67
169 Answer Sheet
5. MK : : : JH : ?
121
TYPE - I TYPE - II
81 64
(a) (b) 1. (A) 12. (A) 1. (C)
100 120
2. (D) 13. (C) 2. (C)
100 100 3. (C) 14. (D) 3. (B)
(c) (d)
81 64
4. (A) 15. (C) 4. (B)
5. (B) 16. (C) 5. (C)
6. (C) 17. (C) 6. (B)
7. (C) 18. (D) 7. (C)
8. (D) 19. (C) 8. (B)
9. (C) 20. (A) 9. (C)
10. (B) 21. (A) 10. (B)
11. (C) 22. (C)

TYPE - III TYPE - IV


1. (C) 1. (A) 10. (D)
2. (B) 2. (C) 11. (B)
3. (D) 3. (A) 12. (B)
4. (B) 4. (A) 13. (B)
5. (A) 5. (C) 14. (A)
6. (C) 6. (A) 15. (D)
7. (A) 7. (A)
8. (D) 8. (D)
9. (A) 9. (B)

TYPE - V
1. (B)
2. (A)
3. (B)
4. (C)
5. (D)

68
EXPLAINATION conditions making one see something that
is not there especially the apperance of a
1. (b) A Touch has Feeling effect. Similarly, sheet of water on a hot road or in a desert.
Greet is Acknowledged.
Rainbow is an arch of seven colours
2. (d) Room is a part of the house. Similarly, formed in the sky when the sun shines
nation is a part of the world. through rain.
3. (c) The second term is antonym of the first 16. (c) Here, Worker - Tool relationship has been
term. Hence, Roof: Floor. shown. Writer uses pen for writteng.
4. (a) Detective collects vital infor-mation with Similarly, painter works on canvas.
the help of informer. Similarly, Reporter 17. (c) Respiratory organ of fish is gills.
collects news from source. Similarly, lung is the respiratory orgain
5. (b) Thunder is immediately followed by rain. of man.
Similarly, dusk is followed by night. 18. (d) The story of a man is called Biography.
6. (c) Scientific study of plants is called Botany. Similarly, the story of a nation is called
Similarly, scientific study of insects is History.
called Entomology. 19. (c) Copper is a good conductor of heat and
7. (c) The legislative house (Parliament) of electricity. Wood a had conductor of
Great Britain is called Parliament. electricity.
Similarly, the Parliament of USA is called 20. (a) The hoof of horse is anologous to foot of
Congress. man.
8. (d) Psychology is the study of mind, mental 21. (a) Mitochondria is referred to as the
condition. Similarly, the scientific study powerhouse of the cell in which
of birds is called ornithology. respiration andenergy production occur.
9. (c) Mother gives birth to child. similarly, DNA is the unit of inheritance.
cloud causes rain. 4. (c) The spacecraft landed on moon was
10. (b) Mountain and hill are similar things. named Chandrayan by the Indian space
Similarly, tree and shrub are of same type. Research Organisation. Similarly, the
11. (c) January is the first month of a year and spaceraft which landed on Mars in 2014
November is the second last month of a was named Mangalyan.
year. Similarly, Sunday is the first day of
a week and Friday is the second last day TYPE-II
of a week.
1. (c) Given set (3, 7, 15)
12. (a) The list of food items is called menu.
1 st Number + 4 = 2 and number 2nd
Similarly, the list of books is called
Number + 8 = 3rd number 3 + 4 = 7 and
catalogue.
7 + 8 = 15 5 + 4 = 9 and 9 + 8 = 17
13. (c) Book is published by Publisher. Similarly,
2. (c) Given set (3, 18, 36)
film is directed by director.
14. (d) Stethoscope is a scientific instrument 3  6 = 18 and 18  2 = 36
which measures heartbeat. Similarly, Similarly,
Thermometer measures temperature. (4, 24, 48)
15. (c) Mirage is an illusion caused by hot air 4  6 = 24 and 24  2 = 48

69
3. (b) (6, 36, 63) Similarly,
8  (4)3
6 6=3 6  6 3 24  (3)3
64  (4)3
Similarly. 10. (b) 7 + 5 = 12; 12 + 10 = 22;
22 + 15 = 37
8 8=6 4  4 6 Similarly,
3 + 5 = 8; 8 + 10 = 18;
4. (b) The differences between the consecutive 18 + 15 = 33
numbers are 8, 16.
TYPE-III
32 __ 24 __ 8
1. (c) 5 + 1 = (6)2  36
8 16
Similarly,
24 __16 __ 0
8 16 6  1  (7) 2  49
2. (b) 18  2 = 36 and
5. (c) The consecutive numbers are given.
36 – 6 = 30
6. (b) 6 + 8 = 14
Therefore,
14 + (8  2) = 30
36  2 = and 72 – 6 = 66
Similarly,
3. (d) 6 + 12 = 18
7 + 5 = 12
Therefore. ? = 4 + 12 = 16
12 + (5  2) = 22
4. (b) 42 : 56 : : 72 : ?
7. (c) 4  3 = 12; 4  5 = 20;
4  7 = 28
Similarly, 6 7 7 8 8  9 9  10
6  3 = 18; 6  5 = 30;
6  7 = 42
90
8. (d) 6  2 = 12; 6  3 = 18;
5. (a) (6)3 + 6 = 222
6  4 = 24
Similarly,
Similarly,
(7)3 + 7 = 350
4  2 = 8; 4  3 = 12;
6. (c) 122 = (11)2 + 1
4  4 = 16
145 = (12)2 + 1
9. (c) In the given set all the numbers are perfect
226 = (15)2 + 1
cubes.
Therefore, ? = (16)2 + 1 = 257
64  (4)3
7. (a) 8  2.5 = 20
216  (6)3
Similarly,
125  (5)3
70
14  2.5 = 35 Similarly,

8. (d) 6 4 1 5 5 3 0 4 F H J L G J M P

Similarly, 4. (a) R E D E F S
7 8 9 6 6 7 8 5

Similarly,

B L U E F V M C
9. (a) 9536 – 6203 = 3333
Similarly,
? = 5873 – 3333 = 2540
5. (c) 6 9 12 13 1 4 7 8
TYPE-IV F I L M A DGH

3 4 5
1. (a) A   D   H  M
3 4 5
Z   W   S  N
Similarly,
Similarly,
3 4 5 13 9 12 11 8 4 7 6
C   F   J  O M I L K H DGF
3 4 5
Y   V   R  M
2. (c) The two terms are the sets of opposite
letters.
B V S C 6. (a) B J N T C I O S

Y E H X
Similarly,
M R C P Similarly,

D H P V E G O U
N I X K

3. (a) E G I K F I L O

71
and the last letters are capital letters while
7. (a) M A S T E R O C U V G T the two middle letters are small letters.
Similarly, in the words RusT and WorK,
the first and the last letters are Capital
letters and the two middle letters ae small
letters.
11. (b) Two letters, one preceding and second
Similarly, following, are given for each letter.

L A B O U R N C D Q W T 1
C  1
 B and C  D
1 1
U   T and U  V
1 1
T   S and T  U

Similarly,

1 1
8. (d) F I G U R T   S and T  U
1 1
I   H and I  J
1 1
F G I R U T   O and P  Q

Similarly,
12. (b) M O N E Y Y E N O M
S T R E S

S R T S E
Reverse order to letters
9. 1
(b) I  KM; L  1
 K; L  M Similarly,
1 1
O  NP; O   N; O  P R I G H T T H G I R
1 1
C  BD; C 
 B; C 
D
1 1
K  JL; K   J; K  L
1 1
E  DF; E   D; E  F
1 1
R  QS; R   Q; R  S
2 1 2
13. (b) A  M   N  P
2 1 2
Similarly, A   C   D  F
2 1 2
L  KM P   R   S  U
E  DF But,
2 1 2
1 1 T   V   W  Y
F  F   E; F  G
1 1
T  T   S; T  U 2 2
14. (a) A  C  E
10. (d) In the words DumB and BonD the first 2
K  2
 I  G

72
Similarly, 5. (d) M  13 and (13)2 = 169
2 2 K  11 and (11)2 = 121
M   O  Q
2
W  2
 U  S 169
MK =
121
15. (d) F L R X E J O T Similarly,
J = 10 and (10)2 = 100
H = 8 and (8)2 = 64

Similarly, 100
Therefore. JH =
64
C K T Y B I Q U

TYPE-V

1. (b) C  3
(3 + 1) = 16
F6

(6 + 1)2 = 49
2. (a) Position Numbers of C and E =3 and 5
CE  35  2 = 70
Similarly,
DE = 45  2 = 90

 20
3. (b) :2  =2
J 10
Therefore,

X 24
 3
H 8

4. (c) C D 3 4

Position Number of letter in the English


alpahebt

Thus. A B 1 2

73
10
TYPE - I 12. (a) January (b) March
(c) July (d) September
1. (a) Insurance (b) Provident Fund
13. (a) Spectacles (b) Dentures
(c) Salary (d) Share
(c) Baisakhi (d) Plaster
2. (a) Sociologist
14. (a) goat (b) lion
(b) Economist
(c) horse (d) cow
(c) Neuroscientist
15. (a) January (b) February
(d) Educationist
(c) July (d) December
3. (a) Paper : Pencil 16. (a) post (b) telegram
(b) Head : Cap (c) telephone (d) electricity
(c) Ink: Medicine 17. (a) Pilot (b) Passenger
(d) Gift: Wrapping-covering (c) Driver (d) Navik
4. (a) square (b) circle 18. (a) Saturn (b) Venus
(c) rectangle (d) triangle (c) Sun (d) Jupiter
5. (a) liver (b) lungs 19. (a) Bhopal (d) Amritsar
(c) kidney (d) ear (c) Hyderabad (d) Bengaluru
6. (a) Newspaper (b) Press 20. (a) square (b) circle
(c) version (d) sound test (c) rectangle (d) triangle
7. (a) pistol (b) sword 21. (a) March (b) April
(c) gun (d) rifle (c) August (d) December
8. (a) Hindi (b) Tamil 22. (a) snooker (b) table tennis
(c) Punjabi (d) Urdu (c) Badminton (d) Billiard
9. (a) drama-actor 23. (a) Moon (b) Mars
(b) building-architect (c) Venus (d) Jupiter

(c) Craft-Craftsman 24. (a) pen (b) marker (chik)

(d) Cloth-Skirt (c) paper (d) pencil

10. (a) Petrol-car (b) Coal-fuel 25. (a) cotton (b) nylon

(c) smoke-fire (d) oil-lamp (c) silk (d) linen

11. (a) 14th November 26. (a) Money Order

(b) 15th August (b) Speed Post

(c) 26 January (c) Letter

(d) 2 October (d) S.M.S.


27. (a) Wheat (b) Mustard
74
(c) Barley (d) Maize 15. (a) 13 (b) 26
(c) 14 (d) 88
TYPE - II
8 16
Find out the pair of numbers from the follow- 16. (a) (b)
9 64
ing which do not belong to a group due to lack
of common property. 32 64
(c) (d)
1. (a) 84, 67 (b) 112, 95 25 36
(b) 79, 63 (c) 167, 150 17. (a) 2010 (b) 2011
2. (a) 95-7 (b) 42-6 (c) 2012 (d) 2013
(c) 35-5 (d) 56-8 18. (a) 325 (b) 360
3. (a) 14, 9 (b) 17, 8 (c) 230 (d) 256
(c) 42, 3 (d) 21, 6 19. (a) 56 (b) 28
4. (a) 72 (b) 45 (b) 36 (d) 35
(c) 81 (d) 28 20. (a) 222 (b) 10
5. (a) 43 (b) 53 (c) 68 (d) 125
(c) 63 (d) 73 21. (a) Year 2012
6. (a) 55  5 (b) 15  15 (b) Year 1998
(c) 5  45 (d) 25  9 (c) Year 2005
7. (a) 81-45 (b) 72-91 (d) Year 1997
(c) 117-99 (d) 135-126 22. (a) 83 (b) 64
8. (a) (9,36,81) (c) 56 (d) 98
(b) 32,64,88 TYPE - III
(b) (55,135,165)
(d) (35,63,78) 1. (a) AFEJ (b) KOPT
9. (a) 206 (b) 125 (c) UYZD (d) EHIL
(c) 27 (d) 8 2. (a) ACE (b) FHJ
10. (a) 357 (b) 581 (c) KLM (d) SUW
(c) 698 (d) 784 3. (a) DEB (b) RTP
11. (a) 162 (b) 405 (b) HIF (d) NOL
(c) 567 (d) 644 4. (a) QOOM (b) WUUS
12. (a) 111 (b) 263 (c) JIIF (d) VTTR
(c) 383 (d) 551 5. (a) A (b) I
13. (a) 16-36 (b) 25-55 (c) D (d) E
(c) 49-64 (d) 25-81 6. (a) DI (b) KQ
14. (a) 35 (b) 37 (c) OU (d) AG
(c) 23 (d) 19 7. (a) YDWB (b) TKRI

75
(c) QNOM (d) HLFJ 25. (a) FIK (b) DGI
8. (a) JKOP (b) MNST (c) MPR (d) KND
(c) CABD (d) OPWX 26. (a) AZBY (b) CXDW
9. (a) DWHS (b) BYDW (c) EVFU (d) TGSH
(c) CWFS (d) EVJQ 27. (a) B C D G (b) G I J L
10. (a) SPOKE (b) SMOKE (c) PRSU (d) UWXZ
(c) KOPES (d) POKES 28. (a) NOTA (b) NATO
11. (a) PQO (b) VWU (c) NASA (d) NAM
(c) AZY (d) TUS 29. (a) MTF (b) SLE
12. (a) ZXVT (b) RPNL (c) RKD (d) UNG
(c) ONKI (d) HFDC
13. (a) XV (b) ZW Answer Sheet
(c) YV (d) WT TYPE - 1
14. (a) BEH (b) CFI 1. (c) 2. (c) 3. (a) 4. (b) 5. (d)
(c) DGJ (d) EHL 6. (d) 7. (b) 8. (d) 9. (d) 10. (c)
15. (a) HIVW (b) MKNL 11. (a) 12. (d) 13. (d) 14. (b) 15. (b)
(c) EFOP (d) CDYZ 16. (d) 17. (b) 18. (c) 19. (b) 20. (b)
16. (a) A B D G (b) C D F I 21. (b) 22. (c) 23. (a) 24. (c) 25. (c)
(c) E F H K (d) G H J K 26. (d) 27. (b)
17. (a) N M L (b) O P Q TYPE - 2
(c) X W V (d) H G E 1. (c) 2. (a) 3. (c) 4. (c) 5. (c)
18. (a) PCAQ (d) LKIN 6. (a) 7. (b) 8. (a) 9. (a) 10. (c)
(c) XGEZ (d) DWEF 11. (d) 12. (c) 13. (b) 14. (a) 15. (a)
19. (a) BEG (b) KNP 16. (b) 17. (c) 18. (d) 19. (c) 20. (d)
(c) WZB (d) JLN 21. (a) 22. (b)
20. (a) MTF (b) SLE TYPE - 3
(c) RKD (d) UNG 1. (d) 2. (c) 3. (b) 4. (d) 5. (c)
21. (a) ZYX (b) FED 6. (a) 7. (c) 8. (c) 9. (c) 10. (b)
(c) NML (d) GHI 11. (c) 12. (d) 13. (a) 14. (d) 15. (b)
22. (a) BADC (b) XWZY
16. (d) 17. (d) 18. (a) 19. (d) 20. (a)
(c) VUST (d) NMPO
21. (d) 22. (c) 23. (a) 24. (c) 25. (d)
23. (a) I (b) N
26. (d) 27. (a) 28. (a) 29. (a)
(c) W (d) H
24. (a) I (b) E
(c) B (d) O

76
Explanation 18. (c) All others are planets, while the Sun is a
star.
TYPE - I 19. (b) Except Amritsar, all others are capitals.
1. (b) Rupees are deposited in all others while 20. (b) Except sphere, all other figures are two
Rupees are withdrawn from Salary dimensional. A sphere is a three
(Central). dimensional figure.
2. (b) Neuroscience is a branch of science while 21. (b) Except April, all other months have 31
the other three options are related to social days.
science. 22. (c) Except badminton all others are played
3. (a) Paper and pencil are two different on the table.
stationery articles. A cap is worn on the 23. (a) Except Moon, all other are planets, Moon
head. The ink is kept in the medicine. The is a satellite.
offering is wrapped with a wrapping 24. (c) Except paper, all other are used for
paper. writing.
4. (b) All the remaining figures have a group 25. (c) Yarn is obtained from trees, nylon and
of lines. linen are artificial fibres. Silk is obtained
5. (b) All other parts of the body except the ear from an insect.
are internal, while the ear is an external 26. (d) Money order, speed post and letter are
part. related to post office. SMS service is
6. (d) Sound test is different from the other three available on cell phones.
as they are all related to publication. 27. (b) Mustard is an oilseed. Wheat, barley and
7. (b) Cartridges are used in all except swords. maize are grains.
8. (d) All other languages except Urdu are TYPE - II
indigenous languages. Urdu is a foreign
language developed from Persian. 1. (c) 84 – 67 = 23  odd Number
9. (d) Except for the cloth-skirt pair, work and 112 – 95 = 23  odd Number
worker relations are shown in other pairs.
79 – 63 = 16  Even Number
10. (c) Smoke comes out of fire. Petrol is used
167 – 150 = 17  odd Number
as fuel in a car. In the same way coal is
used in a steam engine, oil burns in fixed 2. (a) Dividing other numbers by 7 gives the
lamps. latter number.
3. (b) are multiples of each other only in 3.
11. (a) All others are national holidays.
4. (b) No other square number.
12. (d) All the remaining months have 31 days.
5. (b) Except 63, all others are prime numbers.
13. (d) When the concerned organ is damaged
6. (a) 55  5 = 275, 15  15 = 225
in all others, it is used for the related
5  45 = 225 25  9 = 225
work.
7. (b) 8+1=9;4+5=9
14. (b) All others are domestic animals.
1 + 1 + 7 = 9 ; 9 + 9 = 19  1 + 8 = 9
15. (b) All other options except february are of
1+3+5=9;1+2+6=9
31 days.
16. (d) All others are means of communication.
but,
17. (d) All other vehicles are drivers while 7 + 2 = 9 ; 9 + 1 = 10
passengers are different. 1 + 0 = 1

77
8. (a) 9  4 = 36; 9  9 = 81 +1
1. (d) +1
32  2 = 64; 32  2.75 = 88
55  2.454 = 135 ; 55  3 = 165 A E F J K O P T
35  1.8 = 63; 35  2.228 = 78 +9 +9
9. (a) Except the number 206, all other numbers
are perfect cubes. +1 +1
10. (c) Except 698, all others are multiples of 7.
U Y Z D E H I L
11. (d) 162  16  2 = 8
405  40  5 = 8 +9 +7
567  56  7 = 8
644  64  4 = 16 2. (c) 2
A  2
 C  E
12. (c) Except for the number 383, in all other 2 2
F   H  J
numbers the product of the first and 1 1
K   L  M
third digits is equal to the middle digit.
2 2
1  1 = 1; 2  3 = 6; 5  1 = 5; S   U  W

but, 2  3 = 9 3. (b) The rest of the group of letters has a


13. (b) All others have both square numbers. vowel.
14. (a) Except the number 35, all others are 4. (d) Except letter group VTTR, all other
prime numbers. letter groups have two identical
15. (a) 13 is a prime number. vowels.
5. (c) Except the letter D, all other vowels
16 are.
16. (b) One number is exactly divisible by
64
5 6
another number. The number 16 is the 6. (a) D   I; K  Q
6 6
square of 4 and 64 is the perfect cube O   U; A  G
16 -2
of =4
64
Y D W B
4  4 = 16; 4  4  4 = 64 7. (c)
17. (c) 2012 is a leap year. -2
18. (d) The number 256 is a perfect square.
19. (c) The number 36 is a perfect square. -2
36 = 6  6 T K R I
20. (d) The number 125 is a perfect cube.
-2
5  5  5 = 125
21. (a) The year 2012 is a leap year. -1
22. (b) The number 64 is a perfect square.
Q N O M
TYPE - III
-2

78
-2 15. (b) 1
H  I 1
V  W
2 2
H L F J M   K L  N
1 1
-2
E   F O  P
1 1
C   D Y  Z
1 4 1
8. (c) J   K   O  P
1 2 3
1
M  5
 N  1
 S  T
16. (d) A   B   D  G
1 2 3
2
C  1
 A  2
 B  D C   P   F  I
1 2 3
1
O  7
 P  1
 W  X E   F   H  K
1 2 1
G   H   J  K
9. (c) D  W; H  S
pairs of opposite letters 1 1
17. (d) N   M  L
B  Y; D  W 1 1
E  V; J  Q O   P  Q
1 1
The opposite letter of C is X and the X   W  V
opposite letter of F is U. 1
H  2
 G  E
10. (b) In the word SMOKE, one letter is
different from other words. P -2 Q
18. (a) C A
+1
1 2
11. (c) P 
 Q  O L -2 N
1 2
K I
V 
 W  U +2
25 1
A  Z  Y X
G
-2
E
Z
1 2 +2
T   U  S
D -2 F
W U
2 2 2 +2
12. (d) Z   X   V  T
2 2 2
R   P   N  L 3
B  2
 E  G
19. (d)
2 2 2
O   M   K  I 3
K  2
 N  P
2 2 1
H   F   D  C 3
W  2
 Z  B
2 2
13. (a) 2 3 J   L  N
X   V; Z   W;
3 3
Y   V; W   T;
20. (a) 7
M  14
 T  F
7 7
14. (d)
3
B  3
 E  H S   L  E
7 7
3
C  3
 F  I R   K  D
7 7
3
D  3
 G  J U   N  G

but, E 
3 4
 H  L

79
13 8. X and Y are the children of A. Ais the father
TYPE - I
of X but Y is not his son. How is Y related to
1. A is B's daughter. B is C's mother. D is C's A?
brother. How is D related to A ? (a) Sister (b) Brother
(a) Father (b) Grandfather (c) Son (d) Daughter
(c) Brother (d) Son 9. D, the son-in-law of B is the brother-in-law
2. P is Q's brother. R is Q's mother. S is R's father. of A who is the brother of C. How is A re-
T is S's mother. How is P related to T? lated to B?
(a) Granddaughter (b) Great Grandson (a) Brother (b) Son
(c) Grandson (d) Grandmother (c) Father (d) Data inadequate
3. M is the son of P. Qis the granddaughter of O 10. Pointing to a girl. Mihir said "She is the only
who is the husband of P. How is M related to daughter of my grandfather's only child". How
O? is the girl related to Mihir?
(a) Son (b) Daughter (a) Daughter (b) Niece
(c) Mother (d) Father (c) Sister (d) Data inadequate
4. X and Y are brothers. R is the father of Y. S is 11. D is brother of K. R is father of D and S is
the brother of T and maternal uncle of X. What mother of M. How is K related to R?
is T to R? (a) Son (b) Daughter
(a) Mother (b) Wife (c) Son or daughter (d) Data inadeqate
(c) Sister (d) Brother 12. Pointing towards a boy, Veena said, “He is the
5. Based on the statements, given below, find out son of only son of my grandfather.” How is
who is the uncle of 'P' ? that boy related to Veena?
K is the brother of J. (a) Uncle (b) Brother
M is the sister of K. (b) Cousin (d) Data inadequate
P is the brother of N. 13. Arun said, “This girl is the wife of the
grandson of my moter.” Who is Arun to the
N is the daughter of J.
girl?
(a) K (b) J
(a) Father (b) Grandfather
(c) N (d) M
(b) Husband (d) Father-in-law
6. A is B's sister. C is B's mother. D is C's father.
14. Pointing to a woman, Naman said, “She is the
E is D's mother. Then how is a related to D ?
daughter of the only child of my grandmother.”
(a) Grandmother (b) Grandfather
How is the woman related to Naman?
(c) Daughter (d) Granddaughter
(a) Sister (b) Niece
7. A is the son of B, while B and C are sisters to
(b) Cousin (d) None of these
one another. E is the mother of C. If D is the
15. Pointing towards a girl in the picture, Sarita
son of E, which of the following statements
said. “She is the mother of Neha whose father
is correct?
is my son.” How is Sarita related to the girl in
(a) D is the maternal uncle of A
the picture?
(b) E is the brother of B
(a) Mother (b) Aunt
(c) D is the cousin of A
(b) Cousin (d) Mother-in-law
(d) B and D are brothers

80
16. A and B are brothers. C and D are sisters. A’s (a) Mother (b) Daughter
son is D’s brother. How is B related to C? (c) Sister (d) Aunt
(a) Father 8. A man showed a boy next to him and said -
(b) Brother ''He is the son of my wife's sister-in-law, but I
(c) Grandfather am the only child of my parents." How is my
(d) Uncle son related to him?
(a) Nephew (b) Cousin
TYPE - II (c) Brother (d) Uncle
1. Pointing to a photograph Vikas said ''She is TYPE - III
the daughter of my grandfather's only son''.
How is the related to vikas in the photograph? Directions for the following 3 (three) items:
(a) Father (b) Brother Examine carefully the following statements
(c) Sister (d) Mother and answer the three items that follow:
2. Pointing to a man in a photograph, a woman A, B, C, D, E and F are six friends in a
said. " His brother's father is the only son of group. Out of them, two are housewives, one
my grandfather. "How is the woman related is an engineer, one is accountant and one is an
to the man in the photograph ? advocate. There are two married couples in
(a) Daughter (b) Mother this group. Further it is given that :
(c) Aunt (d) Sister 1. Advocate is married to D who is a
3. Rajiv is the brother of Arun. Sonia is the sister housewife.
of Sunil. Arun is the son of Sonia. How is 2. None of the women of group are engineer
Rajiv related to Sunil? or accountant.
(a) Son (b) Brother 3. C is an accountant who is married to F
(c) Father (d) nephew who is a professor.
4. Introducing a man, a woman said, " His wife 4. A's wife is a housewife and E is not a
is the only daughter of my mother." How is housewife.
the woman related with the man? 1. Which of following is/are married couple (s)?
(a) Sister-in-law (b) Wife 1. A and B
(c) Aunt (d) Mother-in-law 2. A and D
5. Rahul and Robin are brothers. Pramod is 3. D and E
Robin's father. Sheela is pramod's sister. Select the correct answer using the code given
Prema is Pramod's niece. Shubha is Sheela 's below:
granddaughter. How is Rahul related to (a) 1 only (b) 2 only
Shubha? (c) 3 only (d) 2 and 3 only
(a) Brother (b) Cousin 2. Which of the following is/are correct
(c) Uncle (d) Nephew statement (s)?
6. Preeti has a son, named Arun. Ram is preeti's 1. E is an engineer and whose spouse is not
brother. Neeta too has a daughter named part of this group.
Reem. Neeta is Ram's sister. What is Arun's 2. C is an advocate who is married to
relationship to Reema ? housewife.
(a) Brother (b) Nephew Select the correct answer using the code given
(c) Cousin (d) Uncle below:
7. Pointing to a man, lady said, "His mother isthe (a) 1 only (b) 2 only
only daughter of my mother". How is the lady (c) Both 1 and 2 (d) Neither 1 nor 2
related to the man?

81
3. How many married members are in this 9. How is T related to S?
group? (a) Brother-in-law (b) Cousin
(a) 2 (b) 3 (c) Sister-in-law (d) Sister
(c) 4 10. If Z is the daughter of T, then how is B related
(d) Cannot be determined. to Z?
Directions fo the following 4 (four) items: (a) Uncle
Examine carefully the following statements (b) Father
and answer the four items that follow: (c) Aunt
P, Q, R, S, T and X are members of a (d) Cannot be determined
family. There are two married couples. Q is Directions : Study the following information
an engineer and the father of T. X is the carefully and answer the questions given be-
grandfather of R and is a lawyer. S is the low:
grandmother of T and is a housewife. There D is the father of A. D is married to P. P
is one engineer. One lawyer, one teacher, one is the mother of J. P has only one daughter. J
housewife and two student is the family. is married to U. U is the son of L.
4. Who is the husband of P? 11. How is J related to L?
(a) R (b) X (a) Daughter
(c) Q (d) S (b) Granddaughter
5. Which of the following are two married (c) Cannot be determined
couples? (d) Niece
(a) XS, QP (b) XS, QT 12. How is A related to U?
(c) XS, RP (d) TS, RX (a) Cannot be determined
6. Which of the following is definitely a group (b) Brother-in-law
of male members?
(c) Brother
(a) QXT (b) XT
(d) Sister
(c) QXP (d) QX
Directions : Study the following information
7. What is the profession of P? carefully and answer the questions given be-
(a) Housewife (b) Teacher low:
(c) Engineer (d) Engineer or Teacher U is the mother of D. S is the sister of D.
Directions : Study the following information L is the father of S. L has only one daughter.
carefully and answer the questions given be- M is the daughter of S. P is the daughter of D.
low: 13. If R married to S, then how is R related to U?
• S is the sister of B, K is the brother of B. (a) Grandson (b) Nephew
• Kis the son of L, L is married to Y. Y is (c) Son-in-law (d) Uncle
the daughter of J. 14. How is S related to P?
• Y has only one daughter. J is married to (a) Aunt (b) Sister
C. (c) Mother (d) Niece
• K is the brother of D. D is married to T. Directions : Study the following information
8. If P is the brother of C, then how is J realated carefully and answer the questions given be-
to P? low:
(a) Brother Q is the sister of T. T is the mother of D.
(b) Cousin T has only one son. D is the brother of J. J is
(c) Sister married to M. Y is the daughter of M.
(d) Cannot be determined 15. How is J related to Q?

82
(a) Son son in the family and D is the eldest. B is a
(b) Cannot be determined male.
(c) Daughter (UPSC-2011)
(d) Nephew 19. How is D related to E?
16. How is Y is related to Y? (a) Husband (b) Son
(a) Daughter (c) Father (d) Wife
(b) Daughter-in-law 20. Who are the females in the group ?
(c) Cannot be determined (a) C and E (b) C and D
(d) Niece (c) E and A (d) D and E
17. There is a family of 6 persons A, B, C, D, E 21. Whose wife is the teacher ?
and F. There are two married couples in the (a) C (b) D
family. The family members are lawyer, (c) A (d) B
teacher, salesman, engineer, accountant and 22. A joint family consists of seven members A,
doctor. D, the salesman is married to the lady B, C, D, E, F and G with three females. G is a
teacher. The doctor is married to the lawyer. widow and sister-in-law of D's father F. B and
F, the accountant is the son of B and brother D are siblings and A is daughter of B, C is
of E. C, the lawyer is the daughter-in-law of cousin of B. Who is E?
A. E is the unmarried engineer. A is the grand- (UPSC-2019)
mother of F. How is E related to F? I. Wife of F
(UPSC-2009) II. Grandmother of A
(a) Brother III. Aunt of C
(b) Sister Select the correct answer using the code given
(c) Father below :
(d) Cannot be established (cannot be deter (a) 1 and 2 only (b) 2 and 3 only
mined) (c) 1 and 3 only (d) 1, 2 and 3
18. In a group of five persons A, B, C, D and E, 23. A family of two generations consisting of six
there is a professor, a doctor and lawyer. A members P, Q, R, S, T and U has three males
and D are unmarried ladies, and do not work. and three females. There are two married
Of the married couple in the Group, E is the couples and two unmarried siblings. U is P's
husband. B is the brother of A and is neither a daughter and Q is R's mother-in-law. T is an
doctor nor a lawyer. Who is the professor? unmarried male and Sis a male. Which one of
(UPSC-2010) the following is correct?
(a) B (UPSC-2020)
(b) C (a) R is U's husband.
(c) A (b) R is S's wife.
(d) Cannot be determined with the available (c) S is unmarried.
data (d) None of the above
Read the following passage and answer the
(19-21) items that follow: TYPE - IV
A, B, C, D and E are members of the same
Directions: Read the following information
family. There are two fathers, two sons, two
carefully and answer the questions which fol-
wives, three males and two females. The
low:
teacher was the wife of a lawyer who was the
son of a doctor E is not a male, neither also a 'A X B' means 'A is father of B'
wife of a professional. C is the youngest per- 'A + B' means 'A is daughter of B'
'A ÷ B' means 'A is the mother of B'
83
'A – B' means 'A is brother of B' 8. Which of the following represents 'S is mother
1. If 'P ÷ R – Q X T', how is P related to T'? of T?
(a) Grandmother (a) S X M ÷ H – T
(b) Mother-in-law (b) S X M + H – T
(c) Sister (c) M X S ÷ H – T
(d) Grandfather (d) M X S ÷ H + T
2. If 'P ÷ Q + R X T' how is T related to Q'? 9. In J – F + R X B, how is R related to J?
(a) Aunt (a) Father
(b) Sister (b) Mother
(c) Brother (c) Paternal Aunt
(d) None of these (d) Cannot be determined
3. Which of the following means that R is wife 10. Consider the following including the
of P? Question and the Statements:
(a) P X R X Q – T (UPSC-2023)
(b) P ÷ T + R – Q There are 5 members A, B, C, D, E in a
(c) P ÷ R – Q + T family.
(d) P X T – Q + R Question: What is the relation of E to B?
4. If 'R – P ÷ J X Q' how is J related to R? Statement-1: A and B are a married couple.
(a) Son/daughter Statement-2: D is the father of C.
(b) Nephew Statement-3: E is D's son.
(c) Niece Statement-4: A and C are sisters.
(d) Grandson Which one of the following is correct in
5. If 'P + Q – R ÷ T' how is T related to P? respect of the above Question and Statements?
(a) Aunt (a) Statement-1, Statement-2 and Statement-
(b) Aunt/Uncele 3 are sufficient to answer the Question.
(d) Grandmother (b) Statement-1, Statement-3 and Statement
4 are sufficient to answer the Question.
(d) None of these
(c) All four statements together are sufficient
6. If 'P X T ÷ Q + R' how is R related to P?
to answer the Question.
(a) Daughter
(d) All four statements are not sufficient to
(b) Hunsband answer the Question.
(c) Son-in-law
(d) Son-in-law/Daughter-in-law
Directions : Study the following information
care fully to answer these questions.
'P X T' means 'P is wife of Q',
'P ÷ T' means 'P is father of Q',
'P + T' means 'P is son of Q',
'P – T' means 'P is sister of Q',
7. In H + I ÷ L, how is L related to H?
(a) Brother
(b) Sister
(c) Cousin
(d) Brother or Sister

84
Answer Sheet
Explanation
Explanations

TYPE - 1I
TYPE - I TYPE - III TYPE - IV
1. (c) 1. (b) 1. (a) 1. (c) A is the daughter of B. B is the mother
2. (b) 2. (a) 2. (d) of C.
3. (a) 3. (b) 3. (d) Therefore, B is the mother of A and C.
4. (b) 4. (c) 4. (b) Dis the brother of C. Therefore. D is the
5. (a) 5. (d) brother of A.
5. (a)
6. (d) 6. (c)
6. (d) 7. (b) 7. (d) 2. (b) P is brother of Q. Therefore. P is a male.
7. (a) 8. (d) 8. (a) R is mother of P and Q and R is daughter
9. (c) 9. (b) of S. S is the son of T. S is the grandfather
8. (d)
of P.
9. (b) 10. (a) 10. (c)
10. (c) 11. (c) ****** 3. (a) O is the husband of P. M is the son of P.

11. (a) 12. (a) Therefore M is the son of O.


12. (b) 13. (b)
4. (b) R is father of X and Y. S is maternal uncle
13. (d) 14. (d) of X and Y Considering the given options,
14. (a) 15. (d) it may be assumed that T is wife of R.
15. (d) 16. (b)
5. (a) P is the son of J. K is the brother of J.
TYPE - II 17. (d) Therefore, K is uncle of P.
1. (c) 18. (a)
6. (d) A is sister of B. C is mother of A and B.
2. (d) 19. (a)
20. (c) D is father of C. So, A is granddaughter
3. (d)
of D.
4. (b) 21. (d)
5. (c) ****** 7. (a) B is mother of A. C is daughter of E. D is
brother of B and C. Therefore, D is
6. (c)
maternal uncle of A.
7. (a)
8. (d) X and Y are children of A. Y is not son of
8. (b)
A. Therefore. Y is daughter of A.

TYPE - II

1. (c) The only son of grandfather (paternal) of


Vikas means father of Vikas.

Therefore, the girl is sister of Vikas.

2. (d) Only son of woman's grandfather means


father of that woman. Father of woman
is the father of man's brother and hence

85
father of that man. Therefore, the woman
is sister of the man in photograph.

3. (d) Rajiv and Arun are sons of Sonia.


Therefore, Rajive is nephew of Sunil.

4. (b) Only daughter of woman's mother means


the woman herself. Therefore, the woman
is wife of that man.

5. (c) Shubha is granddaughter of sheela, who


is sister of Pramod. Rahul is son of
Pramod.

Therefore, Rahul is uncle of Sheela.

6. (c) Arun is the son of Preeti. Ram is the


brother of Preeti and Neeta. Reema is the
daughter of Neeta. Thus, Arun is cousin
of Reema.

7. (a) Only daughter of lady's mother means the


lady herself. Therefore, the lady is mother
of that man.

8. (b) The boy is the son of man's brother-in-


law. Therefore, man's son is the cousin
of that boy.

86
12 (c) Four (d) Six
1. How many such 5s are there in the following
number sequence each of which is 5. In the series given below, count the number
immediately preceded by 3 or 4 but not of 9s, each of which is not immediately
immediately followed by 8 or 9? preceded by 5 but is immediately followed by
either 2 or 3. How many such 9s are there?
3595455358456735755452351
0 19265938393259293482698

(a) None (a) One (b) Three

(b) Three (c) Five (d) Six

(c) Four 6. How many 4's are there preceded by 7 but not
followed by 3?
(d) Five
5932174269746132874138325
(e) None of these
674395820187463
2. How many 5s are there in the folliwng number
(a) Three (b) Four
sequence which are immediately preceded by
7 and immediately followed by 6? (c) Five (d) Six

755945764598756764325678 7. Aman is 16th from the left end in a row of


boys and Vivek is 18th from the right end.
(a) One (b) Two
Gagan is 11th from Aman towards the right
(c) Three (d) Four and 3rd from Vivek towards the right end.
3. How many 6s are there in the following How many boys are there in the row?
number series, each of which is immediately (a) 40
preceded by 1 or 5 and immediately followed
(b) 42
by 3 or 9 ?
(c) 48
2637564296134163915692316
54321967163 (d) Data inadequate

(a) None (b) One (e) None of these

(c) Two (d) Three 8. Sam ranked ninth from the top and thirty-
eighth from the bottom in a class. How many
(e) None of these
students are there in the class?
4. How Many 7s immediately preceded by 6 but
(a) 45 (b) 46
not immediately followed by 4 are there in
the following series? (c) 47 (d) 48

7427643675357843767240674 9. A class of boys stands in a single line. One


3 boy is nineteenth in order from both the ends.
How many boys are there in the class?
(a) One (b) Two

87
(a) 27 (b) 37 15. Rohit is seventeenth from the left end of a row
(c) 38 (d) 39 of 29 boys and Karan is seventeenth from the
right end in the same row. How many boys
10. In a row of boys, Jeevan is seventh from the are there between 'them in the row?
start and eleventh from the end. In another row
of boys, Vikas is tenth from the start and (a) 3
twelfth from the end. How many boys are there (b) 5
in both the rows together? (c) 6
(a) 36 (b) 37 (d) Data inadequate
(c) 39 (e) None of these
(d) Cannot be determined 16. In a row of forty children, P is thirteenth from
(e) None of these the left end and Q is ninth from the right end.
11. In a class of 60, where girls are twice that of How many children are there between P and
boys, Kamal ranked seventeenth from the top. R if R is fourth to the left of Q?
If there are 9 girls ahead of Kamal, how many (a) 12 (b) 13
boys are after him in rank? (c) 14 (d) 15
(a) 3 (b) 7 (e) None of these
(c) 12 (d) 23 17. In a class of 35 students, Kunal is placed
12. Nitin ranks eighteenth in a class of 49 students. seventh from the bottom whereas Sonali is
What is his rank from the last? placed ninth from the top. Pulkit is placed
(a) 18 (b) 19 exactly in between the two. What is Kunal's
position from Pulkit?
(c) 31 (d) 32
(a) 9 (b) 10
13. In a row of boys, A is thirteenth from the left
and D is seventeenth from the right. If in (c) 11 (d) 13
this row A is eleventh from the right then 18. Richard is fifteenth from the front in a column
what is the position of D from the left? of boys. There were thrice as many behind him
(a) 6th as there were in front. How many boys are
there between Richard and the seventh boy
(b) 7th from the end of the column?
(c) 10th
(a) 33
(d) 12th (b) 34
(e) None of these (c) 35
14. In a row of boys, A is fifteenth from the left (d) Data inadequate
and B is fourth from the right. There are three
boys between A and B. C is just left of A. What (e) None of these
is C's position from the right? 19. George is fifth from the left and Peter is twelfth
(a) 9th (b) 10th from the right end in a row of children. If Peter
shifts by three places towards George, he
(c) 12th (d) 13th

88
becomes tenth from the left end. How many (c) 27
children are there in the row? (d) Data inadequate
(a) 21 (b) 22 Directions (24-29): Study the number series
(c) 23 (d) 24 below and answer the questions which follow:
(UPSC-2008)
(e) None of these 6789987977897876968
20. In a row of boys, if A who is tenth from the 977989 76887
left and B who is ninth from the right 24. How many such numbers are there in the given
interchange their positions, A becomes series each of which when subtracted from the
fifteenth from the left. How many boys are following number, has a difference of 2?
there in the row? (a) Three (b) Four
(a) 23 (b) 27 (c) Five (d) Nine
(e) None of these
(c) 28 (d) 31
25. Which of the following numbers is obtained
21. Students line up in a queue in which Ashish when 18th number from the left of the num-
stands fifteenth from the left and Sachin is ber series is added to 19th from the right?
seventh from the right. If they interchange their (a) 15 (b) 20
places, Sachin would be fifteenth from the (c) 10 (d) 17
right. How many students are there in the (e) None of these
queue? 26. How many nines are there in the given series
(a) 21 (b) 22 eachof which is immediately preceded by an
odd number?
(c) 29 (d) None of these
(a) One (b) Two
22. In a row of children, Deepti is ninth from the (c) Three (d) Four
left and Kashish is thirteenth from the right. (e) More than four
They exchange their positions and then Deepti 27. How many such even numbers are there in
becomes seventeenth from the left. Find the the given series each of which is immediately
new position of Kashish from the right end of followed by an even number?
the row. (a) None (b) One
(a) 20th (b) 21st (c) Two (d) Three
(e) More than three
(c) 27th (d) None of these
28. How many such odd numbers are there in the
23. In a row of girls, Rita and Monika occupy the given series each of which is immediately pre-
ninth place from the right end and tenth place ceded by an even number?
from the left end, respectively. If they (a) Five
interchange their places, then Rita and Monika (b) Seven
occupy seventeenth place from the right and (c) Nine
eighteenth place from the left respectively. (d) Eleven
How many girls are there in the row? (e) More than three
(a) 25 29. How many such Sevens are there in the given
series each of which is immediately preceded
(b) 26
by 9 and also immediately followed by 8?

89
(a) None (e) None of these
(b) One (UPSC-2010)
(c) Two 34. What should come next in the following num-
(d) Three ber series?
(e) More than three 975311864229753186422975318642975
30. What should come next in the following let- (a) 1
ter series? (b) 8
PPOPONPONMPONMLPONML (c) 5
(a) P (d) 3
(b) K (e) None of these
(c) J (UPSC-2010)
(d) I 35. Which of the following will be the next letter
(e) None of these in the following letter series?
(UPSC-2009) B C E H L ?
31. What should come next in the following let- (a) Q
ter series? (b) P
H G F E D C B A H G F E (c) O
D C B H G F E D C H (d) Cannot be determined
(a) F (e) None of these
(b) G (UPSC-2010)
(c) B 36. Which one of the following statements is cor-
(d) A rect?
(e) None of these (a) Weight of Mohan is greatest
(UPSC-2009) (b) Weight of Sohan is greatest
32. What should come next in the following let- (c) Weight of Rohan is greatest
ter series? (d) 'Whose weight is greatest' cannot be de
H G F E D C BAG F E D C B AG F E D termined
CB (UPSC-2019)
(a) E 37. Three Statements S1, S2 and S3 are given
(b) G below followed by a question
(c) F S1 C is younger than D, but older than A and
(d) B B.
(e) None of these S2 D is the oldest.
(UPSC-2009) S3 A is older than B.
33. What should come next in the following num- Question Who among A, B, C and D is the
ber series? youngest? Which one of the following is cor-
91827364591827365918273 rect in respect of the above statements and the
591827 question?
(a) 3 (a) S1 alone is sufficient to answer the ques-
(b) 1 tion
(c) 5 (b) S1 and S2 together are sufficient to an-
(d) 8 swer the question

90
(c) S2 and S3 together are sufficient to an- (a) 22
swer the question (b) 28
(d) S1 and S3 together are sufficient to an- (c) 32
swer the question (d) 38
(UPSC-2020) (UPSC-2022)
38. A is 16th from the left end in a row of boys
and V is 18th from the right end. G is 11th
from A towards the right and 3rd from V to-
wards the right end. How many boys are there
in the row?
(a) 40
(b) 41
(c) 42
Answer Sheet
(d) Cannot be determined due to insuffiecient
data
(UPSC-2020) 1. (d) 21. (c)
39. Consider two statements and a question 2. (a) 22. (b)
Statements 3. (d) 23. (b)
1. Each of A and D is heavier than each of 4. (b) 24. (b)
B, E and F, but none of them is the heavi- 5. (b) 25. (a)
est. 6. (d) 26. (c)
2. A is heavier than D, but is lighter than C. 7. (e) 27. (d)
Question Who is the heaviest among A, 8. (b) 28. (c)
B, C, D, E and F? 9. (b) 29. (b)
Which one of the following is correct in 10. (e) 30. (b)
respect of the statements 1 alone is sufficient 11. (c) 31. (b)
to answer the question? 12. (d) 32. (c)
(a) Statement 1 alone is sufficient to answer 13. (b) 33. (c)
the question 14. (a) 34. (c)
(b) Statement 2 alone is sufficient to answer 15. (a) 35. (a)
the question 16. (c) 36. (d)
(c) Both Statement 1 and 2 are sufficient to 17. (c) 37. (d)
answer the question 18. (c) 38. (b)
(d) Neither Statement 1 alone nor 2 alone is 19. (d) 39. (a)
sufficient to answer the question
20. (a) 40. (a)
(UPSC-2021)
40. Three persons A, B and C are standing in a
queue not necessarily in the same order. There
are 4 persons between A and B, 7 persons be-
tween B and C. If there are 11 persons ahead
of C and 13 behind A, What could be the mini-
mum number of persons in the queue?

91
13 and 25 have V.C.R., Only 10 families have
1. There are some flowers in a basket and they
double in size after every minute. After 30 all three and all V.C.R owners also have
minutes the basket is full, then after how T.V., If some families have only radios,
many minutes the basket was half full? how many have only TVs?

(a) 15 minutes (b) 20 minutes (a) 30 (b) 35

(c) 29 minutes (d) 12 minutes (c) 40 (d) 45

2. A group of ten girl students shake hands 6. 10 or 11 Which number is bigger?


11 11-

with each other. What is the total number (a) both are equal
of handshakes?
(b) 1011 > 1110
(a) 20 (b) 56
(c) 1110 > 1011
(c) 90 (d) 45
(d) cannot be determined without using
3. Somewhere there are some boys and dogs. the table
If the total number of heads is 7 and the
total number of legs is 20, then tell how 7. There are 8 members in a circle of friends.
many boys and how many dogs are there. They have congratulated each other by
sending greeting cards on Diwali. Tell how
(a) 2 boys and 5 dogs many greeting cards will be used by the
(b) 3 boys and 4 dogs said circle of friends?

(c) 4 boys and 3 dogs (a) 50 (b) 64

(d) 5 boys and 2 dogs (c) 56 (d) 60

4. A bus leaves from Delhi carrying men and 8. The shadow of an 18 feet high pole is 20
half their number of women. On reaching feet, then what will be the length of the
Meerut, ten men get down and five women shadow of a 27 feet high pole?
board. Now the number of men and women (a) 36 feet (b) 30 feet
in the bus is equal. How many passengers
boarded the bus initially from Delhi? (c) 34 feet (d) 40 feet

(a) 36 (b) 45 9. When a bus started, a certain number of


passengers were sitting in it. At the first
(c) 15 (d) 30 stop, half the passengers got down from
5. Out of 100 families in the neighbourhood, the bus and 35 passengers boarded the bus.
50 have radio, 75 have T.V. (Doordarshan) At the second stop 1/5 of the passengers

92
got down and 40 got on. After that there basket?
were 80 passengers in the bus and it
(a) 20 (b) 30
proceeded towards the destination without
stopping. How many passengers were there (c) 35 (d) 40
in the bus initially?
1
(a) 25 (b) 30 13. Which of the following is the 1 number
2
(c) 40 (d) 50 1
which when added to and 1 multiplied
2
1 by gives the same result?
10. One-fourth   of Ravi's life was spent
4
(a) 1 (b) 3
1
as a boy, one-fifth   as a teenager, one- (c) 5 (d) 7
5

1 14. The ratio of the present ages of father and


third   as a man and thirteen (13) years son is 5 : 1 respectively. After seven years
3
this ratio becomes 3:1. What is the present
in old age. What is his present age?
age (in years) of the son?
(a) 70 years (b) 80 years
(a) 8 (b) 7
(c) 60 years (d) 65 years
(c) 6 (d) 5
11. In an office 1/3 of the employees are
15. At present, the ages of Arun and Deepak
women, 1/2 of the women are married and
are in the ratio 4 : 3. After 6 years Arun's
1/3 of the married women have children.
age will be 26 years. What is the age of
If 3/4 of the men are married and 2/3 of
Deepak at present?
the married men also have children, then
what fraction of the employees are without (a) 15 years (b) 19 years
children?
(c) 24 years (d) 12 years
5 4 16. Two years ago a mother was four times as
(a) (b)
18 9 old as her son. Six years hence his age will
be 10 years more than twice the age of his
11 17 son. What is the present ratio of their ages?
(c) (d)
18 36
(a) 2:1 (b) 38:11
12. There are three baskets of fruits. The first
basket has twice as many fruits as the (c) 19 : 7 (d) 3 : 1
second basket. The third basket contains 17. 24 carat gold is pure gold.
3/4 of the fruits of the first basket. The 18 karat gold is 3/4 gold.
average of fruits in all the baskets is 30. 20 karat gold is 5/6 savarna.
How many fruits are there in the first So what will be the ratio of 18 carat gold

93
to 20 carat gold? three. 5 employees can drink any of the
(a) 5:8 (b) 10:9 three. How many drink only tea?
(c) 15:24 (d) 9:10 (a) 21 (b) 22
18. In a row, 25 trees are planted at equal (c) 18 (d) 20
distance from each other. The distance
22. In a group of 20 people, 8 read Hindi, 11
between the 1st and 25th tree is 30 m. What read English, while 5 read neither. How
is the distance between the 3rd and 15th
many of them read both Hindi and English?
tree?
(a) 8 (b) 6
(a) 8 m (b) 15 m
(c) 4 (d) 2
(c) 16 m (d) 18 m 23. Five children take part in a tournament. Each
19. There are 19 hockey players in a club. On one has to play every other one. How many
games must they play?
a particular day 14 players were wearing
prescribed hockey shirts. None of them was (a) 8 (b) 10
without hockey pants or shirt. 11 players (c) 24 (d) 30
were wearing the prescribed hockey pants.
1
How many players were in full uniform? 24. In a certain office, of the workers are
3
(a) 6 (b) 9
1 1
women, of the women are married and
(b) 7 (d) 8 2 3

20. In a department, 24 employees know 3


of the married women have children. if of
typewriting and 11 know shorthand. 25 4
Knows how to use computer. 7 knows both 2
the men are married and of the married men
typewriting and shorthand, 4 knows 3
shorthand and computer. 12 know typing have children, what part of the workers are
and computer and 3 know all three. If there without children?
are 50 employees in that department, then
find the number of employees who do not 5 4
(a) (b)
know any of the three jobs. 18 9

(a) 40 (b) 10 11 17
(c) (d)
18 36
(b) 47 (d) 33
25. There are some benches in a classroom. If 4
21. A government meeting was attended by students sit on each bench, then 3 benches are
130 departmental employees. 66 of them left unoccupied. However, if 3 students sit on
drink tea, 56 drink coffee and 63 drink each bench, 3 students are left standing. How
juice. 27 can drink tea or coffee, 25 can many students are there in the class?
drink coffee or juice and 23 can drink juice (a) 36 (b) 48
or tea. 5 employees can drink any of the (c) 56 (d) 64
94
26. A man has a certain number of small boxes to of apples from the garden, hid the loot and
pack into parcels. If he packs 3, 4, 5 or 6 in a went to sleep. Before retiring they did some
parce, he is left with one over ; if he packs 7 quick counting and found that the fruits were
in a parcel, none is left over. What is the less than a hundred in number. During the
number of boxes, he may have to pack? night one boy awoke, counted the apples and
(a) 106 (b) 301 found that he could divide the apples into three
equal parts if he first took one for himself. He
(c) 309 (d) 400
1
27. A bird shooter was asked how many birds he then took one apple, ate it up and took of
3
had in the bag. He replied that there were all
the rest, hid them separately and went back to
sparrows but six, all pigeons but six, and all
sleep. Shortly thereafter, another boy awoke,
ducks but six. How many birds he had in the
counted the apples and he again found that if
bag in all?
he took one for himself the loot could be
(a) 9 (b) 18 divided into three equal parts. He ate up one
(c) 27 (d) 36 apple, bagged of the remainder, hid them
separately and went back to sleep. The third
28. What is the product of all the numbers in the
boy also awoke after sometime, did the same
dial of a telephone?
and went back to sleep. In the morning, when
(a) 1, 58, 480 (b) 1, 59, 450 all woke up, and counted apples, they found
(c) 1, 59, 480 (d) None of these that the remaining apples again totaled 1 more
29. If you write down all the numbers from 1 to than could be divided into three equal parts.
100, then how many times do you write 3? How many apples did the boy steal?
(a) 11 (b) 18 (a) 67 (b) 79
(c) 20 (d) 21 (c) 85 (d) None of these
30. A Monkey climbs 30 feet at the beginning of 33. A girl counted in the following way on the
each hour and rests for a while when he slips fingers of her left hand: She started by calling
back 20 feet before he again starts climbing the thumb 1, the index finger 2, middle finger
in the beginning of the next hour. If he begins 3, ring finger 4, little finger 5 and then reversed
his ascent at 8.00 a.m., at what will he first direction calling the ring finger 6, middle
touch a flag at 120 feet from the ground? finger 7 and so on. She counted upto 1994.
(a) 4 p.m. (b) 5 p.m. She ended counting on which finger?

(c) 6 p.m. (d) None of these (a) Thumb (b) Index finger

31. A bus starts from city X. The number of (c) Middle finger (d) Ring finger
women in the bus is half of the number of men, 34. There are deer and peacocks in a zoo. By
is equal. In the beginning, how many counting heads they are 80. The number of
passengers entered the bus? their legs is 200. How many peacocks are
(a) 15 (b) 30 there?

(c) 36 (d) 45 (a) 20 (b) 30

32. One night, three naughty boys stole a basketful (c) 50 (d) 60

95
35. There are 50 students admitted to a nursery 40. A man climbing up a wall of 24 metres high.
class. Some students can speack only English He climbs 16 m on one day but slipped back
and some can speack only Hindi. Ten students by 3 m 40 cms in the evening. How far had
can speack both English and Hindi. If the the man reached on that day?
number of studetnts who can speack English (a) 19 m, 40 cms (b) 12.6 m
21, then how many students can speak Hindi,
How many can speack only Hindi and how (c) 11.4 m (d) 12 m 40 cms
many can speak only English? 41. Sumit wants to get his clothes washed. The
(a) 39, 29 and 11 respectively laundry serveice charges in his city are such
that for the first kg. of clothes, he is charged
(b) 37, 27 and 13 respectively Rs. 150 and after that, he is charged Rs. 50
(c) 28, 18 and 22 respectively for every kg. that follows. if he gets 10 kg. of
(d) 21, 11 and 29 respectively clothes washed, how much mondy does he pay
for the laundry service?
36. A man is engaged for planting trees for 10
hours. He plants 10 trees in an hour. He (a) Rs. 250 (b) Rs. 450
takes rest for 30 minutes after every hour. (c) Rs. 650 (d) Rs. 600
How many trees does he plant in 10 hours? 42. Rahul has Rs. 340 in the denomi- nations of
(a) 100 (b) 50 Rs. 2 notes, Rs. 5 notes and Rs. 10 notes. The
(c) 70 (d) 45 num- ber of notes of each denomina- tion is
equal. What is the total number of notes that
37. If 4 cats can kill 4 rats in 4 minutes, how many Rahul has ?
minutes will it take 8 cats to kill 8 rats?
(a) 40 (b) 60
(a) 8 (b) 4
(c) 20 (d) 80
(c) 2 (d) 16
43. Twenty teams take part in a football
38. Ranga got thrice as many sums wrong as he tournament. Each team has to play every other
got right in a mathematics assisgnment. if he team. How many games would be played in
attempted 72 sums in all, how many did he the tournament?
solve correctly in his mathematics
assignment? (a) 195 (b) 190

(a) 16 (b) 20 (c) 170 (d) 180

(c) 14 (d) 18 44. There are fourteen teams play- ing in a


tournament. If every team plays one match
39. Vineet, Rajesh and Kriti have different with every oth- er team, how many matches
amounts of mondy with them. Rajesh has just will be played in the tournament?
double the amount of money than Vineet. The
total amount of money that rajesh and Kriti (a) 66 (b) 78
have is Rs. 147. Kriti has Rs. 6 more than the (c) 91 (d) 101
amount Vineet has. How much money does 45. Three persons A, B and C have different
Kriti have? amount of rupees with them. If A gives Rs. 11
(a) Rs. 72 (b) Rs. 50 to C, A will have equal amount as B has. A
(c) Rs. 53 (d) Rs. 64 and B together have total Rs. 245. How many
96
rupees does B have? (b) Rs. 260
(a) Rs. 116 (b) Rs. 118 (c) Rs. 280
(c) Rs. 117 (d) Rs. 119 (d) Rs. 180
46. Three persons A, B and C have different 50. 60 students participated in one or more of the
amounts of rupees with them. If C takes Rs.11 three competitions, i.e., Quiz, Extempore and
from A, C will have equal amount as B has. B Debate. A total of 22 students participated
and C together have total Rs. 111. How many either in Quiz only or in Extempore only. 4
rupees does C have? students participated in all three competitions.
(a) 50 (b) 70 A total of 14 students participated in any of
the two competitions only. How many students
(c) 60 (d) 65
participated in Debate only ?
47. Three friends A, B and C have different (a) 22
amounts of rupees with them. If A takes. 10
from B, then A will have . 4 less than C. A and (b) 11
C together have total. 330. How many rupees (c) 20
does C have? (d) 14
(a) 170 (b) 174 51. In a group of 73 friends, 14 friends go to Gym
(c) 172 (d) 176 and Karate classes both, whereas 7 friends
48. Among 160 players in a tournament, 57 did neither go to Gym class nor to Karate class. If
not participate in any of the three games, i.e. a total of 36 friends go to Gym class, then how
Cricket, Hockey and Badminton. A total of 37 many friends go to only Karate class?
players participated in only one game, 10 (a) 30
players participated in both Cricket and (b) 23
Hockey but not in Badminton, 9 players
participated in both Hockey and Badminton (c) 44
but not in Cricket, and 13 players participated (d) 32
in both Cricket and Badminton but not in 52. If you write down all the numbers from 1 to
Hockey. How many students participated in 100, then how many times do you write 3?
all the three games?
(UPPSC-2020)
(a) 30 (b) 34
(a) 20
(c) 32 (d) 38
(b) 11
49. A private taxi company charges a fixed charge
(c) 21
along with a per kilometre charge based on
the distance covered. For a journey of 24 km, (d) 10
the charges paid are Rs. 368 and for a journey 53. A monkey climbs a 12m high slippery pillar.
of 22 km, the charges paid are Rs. 464. How In his first minute, he climbs 2m and in the
much will a person have to pay for travelling next minute, he slips one meter down. In this
a distance of 15 km? way, how much time will he take to reach the
(a) Rs. 290 top of the pillar?

97
(UPPSC-2020)
Answer Sheet
(a) 10 min
(b) 21 min 1. (c) 2. (c) 3. (c) 4. (b) 5. (c)
(c) 12 min 6. (b) 7. (c) 8. (b) 9. (b) 10. (c)
11. (c) 12. (d) 13. (b) 14. (b) 15. (a)
(d) 11 min
16. (b) 17. (d) 18. (b) 19. (a) 20. (b)
54. There ar five persons P, Q, R, S and T each 21. (a) 22. (c) 23. (b) 24. (c) 25. (b)
one of whom has to be assigned one task. 26. (b) 27. (a) 28. (d) 29. (c) 30. (c)
Neither P nor Q can be assigned Task-1. Task- 31. (d) 32. (b) 33. (b) 34. (d) 35. (a)
2 must be assignedc to either R or S. In how 36. (c) 37. (b) 38. (d) 39. (c) 40. (b)
many ways can the assignment be done? 41. (d) 42. (b) 43. (b) 44. (c) 45. (c)
46. (a) 47. (c) 48. (b) 49. (b) 50. (c)
(UPSC-2023)
51. (a) 52. (a) 53. (c) 54. (d) 55. (b)
(a) 6
(b) 12
(c) 18
(d) 24
55. In how many ways can a batsman score exactly
25 runs by scoring single runs, fours and sixes
only, irrespective of the sequence of scoring
shots?
(UPSC-2023)
(a) 18
(b) 19
(c) 20
(d) 21

98
Explanation 20
= ×27 = 30 feet
18

1. (c) The basket will be half full in the 29th 9. (b) Let  be the traveler in the beginning.
minute. x
Passengers after first stop =  35
In the 30th minute it will double and 2
the basket will be filled. after the second stop =
2. (c) Total number of handshakes = (3) 10× xx 
(10 × 1) = 90   35   40  80
22 
3. (c) Four boys and 3 dogs
x (80  40)
Because head = 4 + 3 = 7 and foot = 4   35  5
2 4
× 2 + 3 × 4 = 8 + 12 = 20
4. (b) Let  women board the bus in Delhi. x
 50  35  15

2
Thus, number of men = 2 
According to Question,  x  30
10. (c) Let the present age of Ravi be  years.
2  - 10 =  + 5
According to Question,
2  -  = 10 + 5
 = 15 x x x
   x  13
4 4 3
Initially total number of passengers
leaving from Delhi = 3  = 3×15 = 45 15 x  12 x  20 x
  x  13
5. (c) 25 has VCR and each one having BCR 60
has TV.  47  = 60  – 780
So persons who have TV but not VCR  60  – 47  = 780
= 75 - 25 = 50  13  = 780
10 persons have TV, Radio and BC.R. 780
x  60 year
Hence 40 persons have only TV 13

6. (b) Because there is a difference of 1 in 11. (c) Let the total number of employees in
the base and there is a difference of 1 the office = 
in the powers, then the number with x
Number of women employees =
lower base and higher power is larger. 3
7. (c) Number of greeting cards intended to  Number of male
be used = 8×7 = 56 x 3x  x 2 x
employees  x   
3 3 3
20
8. (b) Shadow of one foot high pole = Number of married employees
18

 Shadow of 27 feet high pillar

99
x 1 x in the first basket 2 x  2  20  40
  
6 2 6
3 3 6 9 1
13. (b) 3  4
Number of married female employees 2 2 2 2
x 1 x
with children    3 9 1
6 3 18 3   4
2 2 2
Number of married male employees 14. (b) Let the present age of son = x years
2x 3 x Hence, present age of father = 5 x
  
3 4 18 years
Number of married male employees
5x  7 3
x 2 x According to Question, 
with children    x7 1
2 3 3
 5 x + 7 = 3 x + 21
Number of employees who have  5 x - 3 x = 21 - 7
children
 2 x = 14
x x 6x  x 7x
     x =7
3 18 18 18
15. (a) Let the present age of Arun be 4 x
Number of employees who do not
years and that of Deepak be 3 x years.
h a v e c h i l d r e n
six years later Arun's age = 4 x + 6 =
7 x 18 x  7 x 11
 x   x 26
18 18 18
 4 x = 26 - 6
12. (d) Let the number of fruits in the second
basket =  20
 x 5
Therefore, the number of fruits in the 4
first basket = 2   Present age of Deepak = 3 x = 15 years
Number of fruits in the third basket 16. (b) Let the present age of son = x years

3 3x Let the present age = y years


 2x  
4 2 According to Question,
4 ( x - 2) = y - 2
3x
Now,  2 x  x   30  3  4x - 8 = y- 2
2
 4 x -y = 6 .....(i)
4 x  2 x  3x
Or,  30  3 2( x + 6) + 10 = y + 6
2
Or, 9 x  30  3  2  2 x + 12 + 10 = y + 6
 2 x -y = -16 ...(ii)
30  3  2
x   20 Number of fruits From equation (i) and (ii) x = 111
9

100
y = 4×11 - 6 = 38 = 10
Required ratio = 38:11 21. (a) There are 130 employees in total.
17. (d) 18 carat : 20 carat

3 5
Hence the golden ratio, : or, 18:
4 6
20
or, 9: 10
Hence, the correct answer would be
option (4).
18. (b) Distance between two trees
21 The employees drink tea only.
30 5
=  m 15 The employees do not drink any
24 4
drinks.
Distance between 3rd and 15th tree =
22. (c) 8 people study Hindi.
5
 12  15m 11 people study English.
4
19. (a) Number of people who read Hindi,
English or both
 (11 + 8) – 15 = 4

6 players were in full uniform.


20. (b)

Number of employees knowing


Typing, Computer or Shorthand or any
two or all the three functions = 8 + 9 +
12 + 4 + 3 + 1 + 3 = 40
Number of employees who do not
know any of the three jobs = 50 – 40

101
14 (c) (d)
TYPE - I

6. 7 9 17 8 10 21
40 54 ?
1. 27 54 42 84 ? 42
4 5 6 3 3 6

(a) (b)
9 14 7 (c) (d)

(a) (b) 7. 21
3 4 27 ? 33
(c) (d) 18 23 27
7 5 9 6 11 6
13 19 71 9 128 32
(a) (b)

2. 4 10 ? (c) (d)

(a) (b) 8. 15 22 13 11 21 ?
(c) (d) 36 16 64
16 9 7 9 15 13
3. 12 18 30 16 32 40 36 18 27
(a) (b)

6 8 ?
(c) (d)

(a) (b) 9. 6 4 7 8 8 9
104 155 ?
(c) (d)
10 8 9 11 5 14

4. 42 39 22 36 17 43
(a) (b)
(c) (d)
81 58 ? 10. 3 8
5 6 3 7
26 62 ?
(a) (b) 2 4 5 4 5 8
(c) (d)
(a) (b)
5. 3 5
3 5 4 7 (c) (d)
39 51 ?
6 3 5 4 5 4

(a) (b)

102
(a) (b)
3 4 7
11. 3 63 4 6 66 5 6 ? 3 (c) (d)

5 3 5
5 3 6
17.
(a) (b)
5 7 ?
(c) (d)
2 8 6 4 9 9

16 64 256
(a) (b)
12. 1 10 9 25 27 100 36 ? 144
(c) (d)
4 16 64
18.
(a) (b) 28 4 25 5 ? 3

(c) (d) 7 11 5 10 8 11

4 5 6
(a) (b)
13. 1 30 3 2 42 4 3 ? 5
(c) (d)
2 3 4
19. 5 4 3 8 9 4
(a) (b)
(c) (d) 20 9 24 11 ? 13

2 3 6 (a) (b)
14. 3 33 2 4 54 2 3 ? 5
(c) (d)
4 5 4

(a) (b) 20.

(c) (d)

15. (a) (b)


2 4 6
3 10 ?
(c) (d)
3 5 7
21.
(a) (b)

(c) (d)

16. 5 13 17 (a) (b)


3 5 ?
(c) (d)
4 12 15

103
(c) (d)
22.

4.
(a) (b)
(c) (d)

23.

(a) (b)

(a) (b) (c) (d)


(c) (d)
5.
TYPE - II

1.

(a) (b)

(a) (b) (c) (d)

(c) (d)
6.

2.

(a) (b)
(a) (b) (c) (d)
(c) (d)
7.
3.

(a) (b) (a) (b)

104
(c) (d)
12.
8.

(a) (b)
(a) (b)
(c) (d)
(c) (d)
14. F J N

9. M Q U
O S ?
(a) U (b) W
(c) X (d) Y

(a) (b) 15. 4 3 6


2 5 4
(c) (d)
3 7 ?

10. 24 105 120


(a) 5 (b) 4
(c) 6 (d) 7
16. 5 7 8
4 6 6
(a) (b) 2 3 ?

(c) (d) 10 14 12
(a) 2 (b) 4
11.
(c) 61 (d) 3
17. 9 8 6
8 7 5
6 5 ?
(a) (b)
4 3 1
(c) (d) (a) 3 (b) 4

105
(c) 2 (d) 1
23.
18. 8 6 4
8 10 12
4 2 ?
256 120 192
(a) (b)
(a) 8 (b) 4 (c) (d)
(c) 2 (d) 0
24.
19.

(a) (b)
(c) (d)
(a) 22 (b) 18
25.
(c) 26 (d) 24
20. 9 30 21
6 ? 6
12 40 28 (a) (b)

(a) 20 (b) 33 (c) (d)


26. Following is a matrix of certain entries. The
(c) 37 (d) 70
entries follow a certain trend row-wise.
21. 13 9 24 Choose the missing entry (?) accordingly.
11 ? 6
16 20 10
(a) 11 (b) 20
(c) 19 (d) 16
(UPSC-2021)
22. (a) 9B
(b) 3A
(c) 3B
(d) 3C

(a) (b)
(c) (d)

106
27. Find the missing number in the following 30. Which of the following character placed at the
figure. sign of question mark (?) to complete the
matrix?

(UPPSC-2021)

(UPPSC-2020) (a) 7C

(a) 94 (b) 88 (b) 10C

(c) 92 (d) 86 (c) 12C

28. Study the given pattern carefully and select (d) 13C
the number that can replace the question mark
(?)

Answer Sheet

TYPE - I
1. (b) 2. (C) 3. (b) 4. (b) 5. (b)
(CGPSC-2022) 6. (b) 7. (b) 8. (a) 9. (d) 10. (b)
(a) 37 (b) 51 11. (d) 12. (d) 13. (a) 14. (b) 15. (b)
16. (b) 17. (c) 18. (b) 19. (a) 20. (b)
(c) 26 (d) 43
21. (a) 22. (b) 23. (b) ************
29. What number would come in the place of
question mark?
TYPE - II
1. (c) 2. (c) 3. (d) 4. (a) 5. (d)
6. (b) 7. (c) 8. (b) 9. (d) 10. (a)
11. (a) 12. (c) 13. (d) 14. (b) 15. (a)
16. (b) 17. (a) 18. (b) 19. (d) 20. (a)
21. (a) 22. (b) 23. (a) 24. (b) 25. (a)
26. (c) 27. (a) 28. (d) 29. (b) 30. (b)

(UPPSC-2020)
(a) 72 (b) 84
(c) 68 (d) 66

107
64 = 8  8 + 13 = 21,
Explanation
TYPE - I 15 + 8 = 23

1. (b) 21 [ 27  2 = 54, 42  2 = 84 That means '23' should come in place of


Thus 21  2 = 42] '?'.

9. (d) (6  4) + (10  8) = 24 + 80 = 104


13  19 71  9
2. (c) 4 = , 10 =
8 8 (7  8) + (9  11) = 56 + 99 = 155

128  32 160 Thus,


Similarly ? =  = 20
8 8
(8  9) + (5  14) = 72 + 70 = 142
3. (b) All the three numbers above the bottom
10. (b) Just as,
number are perfectly divisible.
5  4 + 2  3 = 26
4. (b) 42 + 39 = 81
22 + 36 = 58 5  4 + 6  5 = 62

Similarly,
17 + 43 = 60
3  8 + 5  7 = 24 + 35 = 59
5. (b) (5  5) + (3  3) = 39

(7  5) + (4  4) = 51 11. (d)

Similarly

(5  5) + (3  4) = 37 5  4  3 = 60; 60 + 3 = 63
6. (b) (7 + 9 + 5 + 4)  2 – 10 = 40

(17 + 8 + 3 + 6)  2 – 14 = 54

(10 + 21 + 6 + 3)  2 – 18 = 62
7. (b) 3 + 18 = 21 3  5  4 = 60; 60 + 6 = 66

4 + 23 = 27
? + 27 = 33
? = 33 – 27 = 6
5  3  7 = 105;
8. (a) 36 = 6  6 + 9 = 15,
105 + 6 = 111
6 + 16 = 22
12. (d) Just as,
16 = 4  4 + 7 = 11, 4 + 3 = 13
1  4  9  16

108
= 1 + 2 + 3 + 4 = 10 17. (c) 5 + 5 = 2 + 8
3+7=6+4
And 25  16  100  64
6+?=9+9
= 5 + 4 + 10 + 8 = 27
 ? = 18 – 6 = 12
Similarly, 18. (b) Hence, the correct answer will be option
(1). The sum of the upper left number and
36  64  144  256
the lower left number is equal to the lower
= 6 + 8 + 12 + 16 = 42 right number while the product is equal
to the upper left number. first order
13. (a) (4 + 3 + 2 + 1)  3 = 30
7 + 4 = 11 and 7  4 = 28
And, (5 + 4 + 3 + 2)  3 = 42
Second order
 (6 + 5 + 4 + 3)  3 = 54
5 + 5 = 10 and 5  5 = 25
14. (b) The sum of the squares of the numbers Third order
outside the circle is equal to the number
inside. 8 + 3 = 11 8  3 = 24

(3)2 + (2)2 + (2)2 + (4)2 19. (a) The sum of the upper two numbers is
equal to the lower right number while
= 9 + 4 + 4 + 16 = 33
their product is equal to the lower left
(3)2 + (2)2 + (5)2 + (4)2 number.
= 9 + 4 + 25 + 16 = 54 first order
Similarly, 5 + 4 = 9 and 5  4 = 20
(6)2 + (5)2 + (4)2 + (3)2 Second order
= 36 + 25 + 16 + 9 = 86 3 + 8 = 11 and 3  8 = 24
15. (a)  3  2 ÷ 2 = 3 Third order
and 5  4 ÷ 2 = 10 9 + 4 = 13 and 9  4 = 36

 7  6 ÷ 2 = 21 20. (b) Just as,

Hence, the correct answer would be option (1). 89 – (17 + 8) = 52 + 12

16. (b)  32 + 42 = 3 = 16 = 25 = 52 167 – (24 + 49) = 61 + 33

and 52 + 122 = 25 + 144 =169 = 132 Similarly,


 172 – 152 = 289 +225 ? – (61 + 15) = 121 + 33

= 54 = 82  8  ? = 154 + 76 = 230

109
In other words ‘?’ 230 will come in place 169 = (3 + 5 + 5 )2
of .
Because 132 = 169 It happens
21. (a) (56 + 15) – (22 + 8) = 41
3. (d) 21 + 4 = 25, 30 + 6
(46 + 9) – (10 + 6) = 39
= 33, 27 + 6 = 33
(34 + 11) – (14 + 6) = 25
19 + 3 = 23
Hence, '25' should come in place of '?'.
Hence, '19' should come in place of '?'.
22. (b) 5  6 + 3  3 = 39
4. (a) (21 + 1) – 2 = 22 –2 = 20
and 7  5 + 4  4 = 51
(22 + 2) – 1 = 24 – 1 = 23
 5  5 + 3  4 = 37
(? + 5) – 2 = 43
(6)2 + (5)2 + (4)2 + (3)2  ? = (43 + 2) – 5
= 36 + 25 + 16 + 9 = 86  ? = (45 – 5) = 40
23. (b) 25 + 45 + 35 + 30 = 135 5. (d) 4 – 3 = 1, 1 – 1 = 0

135 9 – 6 = 3, 6–2=4
 27
5 12 – 8 = 4, 6–3=3
60 + 20 + 40 + 30 = 150 12 – 10 = 2, 4–1=3

150
 30 6. (b)
5

25 + 40 + 35 + 65 = 165

165
 33
5
TYPE - II 8. (b) 16+13–14=15

1. (c) 9 + 3 = 12 28 + 12 – 10 = 30

12+(2  3) = 18 29 + 16 – 15 = 30

18+(2  6) = 30 2 2 2
9. (d) 7  14  28  56
2 2 2
30+(2  12) = 54 4   8  167   32
2 2 2
2. (c) 49 = (1 + 4 + 2)2 9   18   367   72

64 = (2 + 2 + 4)2 10. (a) 22  4


Similarly 32  9

110
52  25 1
 1.5  8  1.5  12
Hence : 92  81 2

11. (a) The second number in each column is the 17. (a) 9 
–1
 8 
–2
6
square root of the first number. Then, the 8 
–1
 7 
–2
5
sum of the second and third numbers is
equal to the lowest number. 6 
–1
 5 
–2
3
4 
–1
 29 
–2
1
81 = 9 or 90 – 11 = 9
18. (b)  256 ÷ 8 ×8 = 4
12. (c)
120 ÷ 6 × 10 = 2
12, 13, 15, 18, 22, 27, 33, 40 192 ÷ 12 × 4 = 4

+1 +2 +3 +4 +5 +6 +7 19. (d) 3 × 2 = 6 ; 6 × 2 = 12;
13. (c) 5 + 4 = 9 and 9  2 = 18 12 × 2 = 24; 24 × 2 = 48

6 + 3 = 9 and 9  3 = 27 20. (a) The third number is obtained by


subtracting the first number from the
12 + 4 = 16 and ? second number in each row.

96 First line
= = 6
16 30 – 9 = 21

14. (b) F M O; J Q S Second line


? – 6 = 14
+7 +2 +7 +2
 ? = 14 + 6 = 20
N U W

+7 +2
Third line
40 – 12 = 28
15. (a)  4  2  3 = 24
and 3  5  7 = 105 21. (a) The sum of all the three numbers in each
column is 40.
 6  4  ? =120
first column
120
 ?=  5 13 – 11 + 16 = 40
24

1 Second column
16. (b)  2  5  2  10
2 9 + ? + 20 = 40

6  ? = 40 – 29 = 11
 2  7  2  14
3
Third column
24 + 6 + 10 = 40

111
22. (b) first Box  3 + 9 = 12
(7 + 5) + (2×2)
 12 + 4 = 16
first column
(9 + 7) + (4×4)
 16 + 16 = 32

Third column
(8 + 6) + (3×3)
 14 + 9 = 23

23. (a) 12 + 8 = 20

20 + 16 + 36
36 + 32 = 68
68 + 64 = 132
132 + 128 = 260
24. (b) 6 + 18 = 24
24 + 36 = 60
60 + 60 = 120
120 + 90 = 210

210 + 126 = 336

25. (a) first column

16  36

 4 + 6 = 10
Second column

25  64
 5 + 8 = 13
Second column

9  81

112
15 (a) 1, 2, 4, 5, 3, 6 (b) 4, 1, 2, 3, 4
TYPE - I
(b) 2, 1, 4, 3, 5, 6 (d) 5, 2, 3, 4, 1
Direction: Which of the following four options 8. (a) accident (b) judge (c) doctor (d) lawyer
will represent the meaningful order of the words (e) police
given below?
(a) 1, 3, 4, 2, 5 (b) 1, 3, 5, 4, 2
1. (a) honey (b) flower (b) fly (d) mom
(c) 1, 2, 3, 4, 5 (d) 1, 2, 5, 4, 3
(a) b, a, d, c (b) b, c, a, d
9. (a) Rain (b) Evaporation (c) Water (d)
(c) d, c, b, a (d) a, c, d, b Condensation (e) Cloud
2. (1) plant (2) food (3) seed (4) leaf (5) flower (a) 1, 3, 2, 4 (b) 5, 3, 4, 1, 2
(a) 1, 3, 4, 5 (b) 3, 2, 4, 5, 1 (b) 3, 2, 5, 4, 1 (a) 2, 3, 5, 4, 1
(c) 3, 1, 4, 5 (d) 5, 4, 3, 2, 1 10. (a) Bus (b) Bullock cart (c) Airplane (d)
3. (a) conference (b) registration Horse (e) Rocket
(c) participation (d) invitation (e) delegate (a) 4, 2, 1, 3, 5 (b) 3, 4, 1, 2, 5
(a) a, b, d, e, c (b) a, d, e, b, c (b) 2, 4, 1, 3, 5 (d) 1, 4, 2, 3, 5
(c) d, a, e, b, c (d) d, e, a, c, b 11. Put the following words in a meaningful
4. (1) doctor order
(2) fever 1. Ocean 2. River 3. Sea 4. Glacier 5. River
(3) writing prescription (a) 5, 2, 3, 1, 4 (b) 4, 2, 5, 3, 1
(4) identification of disease (diagnosis) (c) 5, 2, 3, 4, 1 (d) 4, 2, 1, 3, 5
(5) medicine Direction: In each of the following
(a) 2, 1, 3, 4, 5 (b) 1, 4, 3, 2, 5 questions, which one of the given options
(c) 2, 1, 4, 3, 5 (d) 2, 4, 3, 5, 1 will represent a meaningful ascending
order of the following words?
5. (1) Never (2) Sometimes (3) Usually (4)
Occasionally (5) Always 12. 1. Grandfather 2. Brother
(a) 5, 2, 1, 3, 4 (b) 5, 3, 2, 1, 4 3. Father 4. Son
(b) 5, 3, 2, 4, 1 (d) 5, 2, 4, 3, 1 5. Grandson
6. (a) book (b) word (c) letter (d) sentence (a) 4, 5, 2, 1, 3 (b) 5, 1, 3, 2, 4
(e) chapter (f) page (c) 5, 4, 1, 3, 2 (d) 5, 4, 2, 3, 1
(a) c b f e d a (b) c b e d f a 13. 1. Nation 2. Village
(c) c b d e f a (d) c b d f e a 3. Taluk 4. District
7. (1) Rain (2) Monsoon (3) Rescue (4) Flood 5. State
(5) Shelter (6) Relief
(a) 2, 3, 4, 1, 5 (b) 1, 3, 5, 2, 4

113
(c) 2, 3, 4, 5, 1 (d) 1, 2, 3, 4, 5 (c) 2, 4, 3, 1, 5 (d) 1, 3, 5, 4, 2
14. 1. Pupa 2. Egg 20. Arrange the following in ascending order:
3. Butterfly 4. Larva 1. Centimeter 2. Kilometer
(a) 1, 3, 2, 4 (b) 3a 1a 4a 2 3. Decimeter 4. Meter
(c) 2, 4, 1, 3 (d) 4, 3, 2, 1 (a) 1, 3, 4, 2 (b) 2, 4, 3, 1
15. A. House B. Road (c) 3, 1, 2, 4 (d) 4, 2, 1, 3
C. Room D. City 21. Combine in a logical sequence
E. District 1. Study 2. Exam
(a) C, B, A, D, E (b) C, A, D, B, E 3. Employment 4. Income
(c) C, A, B, D, E (d) C, A, B, E, D 5. Application
16. 1. Point 2. Triangle (a) 1, 3, 2, 5, 4 (b) 1, 2, 3, 4, 5
3. Square 4. Cone (c) 1, 3, 5, 2, 4 (d) 1, 2, 5, 3, 4
5. Line 22. Which one of the following options
(a) 4, 1, 5, 2, 3 (b) 3, 2, 1, 5, 4 represents a meaningful order of the words
given below?
(c) 2, 1, 4, 5, 3 (d) 1, 5, 4, 2, 3
1. Absorption 2. Digestion
17. Which of the following is the sequence of
colors in a rainbow? 3. Nutrition 4. Excretion
1. Red 2. Green (a) 3, 1, 2, 4 (b) 2, 1, 3, 4
3. Violet 4. Yellow (b) 3, 4, 2, 1 (d) 3, 2, 1, 4
5. Orange TYPE - II
(a) 3, 2, 4, 5, 1 (b) 4, 3, 2, 5, 1
(c) 3, 4, 2, 1, 5 (d) 2, 5, 4, 1, 3 Direction: Arrange the following words
according to the dictionary.
18. 1. Curd 2. Grass
1. Arrange the following words according to
3. Butter 4. Milk
the lexical arrangement:
5. Cow
1. Wound, 2. Writer,
(a) 5, 2, 3, 4, 1 (b) 4, 2, 5, 3, 1 3. Whiter 4. Worst
(c) 2, 5, 4, 3, 1 (d) 5, 2, 4, 1, 3 5. Worked
19. Which one of the following options (a) 5, 3, 2, 1, 4 (b) 1, 4, 3, 5, 2
represents a meaningful order of the words
(c) 3, 5, 4, 1, 2 (d) 2, 1, 3, 4, 5
given below?
2. 1. Sound 2. Socks
1. Earth 2. Jupiter
3. Shock 4. Sharp
3. Venus 4. Mars
4. Snooker
5. Mercury
(a) 5, 4, 3, 1, 2 (b) 4, 3, 5, 2, 1
(a) 1, 3, 4, 5, 2 (b) 5, 3, 4, 1, 2
(c) 3, 4, 5, 1, 2 (d) 4, 3, 2, 5, 1
114
3. 1. Grasp 2. Granite according to the English dictionary.
3. Grass 4. Graph 1. Apple 2. Appreciate
5. Grape 3. Apply 4. Application
(a) E, A, B, C, D (b) E, A, C, B, D (a) 1, 2, 4, 3 (b) 3, 1, 2, 4
(c) B, E, D, A, C (d) B, E, A, D, C
(c) 1, 3, 4, 2 (d) 1, 4, 3, 2
4. 1. Intricate 2. Interview
11. A. PREMONITION
3. Intransigent 4. Interrogation
B. PRELUDE
5. Intravenous
C. PREMICE
(a) 2, 4, 5, 3, 1 (b) 5, 3, 1, 2, 4
D. PRELIMINARY
(c) 4, 2, 3, 5, 1 (d) 3, 5, 2, 1, 4
E. PREMIUM
5. 1. preposition 2. Preparatively
(a) DBAEC (b) BDCEA
3. Preposterous 4. Preponderate
(c) DBCEA (d) BDACE
5. Prepossess
12. Who will be the fourth in the order of the
(a) 2, 4, 1, 5, 3 (b) 1, 5, 2, 4, 3 dictionary?
(c) 5, 4, 2, 3, 1 (d) 4, 2 ,5, 1, 3 (a) Minority (b) Mistake
6. Arrange the given words in alphabetical (c) Mission (d) Miracle
order and find out the last word.
13. Which of the following words will appear
(a) Frankenstein (b) Frankincense
first in the dictionary?
(c) Frankalmoign (d) Frauendienst
(a) Improve (b) Impress
7. Which of the following word will come at
(c) Imprint (d) Impugn
the fourth position in the dictionary?
14. Arrange the following words in the order
Minister, Mineralogy, Minnow, Miniature,
given in the English dictionary:
Mink
1. Caricature 2. Cardinal
(a) Mink (b) Miniature
3. Carnivore 4. Category
(c) Minnow (d) Mineralogy
(a) 2, 1, 3, 4, 5 (b) 4, 5, 1, 3, 2
8. Which word will come third in the
dictionary order? (c) 1, 2, 3, 4, 5 (d) 2, 1, 3, 5, 4
(a) Sentimentalize (b) Sententius 15. Write the following words according to the
(c) Sentimentally (d) Sentinel order given in the dictionary.
1. Euphrasy 2. Eupepsy
9. Which word will come third in the
dictionary order? 3. Euphonics 4. Eugenics
Rumbustious, Rumanian, Rumour, Ruminate, 5. Euphony
Rumple (a) 4, 3, 2, 1, 5
(a) Rumbustious (b) Ruminate (b) 3, 4, 1, 2, 5
(c) Rumour (d) Rumple (c) 4, 2, 3, 5, 1
10. Arrange the following words in sequence (d) 3, 5, 2, 4, 1

115
16. How many different 5-letter words (with or 20. Which of the following represents the
without meaning) can be constructed using all sequence in which the following words occur
the letters of the word 'DELHI' so that each in English dictionary?
word has to start with D and end with I? (UKPSC-2022)
(UPSC-2020) 1. Telegraph
(a) 24 2. Telephone
(b) 18 3. Teleprinter
(c) 12 4. Telemark
(d) 6 5. Telepathy
17. Consider the following sequence of numbers. (a) 1, 2, 3, 4, 5
514739857263 15 8 6 3 85 22 43496 How (b) 5, 4, 3, 2, 1
many odd numbers are followed by the odd
(c) 1, 4, 5, 2, 3
number in the above sequence?
(d) 2, 3, 4, 5, 1
(UPSC-2020)
(a) 5
(b) 6
(c) 7
Answer Sheet
(d) 8
18. In English alphabet, the first 4 letters are TYPE - I
written in opposite order and the next 4 letters 1. (b) 2. (c) 3. (a) 4. (c) 5. (c)
are written in opposite order and so on and at 6. (d) 7. (c) 8. (b) 9. (c) 10. (c)
the end Y and Z are interchanged. Which will 11. (b) 12. (d) 13. (c) 14. (c) 15. (c)
be the fourth letter to the right of the 13th letter 16. (d) 17. (a) 18. (d) 19. (b) 20. (a)
from the left? 21. (d) 22. (d)

(UPSC-2021) TYPE - II
(a) N
(b) T 1. (c) 2. (b) 3. (c) 4. (c) 5. (a)
6. (d) 7. (a) 8. (d) 9. (b) 10. (d)
(c) H 11. (c) 12. (b) 13. (b) 14. (a) 15. (c)
(d) I 16. (d) 17. (b) 18. (b) 19. (b) 20. (c)
19. By arranging the following words according
to English dictionary, find the third word.
Queasy, Queen, Quality, Question, Quarter
(UPPSC-2021)
(a) Quality (b) Queasy
(c) Quarter (d) Queen

116
Explanation 

TYPE - I 3. Usually

1. (d) (b) flower  (c) fly  (a) honey  


(d) Wax 2. Sometimes
2. (c) (3) Seed  (1) Plant  (4) leaf  
(5) Flower (2) Food 4. Occasionally
3. (a) The meaningful order of the words 
according to the options given in the 1. Naver
question will be as follows:
6. (d) Meaningful order of words:
(a) Conference  (b) Registration  (c) letter
(d) Invitation  (e) delegation  (c)
Participation 
(b) word
Because first the decision to organize the
conference is taken, then for the 
arrangement of its organization, (d) sentence
registration of the participants is done.

After this invitation (letter) is sent to the
registered individuals or organizations, (f) page
then the invited individuals or 
organizations form or determine their
(e) chapter
representative or delegation and
ultimately participate in the conference. 
4. (c) Meaningful order of words: (a) book

2. Fever 7. (c) Meaningful order of words:


2. Monsoon

1. Doctor 
1. Rain

4. Identification of disease (diagnosis) 
4. Flood

3. Writing prescription 
3. Relief

5. medicine 
5. (c) Meaningful order of words: 5. Shelter
Always 

117
6. Rescue 1. Grandfather
8. (b) Meaningful order of words: 13. (c) Meaningful order of words:
1. accident 2. Village
 
3. doctor
3. Taluka

5. police 
4. District

4. lawyer 
 5. State
2. judge

9. (c) Water evaporates, then clouds are
1. Nation
formed, condensation takes place then
it rains. 14. (c) Meaningful order of words:
10. (c) meaningful order 2. Egg
(3) Bullock cart  (4) horse  (1) bus 
 (3) Airplane  (5) Rocket
4. Larva
11. (b) Meaningful order of words:
4. Glacier 

 1. Pupa
2. River 
 3. Butterfly
5. River 15. (c) Meaningful order of words:
 C. Room
3. Sea


A. House
1. Ocean
12. (d) Meaningful order of words: 
5. Grandson B. Road
 
4. Son
D. City


2. Brother
E. District

3. Father 16. (d) Meaningful order of words:

 1. Bindu

118
 5. Mercury
5. Rekha 
 3. Venus
4. Kon 
 4. Earth
2. Tribhuj

 1. Mars
3. varg

17. (a) The order of colors in a rainbow is as
follows: 2. Jupiter
violet, indigo, blue, green, yellow, 20. (a) Ascending order :
orange, and red. 1. Centimeter
3. violet

 3. Decimeter
2. green


4. Meter
4. yellow


2. Kilometer
5. orange
21. (d) Meaningful order of words:

1. Study
1. red
18. (d) Meaningful order of words: 

5. Cow 2. Exam

 
2. Grass 5. Application
 
4. Milk 3. Employment
 
1. Curd 4. Income
 22. (d) Meaningful order of words:
3. Butter
3. Nutrition
19. (b) Meaningful order of words:
119
 B. Granite

2. Digestion 
E. Grape


1. Absorption D. Graph
 
A. Grasp
4. Excretion
4. (c) Sort by dictionary:
4. Interrogation
TYPE - II

1. (c) All the words will be arranged like 2. Interview
this: 
1. Whiter 3. Intransigent


5. Intravenous
5. Worded

 1. Intricate
1. Worst 5. (a) Order of words according to
dictionary:
 2. Preparatively
2. Writer 
2. (b) The order of the given words 4. Preponderate
according to the dictionary will be as 
follows: 1. Prepostion

4. Sharp 
5. Preposition


3. Shock 3. Preposterous
 6. (d) Order of words according to
dictionary:
5. Snooker
3. Frankalmoign
 
2. Socks 1. Frankenstein

 
2. Frankincense
1. Sound

3. (c) The order of the given words
4. Frauendienst
according to the dictionary:

120
7. (a) The dictionary order of the words is 3. Apply
as follows :

Mineralogy, Miniature, Minis-ter, Mink, 2. Appreciate
Minnow
11. (c) Order of words according to English
Hence the desired word would be dictionary:
Mink.
(D) PRELIMINARY
8. (d) Order of words according to

dictionary:
(B) PRELUDE
2. Sentimentalize


(C) PREMICE
3. Sentimentally


(E) PREMIUM
4. Sentinel

9. (b) Order of words according to
dictionary: (A) PREMONITION

Rumbustious 12. (b) Order of words according to


dictionary:

1. Minority
Ruminate


4. Miracle
Ruminate


3. Mission
Rumour


2. Mistake
Rumple
13. (b) Order of words according to
10. (d) Order of words according to English dictionary:
dictionary:
1. Impress
5. Apartment

 3. Imprint
1. Apple

 1. Improve
4. Application

 4. Impugh

121
14. (a) Arrangement of words according to
the order given in the English
dictionary:
2. Cardinal


1. Caricature


3. Carnivore


4. Cartoon


5. Category
15. (c) Order of words according to
dictionary:
4. Eugenics


2. Eupepsy


3. Euphony


5. Euphonics


1. Euphrasy

122
16
1. In an imaginary mathematical operation, '+' (c) 2 (d)
stands for multiplication, '  ' stands for
6. If '+' stands for subtraction, '  ' for addition,
subtraction, '  ' stands for addition and '-' stands
for division. All other rules in this mathematical '-' for multiplication and '  ' for division,
operation are the same as in the current system. then which of the following equations is
correct?
Which of the following is the answer for the
fraction? (a) 46 – 10 + 10  5 = 92
175-25  5 + 20  3 + 10 (b) 265 + 11 – 2  14 = 22
(a) 160 (b) 2370 (c) 66  3 – 11 + 12 = 230
(c) 77 (d) 240 (d) 2 – 14  4  11 = 16
2. If '+' means multiplication, '  ' means division, 7. If means  , means -,  means +,+ means  ,
'-' means addition and '  ' means subtraction,
then find the value of the following equation:
then what will be the answer of the following
equation? (16  5)  5 + 3 = ?
 4 3 + 2 = ? (a) 62 (b) 10
(a) 41 (b) 19 (c) 2 (d)
(c) 16 (d) 8. If '+' stands for division, '  ' for addition, '-
3. If '+' means to divide, '  ' means to multiply,, ' for multiplication, '  ' for subtraction, then
'  ' means to subtract, '-' means to add, then which of the following is correct?
which of the following equations is correct?
(1) 15  5  2 – 6 + 3 = 28
(a) 18  6 – 7 + 5  2 = 20
(2) 15  5 + 2 – 6  3 = 56.5
(b) 18 + 6  7  5 – 2 = 18
(3) 15 + 5 – 2  6  3 = 3
(c) 18  6 + 7  5 – 2 = 16
(4) 15 – 5 + 2  6  3 = 41
(d) 18  6  7 + 5 – 2 = 22
4. If '+' means multiplication, '-' means divide, (a) c (b) a
'  ' means subtract and '  ' means add, then (c) b (d) d
9 + 8 + 8 4 – 9 = ? 9. A is for 'add', B is for 'subtract', C is for 'divide', D
(a) 26 (b) 17 is for 'multiply', E is for 'less than', F is for 'greater
(c) 65 (d) than' and G for 'equal'.

5. If '+' means '  ', '  ' means '-', '-' means '  ', '  ' (a) 18 C 2 A 4 B 6 G 9
means '+' then (b) 6 D 4 B 12 A 4 C 2 F 18
12 + 6  3 – 2  8 = ? (c) 10 C 2 D 4 B 6 E 12
(a) –2 (b) 4 (d) 9 A 7 B 4 C 2 G 14

123
10. If 'a' means '  ', 'b' means '+', 'c' means '-' and 17. Some equations are solved on the basis of a
'd' means '  ', the 24a6d4b9c8 = ? particular method. on the same basis find out
the correct answer of the same equation which
(a) 2 (b) 17
is not solved.
(c) 34 (d) 19
2  3  4 = 432,
11. If A stands for '  ', 'D' stands for '+', and 'G'
5  6  7 = 765
stands for '-', then find the value of
7A4D4A3G2. 7  8  9 = 987

(a) 28 (b) 38 2 5 7 = ?

(c) 44 (d) 48 (a) 572 (b) 752


(c) 725 (d) 257
12. If A is for +, Q is for '-', V is for '  ', R is for
'  ', then what will be the value of the given 18. if 2  16 = 8; 8  8 = 1; 6  12 = 2, 12  144
equation? =?

225 R 5 A 64 Q 13 V 6 = ? (a) 11 (b) 12

(a) 376 (b) 31 (c) 16 (d) 24

(c) 476 (d) 576 19. if 16 – 2 = 2, 9 – 3 = 0, 81 – 1 = 8, so 64 – 4 =


What ?
13. If 'P' stands for multiplication, 'T' for
(a) 4 (b) 2
subtraction, 'M' for addition and 'B' for division,
then what will be the correct value of 12 P 6 M (c) 6 (d) 8
15 T 16 B 4? 20. Choose the appropriate combination of
(a) 70 (b) 75 symbols to solve.

(c) 83 (d) 110 16*8*1*8

14. If L stands for +, M stands for -, N stands for (a) =–  (b) –  =


 and P stands for  , then 14 N 10 L 42 P 2 M (c)  –= (d)  =–
8=? 21. Give the correct set of symbols to solve the
(a) 153 (b) 216 equation.

(c) 248 (d) 251 24 * 16 * 8 * 32

15. 2  4  6 = 4; 9  3  7 = 13: (a) +–= (b)  – =

4  7  6 = 3; 9  7  8 = ? (c) –+= (d)   =


(a) 10 (b) 09 22. * Choose the correct combination of arithmetic
signs to interchange the signs and balance the
(c) 08 (d) 07 given equation.
16. 3  5  7  2 = 24; 2  4  6  8 = 22, 4  4  15 * 24 * 3 * 6 * 17
8  9=?
(a) – += (b) +  – =
(a) 33 (b) 25
(c) +  =  (d) –  = +
(c) 144 (d) 1152

124
23. Choose the correct ordered set of mathematical
symbols to balance the following equation and Answer Sheet
replace the * symbols with:
1. (c) 2. (c) 3. (b) 4. (c) 5. (b)
6 * 4 * 12 * 12
6. (c) 7. (d) 8. (a) 9. (d) 10. (b)
(a)  – = (b) + – 
11. (b) 12. (b) 13. (c) 14. (a) 15. (a)
(c) =–  (d)  – =
16. (a) 17. (b) 18. (b) 19. (a) 20. (b)
24. * Choose the correct symbol of the arithmetic 21. (a) 22. (b) 23. (d) 24. (c) 25. (b)
symbol to interchange the symbols and balance
the given equation. 26. (d) 27. (a) ********************

2*4*3*4*9
(a) +  =– (b)   – =
(c) –+= (d) + – = 
25.  +1  +5  +   +  1=1  
For which digit does the sysmbol  stand?
(UPSC-2020)
(a) 2 (b) 3
(c) 4 (d) 5
26. If $ means ‘divided by,’@ means ‘multiplied
by,’# means ‘minus’, then the value of 10 # 5
@ 1 $ 5 is
(UPSC-2019)
(a) 0
(b) 1
(c) 2
(d) 9

27. If 7  9  10 = 8, 9  111  30 = 5, 11 1
17  21 = 13, What is the value of 23  4  15?
(UPSC-2023)
(a) 6
(b) 8
(c) 13
(d) 15

125
Explanation Hence, the correct answer would be
option (c).
1. (c) 175 – 25  5+ 20  3 + 10 5. (b) 12 + 6  3 – 2  8
 175  25 + 5  20 – 3  10 = 12  6 – 3  2 + 8
= 7 + 100 – 30 = 77
=2–6+8= 4
2. (c) 20 – 8  4  3 + 2
6. (c) Option (1)
 20 + 8  4 – 3  2
46 – 10 + 10  5
= 22 – 6 = 16
 46  10 – 10  5
3. (b)   = 460 – 2 = 458  92 (It is wrong)
 
Option (2)
Option 265 + 11 – 2  14
18  6 –7 + 5  2 = 20  265 – 11  2  14
 18  6 + 7  5 – 2 = 20 22
= 265 –  22 (It is wrong)
7 14
 18  6 + – 2  20
5 Option (3)
Option 66  3 – 11 + 12
18 + 6  7  5 – 2 = 18  66  3  11 – 12
 18  6  7 – 5 + 2 = 18 = 22  11 – 12 = 242 – 12
 21 – 5 + 2 = 18 = 230 = 230 (true)
Option 7. (d) ? = (16 – 5) + 5  3
18  6 + 7  5 – 2 = 16  ? = 11 + 15 = 26
 18 – 6  7  5 + 2 = 16
8. (a)  
6  
 18 –  5 + 2  16
7
(1) 15  5  2 – 6 + 3 = 28
Option
 15 – 5 + 2  6  3 = 28
18  6  7 + 5 – 2 = 22
 15 – 5 + 2  2 = 28
 18  6 – 7  5 + 2 = 22
 15 – 5 + 4  28
7
 18  6 – + 2  22 (2) 15  5 + 2 – 6  3 = 56.5
5
 15 + 5  2  6 – 3 = 56.5
4. (c) 9+8  8–4  9
 15 + 2.5  6 – 3 = 56.5
=9  8+8  4–9
 15 + 5 – 3  56.5
=9  8+2–9
= 72 + 2 – 9 = 65 (c) 15 + 5 – 2  6  3 = 3
 15  5  2 – 6 + 3 = 3

126
 3  2–6+3=3 or, 4  4 + 3 – 8
 6–6+3=3 or, 16 + 9 – 8 or, 25 – 8 = 14
(d) 15 – 5 + 2  6  3 = 41 11. (b) 7A4 D 4A3 G 2
 15  5  2 + 6 – 3 = 41  7 4+4  3–2
 15  2.5 + 6 – 3 = 41  28 + 12 – 2 = 28 + 10 = 38
 37.5 + 6 – 3  41 12. (b) A Q
9. (d) According to Question A the meaning V R
of
B the meaning of 250 R 5A 64 Q 13V6 = ?
C the meaning of   ? = 225  5 + 64 – 13  6
D the meaning of   ? = 45 + 64 – 78 = 31
E the meaning of <
13. (c) P T
F the meaning of >
M B 
and G the meaning of =
Therefore, 12 P 6 M 1 5 T 16 B 4 = ?
18 C 2 A 4 B 6 G 9  ? = 12  6 + 15 – 16  4
 18  2 + 4 – 6 = 9  ? = 72 + 15 – 4 = 83
 6+4–6=9
14. (c) L >
6 D 4 B 12 A 4 C 2 F 18
N P
 6  4 – 12 + 4  2 > 18
 24 – 12 + 2 > 18 14 N 10 L 42 P 2 M 8 = ?
 14 > 18  ? = 14  10 + 42  2 – 8
The relation is not correct.  ? = 14  10 + 21 – 8
10 C 2 D 4 B 6 E 12  ? = 140 + 21 – 8 = 153
 10  2  4 – 6 < 12
15. (a) 2 4 6=4
 5  4 – 6 < 12
 6 – 4 = 2; 2 + 2 = 4
 14 < 12 3rd number - 2nd number + 1st number
The relation is not correct. = result
9 A 7 B 4 C 2 G 14 9  3  7 = 13
 9 + 7– 4  2 = 14  7 – 3 + 9 = 16 – 3 = 13
 16 – 2 = 14 4  7  6=3
 14 = 14 6 – 7 + 4 = 3
10. (b) a   , b  +, c  –, d    10 – 7 = 3
Therefore 24  6  4 + 9 – 8 = 17 9  7  8

127
8 – 7 + 9 144
? = = 12
 17 – 7 = 10 12
16. (a) 3  5  7  2 = 24 19. (a) 16 – 2 = 2
 3  5 + 7 + 2 = 24  16 = (2 + 2)2 = (4)2
 15 + 9 = 24 9–3=0
2  4  6  8 = 22  9 = (3 + 0)2 = (3)2
 2  4 + 6 + 8 = 22 81 – 1 = 8
8 + 14 = 22  81 = (1 + 8)2 = (9)2
4 4  8 9 = ? Similarly,
? = 4  4 + 8 + 9 64 – 4 = ?
 ? = 16 + 17 = 33  64 = (4 + 4)2 = (8)2

2  3 4 4 3 2 20. (b) 16 – 8  1 = 8


17. (b)
 16 – 8 = 8
21. (a) 24 * 16 * 8 * 32
5  6 7 7 6 5  24 + 16 – 8 = 32
 40 – 8 = 32
22. (b) 15 * 24 * 3 * 6 * 17
7  8 9 9 8 7  16 + 24  3 – 6 = 17
 15 + 8 – 6 = 17
23. (d) 6 * 4 * 12 * 12
2  5 7 7 5 2  6  4 – 12 = 12
 24 – 12 = 12
24. (c) 2*4*3*4*9
18. (b) 2  16 = 8 2  4 – 3 + 4 = 9
16 8 – 3 + 4 = 9
 =8
2 5 + 4 = 9
8 8=1
8
 =1
8
6  12 = 2
12
 =2
6
Therefore,
12  144 = ?

128
17
TYPE - I (c) CAUTION (d) MOTION

A word is given capital letters. It is followed 8. HALLUCINATION


by four. words. Out of these four words, three (a) LION (b) LOAN
cannot be formed from the letters of the word
in capital letters. Point out the word which (c) NATION (d) LOTION
cannot be formed from the letters of the given 9. SPECIFICATION
word in capital letter.
(a) FAINTING (b) TONIC
1. PHOTOSYNTHETIC
(c) PACIFIC (d) FACTION
(a) THOSE (b) SCENT
10. MANUSCRIPT
(c) PRONE (d) COTTON
(a) MASTER (b) PRIMUS
2. AUTOGRAPHS
(c) SMART (d) RUSTIC
(a) GRAPH (b) TROUGH
TYPE - II
(c) PATHOS (d) GREAT
1. A word is given capital letters. It is followed
3. ADMINSTRATION by four. words. Out of these four words, three
cannot be formed from the letters of the word
(a) Mind (b) Ration in capital letters. Point out the word which can
(c) Minister (d) Station be formed from the letters of the given word in
capital letter.
4. PERMANENT
NEWSPAPER
(a) REMNANT (b) TRAMP (a) SWEET (b) REPEAT
(c) MENTOR (d) AMPERE (c) SOUR (d) WASP
5. BEAUTIFUL 2. ENVIRONMENT

(a) LIFT (b) FULL (a) EMINENT (b) ENTRANCE


(c) ENTERTAIN (d) MOVEMENT
(c) BEAT (d) FUTILE
3. ARISTOCRATIC
6. GOVERNMENT
(a) CREATION (b) STATIC
(a) ENTER (b) MERGE
(c) SECRET (d) ARREST
(c) VETERAN (d) GREET 4. SOMNAMBULISM
7. COMBINATION (a) NAMES (b) BASAL
(a) NATION (b) AMBITION (c) SOUL (d) BIOME

129
5. MUSPOPAPOTIH 8. MANUFACTURE
(a) METAMORPHIC (a) FRACTURE (b) MANNER
(b) PHILANTHROPIST (c) MATTER (d) FACE
(c) HIPPOCAMPUS 9. If every alternative letter of English alphabet
(d) HIPPOPOTAMUS from B onwards (including B) is written in
lower case (small letters) and the remaining
6. MULTIPLICATION letters are capitalised, then how is the first
(a) MUTUAL month of the second half of the year written?
(b) LIMITATION (UPSC-2019)
(c) APPLICATION (a) JuLY (b) JULY
(d) NOTION (c) july (d) jUIY
7. STRANGULATION 10. Which one of the following word can not be
(a) TRIANGLE formed using the letters of the word
PROFESSIONAL'?
(b) GARLAND
(UPPSC-2021)
(c) ROASTING
(a) PASSION (b) FINAL
(d) TRAUMA
(c) NORMALD (d) LESSION

Answer Sheet

TYPE - I
1. (c) 2. (d) 3. (c) 4. (c) 5. (b)
6. (c) 7. (c) 8. (d) 9. (a) 10. (a)
TYPE - II
1. (d) 2. (a) 3. (b) 4. (c) 5. (d)
6. (c) 7. (c) 8. (d) 9. (d) 10. (c)

130
Explanation words. But there is no letter ‘E’ in the
keyword.
TYPE - I
4. (c) SO M N A M B U L I S M
1. (c) The letter ‘R’ is not present in the word
PHOTOSYNTHETIC. 5. (d) Meaningful word 
2. (d) There is no letter ‘E’ in the keyword. HIPPOPOTAMUS
Therefore, the word GREAT cannot be 6. (c) There is only one ‘U’ in the keyword.
formed.
There are one ‘A’ and one ‘P’ in the
3. (c) There is no ‘E’ letter in the keyword. keyword.
Therefore, the word Minister cannot be
There is only one ‘O’ in the keyword.
formed.
4. (d) There is no ‘O’ letter in the keyword. 7. (c) S T R A N G U L A
5. (b) There is only one ‘L’ in the given word T I O N
but in the word FULL there are two
Ls. M A N U F A C T U R E
8. (d)
6. (c) There is no letter ‘A’ in the key word.
7. (c) There is no ‘U’ letter in the given word.
8. (d) There is only one ‘O’ in the given word.
9. (a) There is no ‘G’ letter in the given word.
Therefore, the word FAINTING cannot
be formed.
10. (a) There is no ‘E’ letter in the given word.
Therefore, the word MASTER cannot
be formed.

TYPE - II

1. (d) The letter ‘T’ is not present in the


keyword and hence the words SWEET
and REPEAT cannot be formed from
the letters of the word NEWSPAPER.
Similarly, the letters ‘O’ and ‘U’ are not
present in the keyword and hence the
word SOUR cannot be formed.

1 4 3 2 5 6 7
2. (a) E N V I RO N M E N T

3. (b) Except in the word STATIC, there is


letter is letter ‘E’ in all the other three

131
NON-VERBAL
REASONING

132
18
1. Two Positions of a dice is given. Which (a) (b)
number will be on top face, when 2 is in the
(c) (d)
bottom?
5. Three figures of a common cube is given.
(SSC -2013) Number 1 to 6 has been marked on all faces
3 of the cube. Select a figure that will represent
3
the open cube?
2 5 6 1
(SSC-2013)
Question Figure:
(a) 4 (b) 1

(c) 5 (d) 6 3 3 4

2. From the following two different figures of a


2 5 1 5 3 2 ?
dice, find the colour that is opposite of white?
(SSC -2012) Answer Figure:

Yellow Yellow
d
e

Re
Blu

White Red

(a) Blue (b) Black

(c) Yellow (d) Red


(a) (b) (c) (d)
3. What is the number on the face opposite 5? 6. Two positions of a dice is presented.
(SSC -2012) (SSC-2008)

5 5 5 5
3 1 4 2
1 3 and 2 4

(a) (b)
If 5 is on the top, then, What number will be
(c) (d) in bottom?
4. What is the number on the face opposite to 4? (a) (b)
(SSC - 2012) (c) (d)
7. Two position of a dice is presented below. If
5 5
1 is in bottom then what will be at top?
2 3 3 4
(CPF-2012)

133
(SSC-1999)

-
  + +
(a) 3 (b) 5
(a)  (b) +
(c) 6 (d) Cannot say
(c) O (d)
8. Some Position of a dice is given below then
12. Shaded portion of the figure is the base of a
what number is opposite of 4
cube you are facing face with digit
(CPF-2008) 1. What is the digit on the face opposite to it.
5 3 (SSC-2006)
4 2
1 4 3 3
3 5 2 1
1
(a) 6 (b) 5
2 3
(c) 2 (d) 1 4
9. Consider the following figures of a dice: 5
(UPSC-2000)
(a) (b)
6 6 5 1 (c) (d)
3 2 2 6 4 2
4 4 13. Two Position of a cubical block is given below.
small triangles are designed on every face. If
in a different position of the cube, if one
What number is opposite of 3?
triangle is in bottom , then how many triangles
(a) will be on top face?
(b) (SSC-2010)
(c)
(d) Data Inadequate
10. If a dice is formed using the given positions,
what colour will be oppsite to green?
(SSC-2005)
(a) (b)
Green Orange Silver
(c) (d)
e d le Red
ng Re P ur p Green Red
ra
O 14.
Five
(a) Orange (b) Red One Two
(c) Silver (d) Purple
Four Three
11. Four position of a cube is shown below. What Six
sign will be on-the face opposite to  ?

134
If a cube is made from the above given
situation, the what word will be on the face
opposite to 'one' ? (SSC-2006)

(a) Three (b) Six (a) (b) (c) (d)


(c) Four (d) Five 18. Two Position of a dice is given below:
(SSC-2004)
15. Cousider the faces of the dice and answer the
question that follow:
S.S.C -2006

Red Blue Pink Yellow


k

ow

Blue Red Red


When 2 is at bottom what number is on top?
k

B l ac k
Pi n

en
Pin

ll

re
Ye

(a) (b)
Which colour is on opposite face to Red? (c) (d)
19. Consider the four different position of a cube,
(a) Yellow (b) Pink
which faces are numbered 1 to 6. Find the
(c) Green (d) Black number opposite to the face with digit 3?
(SSC-2000)
16. Four postions of a dice is given below, which
sign is opposite to +

(SSC-1999)

- (a) (b)
  + + (c) (d)
20. Three positions of a dice is presented. Then
(a)  (b)  explain the other side of the three dotted face?

(c) – (a)  (SSC-2008)

17. Which cube among the four cubes can be Question Figure:
coustructed from the design given below?
(SSC-1999)
Given Design

(a)

(b)

(c)
Answer Figure:
(d)
135
21. How many point are on face opposite to three
point face?
(SSC-2006)

(a) (b)
(c) (d)
25. Two positions to a dice is given below:
(a) Two (b) Four (SSC-2008)
(c) Five (d) Six
4 2
22. Two Position of a dice is shown below: 3 3
6 5
(SSC-2001)
When '2' is at base, what will be at top?
(a) (b)
(c) (d)
26. Four positions of a dice is given below.
Recognize the number at bottom when 5 is
When 'heart' figure will be at top, then figure
on top?
will be at bottom?
(SSC-2012)
Answer Figure:
4 3 1 2
3 2 1 4 3 5
5 6

(a) (b)
(a) (b) (c) (d) (c) (d)
23. Three positions of a dice is given. on the basis 27. Four figures of a dice is given, what number
of this, find the digit opposite to digit 2? is opposite to the face of 6?
(SSC-2011) (SSC-2012)

6 5 4 6 6 5 1
1 4 3 6 1 2 3 2 2 4 6 4 4 2

(a) 1 (b) 2
(a) (b)
(c) 3 (d) 4
(c) (d)
28. Which cube can be coustructed by using the
24. Two position of a dice is given below, when 3 given figure?
is at bottom, then what number will be on top?
(SSC-2013)
(SSC-2001)

136
Question Figure: (a) (b)
(c) (d)
F 31. Six coloured (both back and front) square
figures, red (R), Blue (B), Yellow (Y), Green
A E (G), White (W) and orauge (O) are attached
B to one another. If these figures are converted
into a cube, then what colour will opposite to
C D
white?
(UPSC-2012)
Answer Figure:
R B
F F E D
G Y O
B E E D B C F A
W
(a) (b) (c) (d)
(a) R (b) G
29. Four figures of a dice is given below. a special
sign is marked on each face. In figure 3. What (c) B (d) O
sign will appear on the face opposite to
32. Three diferent postions of an special cube is
sign?
given below:
(SSC-2006)
(UPSC-2012)

+ K H B
A B M K P H
Answer Figure:

What letter is opposite to A?


+ (a) H (b) P

(a) (b) (c) (d) (c) B (d) M

30. Consider the different positions of a cube on 33. Digits 1,2,3,4,5 and 6 are marked on the faces
which face dots 1 to 6 has been marked find of a cube. Three position of a cube in shown
the number of dots on the face opposite to below:
three dots ?
(UPSC-2013)
(SSC-2005)
1 3 3
6 4 1 2 5 6

What can be on the two faces of the cube that


are marked A and B respectively?

137
(a) 32 (b) 24
B
(c) 16 (d) 8
A 5 37. The outer surface of a 4 cm  4 cm  4 cm cube
is painted completely in red. It is sliced parallel
to the faces to yield sixty four 1 cm  1
(a) 2 and 3 (b) 6 and 1 cm  1cm small cubes. How many small cubes
do not have painted faces?
(c) 1 and 4 (d) 3 and 1
(a) 8 (b) 16
34. Six faces of a cube are numbered from 1 to 6,
(c) 24 (d) 36
each face carrying one different number.
Further, Directions for the following 38-40 (three)
items:
(UPSC- 2007)
Rotated positions of a single solid are shown
1. The face 2 is opposite to the face 6.
below. The various faces of the solid are
2. The face 1 is opposite to the face 5. marked with different symbols like dots, cross
3. The face 3 is between the face 1 and the and line. Answer the three items that follow
face 5. the given figures.
4. The face 4 is adjacent to the face 2.
Which one of the following is correct?
(a) The face 2 is adjacent to the face 3.
(b) The face 6 is between the face 2 and the
face 4. 38. What is the symbol on the face opposite to
that containing a single dot?
(c) The face 1 is between the face 5 and the
face 6. (a) Four dots (b) Three dots
(d) None of the above. (c) Two dots (d) Cross
35. Each of the six different faces of a cube has 39. What is the symbol on the face opposite to
been coated with a different colour i.e., V, I, that containing two dots?
B, G, Y and O. Following information is given: (a) Single dot (b) Three dots
1. Colours Y, O and B are on adjacent faces. (c) Four dots (d) Line
2. Colours I, G and Y are on adjacent faces. 40. What is the symbol on the face opposite to
3. Colours B, G and Y are on adjacent faces. the face with the cross?
4. Colours O, V and B are on adjacent faces. (a) a single point (b) two points
Which is the colour of the face opposite to (c) line (a) four points
the face coloured with O? 41. A solid cube is painted yellow, blue and black
such that opposite faces are of same colour.
(a) B (b) V
The cube is then cut into 36 cubes of two
(c) G (d) I different sizes such that 32 cubes are small
36. A cube has all its faces painted with different and the other four cubes are big. None of the
colours. It is cut into smaller cubes of equal faces of the bigger cubes is painted blue. How
sizes such that the side of the small cube is many cubes have only one face painted?
one-fourth the big cube. The number of small (UPSC-2018)
cubes with only one of the sides painted is : (a) 4 (b) 6

138
(c) 8 (d) 10
42. The outer surface of a 4 cm x 4 cm x 4 cm
cube is painted completely in red. It is sliced
parallel to the faces to yield sixty four 1 cm x
1 cm x 1 cm small cubes. How many small
cubes do not have painted faces?
(UPSC-2017)
(a) 8 (b) 16
(c) 24 (d) 36

Answer Sheet
1. (d) 2. (a) 3. (c) 4. (a)
5. (d) 6. (a) 7. (c) 8. (d)
9. (b) 10. (d) 11. (c) 12. (c)
13. (b) 14. (a) 15. (d) 16. (b)
17. (b) 18. (c) 19. (b) 20. (c)
21. (c) 22. (d) 23. (d) 24. (d)
25. (c) 26. (c) 27. (a) 28. (b)
29. (d) 30. (b) 31. (c) 32. (a)
33. (a) 34. (a) 35. (c) 36. (b)
37. (a) 38. (b) 39. (c) 40. (c)
41. (c) 42. (a) ***************

139
19
1. A cube is painted red on all the six sides. Then (c) 8 (d) 16
it is cut into 27 equal cubes. How many cubes 8. A solid cube of size four inches has opposite
are colored on one side only? pairs of faces painted red, green and black. It
(a) 0 (b) 6 is cut into one inch cubes. How many cubes
(c) 8 (d) 18 have only red and green faces?
2. A solid cube made up of 27 smaller cubes has (a) 4 (b) 8
two opposite faces painted red, two painted (c) 16 (d) 24
yellow and the other two painted white. How 9. A solid cube of size 9.4 inches has pairs of
many cubes have two colours? opposite faces painted red, green and black. It
(a) 8 (b) 12 is cut into cubes of one inch. How many cubes
(c) 16 (d) 24 have only two faces painted?
3. A solid cube of side three inches is made by (a) 8 (b) 16
placing small wooden cubes of side one inch (c) 24 (d) 32
together. This big cube was completely painted 10. A solid cube of size 4 inches has pairs of op-
red from outside. When the big cube was bro- posite faces painted red, green and black. It is
ken into the original smaller cubes, how many cut into one inch cubes. How many cubes do
cubes would not have any colour? not have any color on any face?
(a) 0 (b) 1 (a) 0 (b) 4
(c) 3 (d) 4 (c) 8 (d) 16
4. If 64 small cubes are made into a solid cube, 11. A solid cube of size 4 inches has pairs of op-
then how many small cubes will have two posite faces painted red, green and black. It is
faces? cut into one inch cubes. How many cubes have
(a) 24 (b) 32 only one face painted?
(c) 40 (d) 42 (a) 4 (b) 8
5. Two opposite faces of a solid cube made up (c) 16 (d) 24
of 27 smaller cubes are painted red, two op- 12. What is the number of cubes in this figure?
posite faces yellow and two other faces white.
How many small cubes are colored only yel-
low and white?
(a) 4 (b) 8
(b) 12 (d) 16
6. A solid cube of size four inches has pairs of
opposite faces red, green and black. It is cut
into one inch cubes. How many cubes have
only four faces painted? (a) 10 (b) 8
(a) 0 (b) 4
(c) 16 (d) 12
(c) 8 (d) 16
7. A solid cube of size 4 inches has pairs of op- 13. All the faces of a red colored solid cube are
posite faces painted red, green and black. It is colored yellow. After this, that cube is cut into
cut into one inch cubes. How many cubes have 125 identical cubes. Accordingly, how many
all three colours? faces are there, whose three faces will be of
(a) 0 (b) 4

140
yellow colour? 17. In the figure given below you can see a picture
in which a group of blocks has been arranged
(a) 10 (b) 4
on the basis of a certain sequence. All the blocks
(c) 8 (d) 12 are of the same size. In a group, you can see
only some segments. All others are hidden
14. What is the number of cubes in the following behind their front wings. Try counting the
figure? segments with the currently invisible segments
in the group?

(a) 4 (b) 8

(c) 9 (d) 6
(a) 69 (b) 180
18. Small wooden cubes each of edge one inch are
(c) 144 (d) 84 put together to form a solid cube of edge three
15. How many cubes are invisible in the figure? inches. This big cube was painted red from all
sides. When the big cube was divided into
original smaller cubes, how many cubes would
have two faces painted?

(a) 4 (b) 8

(c) 12 (d) 0

19. Some cubes are arranged as shown in figure.


(a) 5 (b) 6 How many cubes are not visible?

(c) 10 (d) 15
16. Count the number of cubes in the given figure.

(a) 8 (b) 10

(c) 12 (d) 14
20. A solid cube of size 4 inches has opposite pairs
(a) 5 (b) 6 of faces painted red, green and black. It is cut
into one inch cubes. How many cubes have only
(c) 10 (d) 15 three faces painted?
(a) 4 (b) 8

141
(c) 12 (d) 16 25. A cube of white material is painted black on
21.Count the number of cubes in the given figure. all its surfaces. If it is cut into 125 smaller
cubes of the same size, then how many cubes
will have two sides painted black?
(a) 8 (b) 16
(c) 22 (d) 44
(e) None of these
Directions : A cube is coloured red on all faces.
It is cut into 64 smaller cubes of equal size.
(a) 14 (b) 12 Now, answer the following questions based
(c) 10 (d) 8 on this statment:

22. Count the number of cubes in the given figure. 26. How many cubes have no face coloured?
(a) 24 (b) 16
(c) 8 (d) 0
Directions : One hundred ans twenty-five
cubes of the same size are arranged in the form
of a cube on a table. Then a column of five
cubes is removed from each of the four
corners. All the exposed faces of the rest of
the solid (except the face touching the table)
are coloured red. Now, answer these questions
based on the above statement :
(a) 8 (b) 9 27. How many small cubes are there in the solid
after the removal of the coloumns?
(c) 12 (d) 15
(a) 120 (b) 110
23. A cube whose two adjacent faces are coloured
is cut into 64 identical small cubes. How many (c) 105 (d) 100
of these small cubes are not coloured at all? 28. How many cubes do not have any coloured
face?
(a) 12 (b) 24
(c) 36 (d) 48
29. How many cubes have only one red face each?
(a) 60 (b) 48
(a) 40 (b) 25
(c) 36 (d) 24
(c) 20 (d) 15
24. A cube, painted yellow on all faces is cut into
27 small cubes of equal size. How many small 30. How many cubes have more than two coloured
cubes are painted on one face only? faces each?

(a) 1 (b) 6 (a) 8 (b) 20


(c) 8 (d) 12 (c) 36 (d) 44

142
31. A solid cube of 3 cm side, painted on all its 34. 125 identical cubes are arranged in the form
faces, is cut up into small cubes of 1 cm side. of a cubical block. How many cubes are
How many of the small cubes will have exactly surrounded by other cubes from each side?

two painted faces? (UPSC-2023)

(UPSC-2018) (a) 27 (b) 25

(a) 12 (c) 21 (d) 18

(b) 8 35. A cuboid of dimensions 7 cm x 5 cm x 3 cm


is painted red, green and blue colour on each
(c) 6 of dimensions pair of opposite faces 7 cm x 5
(d) 4 cm, 5 cm x 3 cm, 7 cm x 3 cm respectively.
32. A solid cube is painted yellow, blue and black Then the cuboid is cut and separated into
such that opposite faces are of same colour. various cubes each of side following length
The cube is then cut into 36 cubes of two 1cm. Which of the statements is/are correct?
different sizes such that 32 cubes are small (UPSC-2023)
and the other four cubes are big. None of the 1. There are exactly 15 small cubes with no
faces of the bigger cubes is painted blue. How paint on any face.
many cubes have only one face painted?
2. There are exactly 6 small cubes with
(UPSC-2019) exactly two faces, one painted with blue
(a) 4 and the other with green.
(b) 6 Select the correct answer using the code given
(c) 8 below:

(d) 10 (a) 1 only

33. A cube of edge 6 cm is painted on all faces of (b) 2 only


it. It is then cut into cubes of edge 1 cm. How (c) Both 1 and 2
many cubes have no face painted? (d) Neither 1 nor 2
(UKPSC-2022)
(a) 8
(b) 64
Answer Sheet
(c) 16
1. (b) 2. (b) 3. (b) 4. (a) 5. (a)
(d) 24
6. (a) 7. (c) 8. (c) 9. (c) 10. (c)
34. 125 identical cubes are arranged in the form
11. (d) 12. (d) 13. (c) 14. (c) 15. (a)
of a cubical block. How many cubes are
surrounded by other cubes from each side? 16. (d) 17. (b) 18. (b) 19. (b) 20. (b)
(UPSC-2023) 21. (c) 22. (d) 23. (c) 24. (b) 25. (e)
(a) 27 (b) 25 26. (c) 27. (c) 28. (c) 29. (b) 30. (a)

(c) 21 (d) 18 31. (a) 32. (c) 33. (b) 34. (a) 35. (a)

143
7. (c)
Explanation
1. (b) Number of cubes painted on one side =
6  1= 6
(on each face there will be only one
central cube which will be colored from
one side only)
2. (b) Number of two colored cubes = 4 3 =
12
3. (b) There will be no color on the center cube
The four corners of level 1 and the four
of the middle row.
corners of level 4 have colors on all three
4. (a) Two surfaces will be visible in 24 small
surfaces.
cubes.
8. (b)
5. (a)

The cubes in the middle row do not have Level-1 and Level-4 each have 4 + 4 such
red color and the number of such cubes cubes which have only red and green
is 9. But there is no color on the central faces. Hence the total number of such
cube. Now out of eight remaining cubes, cubes is 16.
four cubes are either white or yellow in
9. (c)
colour.
6. (a)

There are 8 cubes in level 1, 4 in level 2,


All the four faces of any cube will not be 4 in level 3 and 8 in level 4 whose only
two faces are coloured.
painted.
Hence the total number of such cubes
= 8 + 4 + 4 + 8 = 24

144
10. (c) In Level-II, III, and IV PtA each, the 9
cubes in the middle have no surface
coloured, the four cubes at the corners
have two levels coloured, and the
remaining 12 cubes have only one surface
coloured.
In Level-V, the 9 cubes in the center have
only one surface coloured, the 4 cubes at
the corners have three surfaces colored
and the remaining 12 cubes have two
The four central cubes of level-2 and the
surfaces coloured.
four central cubes of level-3 have no
14. (b) Total number of cubes = 6  6  4 = 144
painted surface. Hence the number of
15. (a) There are 15 cubes in the given figure.
such cubes is 8.
Showing 10 cubes. Hence, 5 cubes are
11. (d)
invisible.
16. (d) All the eight cubes have three faces
painted red.
17. (d) There are clearly four anterior and four
posterior segments.
18. (b)

In level 1 and 4 only one face of the four


cubes located in the middle is colored. In
Level 2 and 3, only one face of the eight-
eight cubes located in the middle is
colored. Hence total number of such
cubes = 8 + 16 = 24
12. (d) There are 12 cubes in all.
13. (c)
There will be color on two sides of the
four-four cubes of the first and third level.
Therefore, the number of colored cubes
I
on only two surfaces = 4  2 = 8
II
19. (b) Three cubes each are not visible in the
III
first and second column from the left.
IV
Similarly, two cubes each are not visible
V
in the third and fourth column.
In Level-I, only one surface of the 9 cubes
Total number of cubes not visible = 3 + 3
in the middle is coloured, three of the
+ 2 + 2 = 10
cubes at the corners are coloured, and of
the remaining 12 cubes, 2 are coloured.

145
20. (a)

In Level I and IV, three faces of the four cubes


C3=8 located at the four corners will be colored.
Counting the Number of Cubes/Blocks in
the given Figure When the number of cubes
(or blocks/cuboids) in a figure are to be
counted, the procedure to be adopted is as
described in the following :

146
20
1. Which of the following diagram represent
Mammals, Cows and Crows? (a) (b)

(c) (d)

(a) (b) (c) (d)


6. Which diagram correctly represents the
relationship between politician, poets and
2. Which of the diagrams given below correctly women?
represents the relationship among husband,
wife and family?

(a) (b) (c) (d)

Directions: In each of the following questions,


(a) (b) (c) (d)
identify the diagram that best represents the
relationship among classes given below:
3. Which diagram represents the relationship
among female, mothers and doctors? 7. Government Servants, Lecturers, Doctors

(a) (b)

(a) (b) (c) (d)


(c) (d)
4. Which one of the following figures represents
8. Singers, Boys, Dancers.
the relationship among males, fathers,
advocates? (a) (b)

(a) (b) (c) (d)

Direction: In each of the following questions,


(c) (d) identify the diagram that best represents the
relationship among the classes given below.
5. Choose the correct figure that represents the 9. Teacher, Writer, Musician
given relation: Blue eyed, females, doctors

147
(a) (b) (c) (d) (a) (b) (c) (d)

10. School, Women, Children 15. Which diagram correctly represents the
relatonship between Human beings, Teachers,
Graduates?

(a) (b)
(a) (b) (c) (d)

11. Identify the diagram that best represents the


relationship among classes given below: (c) (d)
Sportsmen, Cricketers, Batsmen
16. Which statement is true with respect to the
Venn diagram?
(a) (b)

4
3 4
2
(c) (d) 8
1 6 7

12. Which of the following figures represents the


9
relationship among DOG, CAT and PET?
14

(a) (b)
(a) 6,7 and 8 are in all the figures.
(b) 1,5 and 9 are in all the figures.
(c) (d) (c) 1,9 and 10 are in all the figures.
(d) 1,2 and 6 are in the triangle.
13. Which figure represents the relation among 17. Find out the number of all those people who
Lawyers, Teachers and Educated?
can speak Tamil and Telugu?

Telugu Tamil
12 24
35
9
(a) (b) (c) (d) 13 19

14. Which one of the following diagrams 27


represents bachelors, teachers and fathers? English
(a) 3 (b) 59
(c) 21 (d) 112

148
18. In the given figure, the circle stands for
intelligent, square for hard-working, triangle
for postgraduate and the rectangle for loyal 21
employees. Study the figure and indicate the
number which represents post-graduate 14 87
employees who are hard-working and
intelligent but not loyal. 47 32 37 43

8 (a) 21 (b) 14
10
(c) 32 (d) 37
11 7 6 21. In the given diagram, circle represents
9 12 professionals, square represents dancers,
2 4 triangle represents musicians and rectangle
5
1 represents Europeans. Different regions in the
diagram are numbered 1 to 11. Who among the
3 following is neither a dancer nor a musician
but is professional and not a European?

(a) 12 (b) 11
1
(c) 10 (d) 9
2 3
19. In the following figure, the boys who are 11
4 5 10
cricketer and sober are indicated by which
number? 6 7
8 9

Cricketer (a) 10 (b) 8


10
7 Sober (c) 11 (d) 1
6 22. In the above figure, the circle stands for
9 employed, the square stands for social worker,
2
4 Boys the triangle stands for illiterate and the rectangle
1
Girls 8 stands for truthful. Employed, truthful and
illiterate social workers are indicated by which
region?
(a) 6 (b) 5
(c) 4 (d) 2
20. In the given figure, the triangle represents 6
Graduates, rectangle represents Married 7
Persons and circle represents Women, What is 2
8 1 3
the number of those Women who are Graduates 9
10
but not Married? 11 5
4
12

149
(a) 5 (b) 4 25. In the following diagram, police officer
(c) 2 (d) 1 represents circle corrupt represents triangle
poet represents square married represents
23. In the above figure, the circle stands for rectangle. The area representing unmarried
employed, the square stands for social worker, police officers who are not corrupt but are poets
the triangle stands for illiterate and the rectangle is
stands for truthful. Study the figure and answer
the questions which region represents Literate,
employed people who are neither truthful nor 11
social worker. 1
10
2 3 45
6
12 13
6 9 8 7
7
2
8 1 3
9
10 (a) 8 (b) 9
11 5
4 (c) 2 (d) 4
26. Find the urban, corrupt employees in the
following diagram:
Rural
(a) 11 (b) 4 People 2 Urban
People
(c) 9 (d) 10 3 4 66
7 8 9 11
24. In the given diagram, Circle represents
Employees
professionals, Square represents dancers, 12 Corrupt People
Triangles represents musicians and Rectangle
(a) 11
represents Europeans. Different regions in the
(b) 9
diagram are numbered 1 to 11. One the basis
(c) 7
of the diagram, which among the following
(d) 12
represents non-European professional dancers?
27. Triangle represents singers. Rectangle
represents Dancers. Circle represents
Musicians. Square represents Instrumentalists.
1 Names the portion which excludes singers.

11 2 3 10 2
4 5 2 3
6 7 9 4 7
8 6
5

(a) 6 (b) 9 (a) 1256 (b) 4567


(c) 1234 (d) 2347
(c) 5 (d) 7

150
28. In the following figure, the boys who are 31. Which diagram shows District, State and
athletes and are disciplined are indicated by Country in following diagrams?
which number? (CGPSC-2019)

10
6 7 (a)
5 3 2 9
4
1
8 (b)

Girls Athletics
Boys Disciplined

(a) 1 (b) 2 (c)


(c) 6 (d) 10
29. Who are educated males but who do not live
in urban area?

7 (d)
3 10
6 8
12
9 Directions (Q. Nos. 32-34) Read the diagram
4
11
given below. Choose the answer from the given
5 options of the following questions.
(CGPSC-2019)
Urban People

Civil Staff

Male

Educated
(a) 4
(b) 11
(c) 5
(d) 9 = Number of persons who drink tea
30. Which number is in the square, elipse and
= Number of persons who drink coffee
triangle?
= Number of persons who drink wine
4
10 32. Number of those persons who drink only coffee
1 2 5 6 9
7 (a) 25
3 11
(b) 45
(a) 1 (c) 20
(b) 5 (d) 90
(c) 6
(d) 7
151
33. Number of those persons who drink only tea
and coffee both
(a) 7 Answer Sheet
(b) 17 1. (b) 19. (d)
(c) 20 2. (d) 20. (d)
(d) 25
3. (c) 21. (d)
34. Number of those persons who drink all, tea,
4. (a) 22. (d)
coffee and wine
(a) 15 5. (d) 23. (c)
(b) 32 6. (d) 24. (b)
(c) 22 7. (d) 25. (a)
(d) 24 8. (c) 26. (b)
35. Which one of the following diagrams properly
9. (b) 27. (b)
represents the relationship among ‘Body’,
10. (c) 28. (b)
‘Ear’, ‘Mouth’?
11. (c) 29. (b)
(UPSC-2020)
12. (d) 30. (d)
13. (c) 31. (d)
(a) 14. (a) 32. (a)
15. (a) 33. (a)
16. (c) 34. (a)
17. (c) 35. (c)
(b) 18. (a) *****

(c)

(d)

152
21
1. The maximum number of squares in the given (a) (b)
figure is (c) (d)
5. How many triangles are there in the given
figure?

(a) (b)
(c) (d)
2. How many squares are there in the square
figure ABCD? (a) (b)

A B
(c) (d)
6. How many triangels are there in the following
figures?

D C

(a) (b)
(c) (d)
3. How many squares are there in the given
figure? (a) (b)
(c) (d)
7. How many triangles are there in the given
figure?
A B

(a) (b)
(c) (d)
4. How many triangles are there in the following
figure? C D

(a) (b)
(c) (d)
8. How many triangles are there in the given
figure?

153
12. How many rectangles are there in the figure
given?

(a) (b)
(a) (b)
(c) (d) 11
(c) (d)
13. How many rectangles are formed in the figure
9. How many triangles are there in the given
given below?
figure?

B C D E F (a) (b)
(a) (b) (c) (d) 13
14. How many rectangles are there in the given
(c) (d)
figure?
10. Count the number of triangles in the following
figure.

(a) (b) (a) (b)


(c) (d) (c) (d)
11. How many rectangles are there in the given
15. Consider the following figure and answer the
figure?
item that follows:

What is the total number of triangles in the


(a) (b)
above figure?
(c) (d) 14

154
(a) 44 (b) 48 A person goes from A to B always moving to
(c) 50 (d) 58 the right or downwards along the lines. How
16. Consider the following figure and answer the many different routes can he adopt?
following question given below ?
A

B
What are the total number of squares in above
figure ? Select the correct answer from the codes given
(a) 8 (b) 14 below:
(c) 11 (d) 18 (a) 4 (b) 5
17. Consider the following figure and answer the (c) 6 (d) 7
item that follow:
20. Consider the following figure and answer the
itemthat follow:

What is the total number of triangles in the


above figure?
What is the total number of triangles in the (a) 16 (b) 18
above figure? (c) 14 (d) 15
(a) 6 (b) 8 21. Consider the following three-diamensional
(c) 10 (d) 12 figure:
18. Consider the following figure and answer the (UPSC-2018)
question given below:

How many total squares are in above grid?


(a) 30 (b) 36
How many triangles does the above figure
(c) 40 (d) 46 have?
19. Study the following figure: (a) 16 (b) 18
(c) 14 (d) 15

155
22. There are 4 horizontal and 4 vertical (a) 12
lines,parallel and equidistant to one another (b) 17
on a board. What is the maximum number of (c) 13
rectangles and squares that can be formed? (d) None of the above
(UPSC-2017)
(a) 16 (b) 24
(c) 36 (d) 42
23. The total number of triangles in the following
figure?
Answer Sheet
1. (d) 14. (d)
2. (d) 15. (c)
3. (d) 16. (b)
(UPPSC-2020) 4. (a) 17. (c)
(a) 15 (b) 12 5. (b) 18. (c)
(c) 17 (d) 16 6. (b) 19. (c)
7. (a) 20. (b)
24. find the number of triangles in the following
8. (c) 21. (b)
figure.
9. (d) 22. (c)
(MPSC-2020) 10. (c) 23. (a)
11. (b) 24. (c)
12. (d) 25. (b)
13. (a) ******

(a) 16 (b) 18
(c) 20 (d) 24
25. What is the number of squares in the following
figure?
(UKPSC-2022)

156
Explanations QFJL; NQSL

4. (A) A
1. (d) A B C D
M N
L E F G E
K P
K F
B D C
J I H G
The triangles are:
The squares are : AFG, AGE, BFG, BDG, CGE,
ABML, BCNM, CDEN, LMOK, CGD, AGB, AGC, BEC, BCF,
MNPO, NEFP, KOIJ, OPHI, ABE, ACD, ADB, ADC, GCB,
PFGH, ACPK, BDFO, LNPK, ABC
MEGI, ADGJ Thus, there are altogether 16 triangles.
Thus, there are 14 squares. A
5. (B)
2. (d) A B C D E H I B
Q R S J M K
P F G C
L
T U V D
O G F
W X Y
N H
E
M L K J I The squares are :
The squares are : AFG, AGE, BFG, BDG, CGE,
ABQP, BCRQ, CDSR, DEFS, PQTO, CGD, AGB, AGC, BEC, BCF,
QRUT, RSVU, SFGV, OTWN, TUXW, ABE, ACD, ADB, ADC, GCB, ABC
UVYX, VGHY, NWLM, WXKL, XYJK, There are 28 triangles.
YHIJ, ACUO, BDVT, CEGU, PRXN, 6. (B) A
QSYW, RFHX, OUKM, TVJL, UGIK, D E
ADYN, BEHW, PSJM, QFIL, AEIM G H
F
Thus, There are 30 squares.
I F D L
3. (d) A B
C D
P Q B M N O C
E F
O
R SG H
The squares are :
N
 ABC;  ADE;  AFH;  AIL;  D F G ;
M L K J I
 DIK;  DBO;  GDE;  EGH;  EJL;
The squares are :  EMC;  FIJ;  FBN;  JFG;  GJK;
ABCP; PCQO; CDEQ;  KGH:  HKL;  HNC;  NFH;  G M O ;
OQRN; QESR; EFGS;  IBM;  MIJ;  JMH;  NJK;  K N O ;
NRLM; RSKL; SGJK;  OKL;  LOC;
GHIJ; PDSN; OEKM;

157
18 + 14 + 6 + 2 = 40
A E B
7. (A) 9. (d) The triangles are:  AFE,  EFI,  BFG,,
 FGI,  GCH,  GHI,  DEH,  HEI,
H I F
 EFG,  EGH,  FHE,  FHG

C G D

The triangles are:


 AIH;  AIE;  EIB;  BFI
 IHC;  IGC;  IGD;  DFI; B D E F C
 IAB;  IGD;  ICD;  IAC;
 BAC;  ACD  BDC;  BDA 10. (C) C D E

B M N F
8. (C) A B C
J
P Q D G
A L K I H
R S
E The triangles are:
O N M G F
T
L H  ABL;  BLK;  BMK;  BMC;
I
 CMJ;  CDJ;  DEJ;  MKJ;
K J  EJN;  EJN;  JIN;  ENF;  NFI;
 FIH;
The simplest triangles are:
 FGH;  ABK;  BCK;  BCJ;
 PNO;  PNM;  MPQ;  MQR;  KBJ;  JCE;  EFJ;  IFJ;  FEI;
 AQP;  AQR;  BRA;  BRC;  FEI;  FGI;  CKA;  CKE;  CEI;
 SRC;  SCD;  SGR;  SGD;  EGI;  CJK;  EJI;
 DFG;  DFE  TLM;  TJK;
 TLK;  TIH; 11. (B) A E F B
The triangles composed of two
components are: L M
 PON;  PMA;  APR;  RAM; N G
K
O P
 RAC;  RGC;  DGC;  DGE;
Q H
 MPR;  GRD;  DCR;  TMK;
 TKI;  TIG; D J I C
The triangles composed of four
components are: The triangles are: ABGK, KGCD,A F P K ,
 AMO;  AMC;  CAG; NGCJ, AEML, LMOK, EFPO, FBGP, AEID,
EBCI, EBHQ, LMID, KNJD, NOIJ, OGHQ,
 CGE;  MKI;  GIK;
QHCI
Other triangles are:  SPI;  DQK;
Thus, there are 30 squares.
Total number of triangles

158
=6
There are 6 triangles in figure like  ABC
12. (D) A E G B

K J
 Total number of such triangles = 6  6 =36
There are 6 triangles in hexagon B D F G
E C which join at centre O.
D I C Number of triangles like A D E = 6
large size of A J K and I L H and 2 more
The triangles are: AEFK, EGHF, GBJH, HJCI, triangles
AGHK, ABJK. GBCI, KHID, EBJF,A G I D , Thus, number of triangles in figure = 36
ABCD + 6 + 6 + 2 = 50

13. (B) A B 16. (b) A B


L C D N
K
C D E
J I F E
O
G H M
H G F

The triangles are:


ABCL, DEFC, GHIF, KJIL, ABFI, ABGH, L
I J K
DEIL, DEJK, GHLC, KCFJ, LCFI
14. (*) A B C It is clear from the figure that there are 6 squares
I J K like ABCD. There are 6 squares like CNDO.
L M N
There are two big squares FDHJ and CEKI.
H D
Thus, tatol number of squares = 6 + 6 + 2 = 14
Q squares.
O P
G F E Hence option (b) is correct.
The triangles are: 17. (c) Number of squares in the first type of grid-
IJPO; JKQP; IKNL; LNQO; ABFG; BCEF;
ACDH; HDEG; 1 2 3 4
Squares are also rectangles : there are 10 2
squares: 3
ABMH; BCDM; HMFG; MDEF; IJML;
4
JKNM; MNQP; LMPO; ACEG; IKQO;
15. (c) A 4  4 = 16
B C  3 3 = 9
I L
X Y
 2 2 = 4
D
O
E  1 1 = 1
= 30
J K
F G

H
Number of triangles in ABC = (1 + 2 + 3 )

159
 Number of squares of square = 5 + 5 = 10
 Total number of squares = 30 + 10 = 40
Hence, option (c) is correct.

18. (c) A
B C
G H

I J

D E

Number of ABC type triangles = 6


Number of ADE type triangles = 2
There are two more triangles FIJ and AIJ on line
IJ.
So, total number of triangles in given figure
= 6 + 2 + 2 = 10

19. (b) B C

F H
G
E I
M N
A D
L K J

We may label the figure as shown.


The simplest triangles are BFG, CGH,
EFM, FMG, GMN, GHN, HNI, LMK,
MNK and KNJ i.e. 10 in number.
The triangles composed of three components
each are FAK and HKD i.e. 2 in number.
The triangles composed of four components
each are BEN, CMI, GLJ and FHK i.e. 4 in
number.
The triangles composed of eight components
each are BAJ and CLD i.e. 2 in number thus,
there are 10 + 2 + 4 + 2 = 18 triangles in the
given figure.
160
22
Directions for the following 3 (three) items:
In each item, there are two sets of figures. first
(d)
four figures 1, 2, 3 and 4 are named as problem
figures and next four figures named answer figures
indicated as (a), (b), (c) and (d). The problem
figures follow a particular sequence. In accordance
with the same, which one of the four answer figures 2. Problem figures:
should appear as the fifth figure?
1. Problem figures:

1 2

1 2

3 4
Answer figures:

3 4
(a)
Answer figures:

(a)
(b)

(b)
(c)

(c)
(d)

161
3. Problem figures: 3 4
Answer figures:

(a)

1 2

(b)

3 4
(c)
Answer figures:

(a)
(d)

5. Problem figures:
(b)

(c)

Answer figures:
(d)

(a)
4. Problem figures:

(b)

1 2 (c)

(d)

162
6. Problem figures:

(d)

8. Problem figures:

?
Answer figures:

Answer figures:
(a)

(a)

(b)

(b)

(c)

(c)

(d)

(d) 9. Problem figures :

7. Problem figures:

Answer figures :
Answer figures:

(a) (a)

(b)
(b)

(c)

163
(c) (a)

(d) (b)

10. Problem figures :

(c)

Answer figures :
(d)

(a) 12. Question Figures:

(b)
1 2 3 4
Answer Figures:
(c)

(a)

(d)

11. Question Figures: (b)

(c)
1 2 3 4
Answer Figures:

164
(d) (b)

13. Problem Figures:

(c)

1 2 3 4
Answer Figures: (d)

15. Problem Figures:


(a)

1 2 3 4
(b)
Answer Figures:

(a)
(c)

(b)
(d)

14. Problem Figures:


(c)

1 2 3 4
(d)
Answer Figures:

16. Problem figures:

(a)

165
(c)

1 2 3 4
(d)
Answer figures:

18. Problem figures:


(a)

(b)
Answer figures:

(c) (a)

(d) (b)

17. Problem Figures:

(c)

1 2 3 4
Answer figures: (d)

19. Problem figures:


(a)

(b)
1 2 3 4
Answer figures:

166
21. Problem figures:

(a)

1 2 3 4
(b)
Answer figures:

(c)
(a)

(d)
(b)

20. Problem figures:

(c)

1 2 3 4
(d)
Answer figures:

22. Problem figures:


(a)

1 2 3 4
(b)
Answer figures:

(a)
(c)

(b)
(d)

167
Answer figures:

(c)

(a)

(d)

(b)
23. Problem figures:

(c)
1 2 3 4
Answer figures:

(d)
(a)
25. Problem figures:

+ =
+

=
(b) +
=

= +

1 2 3 4
Answer figures:
(c)

=
(a)
+
(d)

+
24. Problem figures:
(b) =

+
1 2 3 4 (c) =

168
+
(d) = (a)

26. Problem figures:

(b)

1 2 3 4
Answer figures: (c)

(a)
(d)

28. Problem figures:


(b)

1 2 3 4
(c)
Answer figures:

(a)
(d)

27. Problem figures:

(b)

1 2 3 4
Answer figures: (c)

169
(d) (a)

29. Problem figures:

(b)

1 2 3 4
Answer figures: (c)

(a)
(d)

31. Consider the figures given below:


(b)
(UPSC-2018)

(c)

(a)
(d)
(b)

30. Problem figures: (c)

(d)
32. Consider the figures given below:
(UPSC-2018)
1 2 3 4
Answer figures:
(I) (II) (III) (IV) (V) (VI)

170
In the figures (I) to (VI) above, some parts are
shown to change their positions in regular
directions, Following the same sequence which
of the figures given below will appear at (VII)
stage?

(a) (b) (c) (d)

Answer Sheet
1. (b) 2. (c) 3. (c) 4. (c) 5. (c)
6. (d) 7. (d) 8. (c) 9. (b) 10. (b)
11. (c) 12. (b) 13. (c) 14. (b) 15. (b)
16. (c) 17. (b) 18. (b) 19. (b) 20. (c)
21. (b) 22. (b) 23. (c) 24. (c) 25. (d)
26. (a) 27. (b) 28. (a) 29. (b) 30. (c)
31. (a) 32. (b) ********************

171
Explanations  Fifth figure will be (c)
1. (b) In given figure; star, circle and arrow are 5. (c) Second figure is the water image of first
arranged in specific rule. There is
figure. Third figure is formed by rotat-
triangle in circle in first figure. There is
circle within circle in second figure. ing 900 anticlockwise to second figure.
There is triangle in circle again. fourth Again fourth figure is water image of
figure has circle within circle again. third figure there-fore fifth figure will
Therefore, in next figure, there would
be 900 anticlockwise rotation of fourth
be triangle within circle which is in
options (b) and (c). Again, star is figure.
changing from first to fourth figure. In
first figure, it is outside of triangle. In
second figure it is within it. In third, it is
outside. In fourth, it is within therefore
in fifth, it will be outside alongwith its
position will be towards cen tre.
therefore, (b) is option.  Thus answer will be (c)
6. (d) Circle is rotating clockwise 900 in prob-
2. (c) In given figure, every sign is displacing
in certain directions. Triangle, circle and lem figures.
square are moving anti-clockwise. Si-
multaneously there is no change in the
base of triangle, while triangle above it
moves left or right. Therefore, (c) will Dot is rotating anticlockwise 900 in prob-
be correct option. lem in square.
3. (c) In given figure, inner circle is changing
anti-clockwise. Figure first has four parts
of outer circle while figure second has
verti-cally half left part undivided. Fig- Therefore, answer will be (d).
ure third has vertically half right part un-
divided outer circle then fourth figure 7. (d) After observing question figures, it
has four parts of outer circle. Therefore, becomes clear that innermost triangle
next figure would have half vertically changes from blank to dark alternately
left undivided outer circle. This is in and outermost triangle has a line on one
vertice then another line adds on other
only in option (c).
vertex in clockwise direction then again
4. (c) When a circle comes out, it becomes sec- a line adds on its vertices clockwise. This
ond figure. Second and third figures are pattern follows in answer figure (d).
same. Again this process is repeated for Hence the next figure will be option (d).
figure four and five. 8. (c) It is clear from question figures that
every circle has one more line to its
previous circle. Therefore, option (c)
would be correct answer.
9. (b) Shaded part of circle moves two-step
clock wise then three-step clock wise

172
then two-step clock wise then it should triangle then again circle to triangle.
move 3-step clockwise which is in Hence next inner figure will be circle.
option (b).
17. (b) By observing the figure, we find that
10. (b) From observing question figure, one line dark circle moves two spaces
adds perpendicularly in next figure anticlockwise and rectangle move
anticlockwise. Thus there would be 5 clockwise. Hence (b) will be answer.
line and fifth line would be parallel to
original first line because it starts the 17. (b) By observing the figure series, we find
order. Point is moving perpendicularly the patter moves two spaces
anticlockwise. Thus final point would be anticlockwise then one space clockwise
parallel to the first original point. It is in then two spaces anticlockwise. Thus
option (b). next pattern will be one space clockwise.
11. (c) After observing question figures, arrow Hence (b) will be answer.
moves 900 anticlockwise from figure
19. (b) Star is rotating clockwise at each corner.
first and corner-circle line moves
clockwise at each corner from figure Option (b) satisfies this pattern
first. Option (c) clearly satisfies it. 20. (c) Since, triangle changes into square and
12. (b) C moves anticlockwise in reverse order square changes into triangle and so on.
at each corner. Standing line moves Inner parallel becomes outer one and
clockwise at each-corner with one outer parallel becomes inner one and so
additional line. Arrow line moves on. Option (c) satisfies this relation.
anticlockwise on opposite sides. Middle
21. (b) Dark circle is moving anticlockwise
figure moves from 1 to 2 by water image
from inner square to outer square and
then from 2 to 3 by mirror image them
form 3 to 4 by water image then from 4 outer square to inner one and so on.
to answer figure by mirror image ie. Option (b) satisfies this relation.
Option (b) satisfies this pattern. 22. (b) Lines at the corner of square increase
13. (c) Diagonal line moves 45 clockwise
0 by 1 line at each corner by moving
while arrow line moves 135 0 anticlockwise. Inner line of hexagon is
anticlockwise. Thus, (c) will be answer moving one step anticlockwise while
figure. outer line is moving one step clockwise
around hexagon. Thus, Option (b) will
14. (b) Triangle moves one-step clockwise
while dot mores one-step anticlockwise. be correct answer figure.
Thus (b) will be answer.
23. (c) In given diagrams dot successfuly move
15. (b) Triangle moves one-step anticlockwise hozontally in the sequence therefore
and dark dot also moves one-step according to this pattern option (c)
anticlockwise while square moves one- correct.
step clockwise. Hence, (b) will be
correct answer. 24. (c) Outer square of the diagram rotate 450
anti-clockwise while inner square shape
16. (c) By observing the figure, we find that structure move clockwise with 1350. So
innermost figure changes from circle to option (c) is correct.

173
25. (a) On observing carefully. We notice that
(=) rotates anticlockwise from outside
to inside, hence, next figure would (d).
26. (b) On observing carefully, we notice that
open figure move clockwise with 450 at
stage. Shaded part moves anticlockwise
with 1350 at each step. Hence, option (b)
will be correct option.
27. (b) Shaded part moves two step clockwise
then it moves three steps anticlockwise
within sub-divided circle. Again, it
moves two step clockwise then again
should move three step anticlockwise.
Thus, option (b) will be answer.
28. (b) Problem figures 1,2 and 3 form a pattern
of series. The next pattern starts with
figure 4 and follows the same pattern.
Hence, option (b) will be next figure
according to this pattern.

29. (b) Shaded part within circle moves three


steps anti-clockwise then two steps
clockwise then again follows this
pattern. Hence, the next figure will be
option (b).

30. (c) Inner triangle moves vertically opposite


to its previous position in all figures.
Inner circle changes to rectangle then
circle to rectangle. Hence next figure
will be option (c).

174
23
1. Question Figures: 5. Question Figures:

Answer Figures: Answer Figures:

(a) (b) (c) (d) (a) (b) (c) (d)

2. Question Figures: 6. Question Figures:

Answer Figures:

Answer Figures:

(a) (b) (c) (d)


3. Which of the given answer figures will be the
exact mirror image of the question figure when (a) (b) (c) (d)
the mirror is placed at AB? 7. Question Figures:
Question Figures:

Answer Figures:
Answer Figures:

(a) (b) (c) (d)


(a) (b) (c) (d) 8. Question Figures:
4. A clock having dots only at positions 3, 6, 9
and 12 is placed inverted in front of a
mirror. In its image, the time 4:45 will
approximately look like which of the Answer Figures:
following?
(a) 2:15 (b) 8:45
(c) 8:50 (d) 9:45

175
(a) (b) (c) (d)
9. Question Figures:
(a) (b) (c) (d)
13. Question Figures:

Answer Figures:

(a) (b) (c) (d)


Answer Figures:
10. Question Figures:
(a)
(b)
(c)
(d)
Answer Figures:
14. Question Figures:

(a) (b) (c) (d)


Answer Figures:
11.
AB
Question Figures:
(a) (b) (c) (d)
15. Question Figures:
Answer Figures:

(a) (b) (c) (d)


12.
Answer Figures:
(a)
Question Figures: (b)
(c)
(d)
16. Question Figures:

Answer Figures:

176
(d)
19. Question Figures:

Answer Figures:

(a)
(b) N
(c)
Answer Figures:
(d)
(a)
17. Question Figures: (b)
x (c)
(d)
20. Question Figures:

y
Answer Figures:

(a)
(b)
Answer Figures:
(c)
(d)
(a)
18. Question Figures:

M (b)

(c)

N
(d)
Answer Figures:
(a)
(b)
(c)

177
Directions : In each of the following
questions, you are given a combination of
alphabets and/or numbers followed by fouor
21. Question Figures:
alternatives (a), (b), (c) and (d). Choose the
alternative which msot closely resembles the
X mirror-image of the given combination.
24. PAINTED
A
B C

Y
25. NATIONAL
Answer Figures:

26. ANS43Q12
(a) (b) (c) (d)
22. Question Figures:

REGION 27. BR4AQ16HI

Answer Figures:

28. 1965 I N DOP A K

(a) (b) (c) (d)


23. Question Figures: 29. Which of the following collections of letters
will look the same in the mirror?
M
(a) OSMIHOM
TEACHER
(b) VHRTRVH
N (c) HIMOSTA
Answer Figures: (d) AOVIVOA
Directions (Questions 30-32) : In each of the
following questions, choose the correct
mirror-image of the fig. (x) from amongst the
four alternatives (a), (b), (c) and (d) given
(a) (b) (c) (d) along with it.

178
30. 37.

31.

38.

32.

Directions : in each of the following


questions, you are given a combination of
alphabets and/or numbers followed by four
alternatives (a), (b), (c) and (d). Choose the
alternative which closely resembles the water.
image of the given combination.
33. DISC
39. Find out the correct option for mirror (vertical)
image of the following figure.
(MPPSC-2020)
34. FAMILY

35. NUCLEAR

36. A1M3b (a) (b) (c) (d)

Directions : In each of the following questions,


choose the correct water-image of the Fig. (X)
from amongst the four alternatives (a), (b), (c)
and (d) given along with it.

179
Answer Sheet

1. (b) 2. (c) 3. (d) 4. (a) 5. (c)


6. (d) 7. (c) 8. (d) 9. (a) 10. (a)
11. (d) 12. (c) 13. (a) 14. (d) 15. (d)
16. (c) 17. (d) 18. (c) 19. (c) 20. (c)
21. (c) 22. (a) 23. (d) 24. (b) 25. (b)
26. (b) 27. (a) 28. (d) 29. (d) 30. (d)
31. (c) 32. (c) 33. (c) 34. (d) 35. (d)
36. (c) 37. (c) 38. (b) 39. (b) *****

180
Explanation
14. (d)

1. (b) 15. (d)

16. (c)
2. (c)
17. (d)

18. (c)
3. (d)
19. (c)

20. (c)
4. (a)

21. (c)

5. (c)
22. (c)

6. (d)
23. (c)

7. (c)

8. (d)

9. (a)

10. (a)

11. (d)

12. (c)

13. (a)

181
24
1. Question Shapes:

Answer Figures:
Answer Figures:

(a) (b) (c) (d)


(a) (b) (c) (d) 5. Question Shapes:
2. Question Shapes:

Answer Figures:
Answer Figures:

(a) (b) (c) (d)


(a) (b) (c) (d) 6. Question Shapes:
3. Which answer figure will the paper look like
after it is folded, cut and opened as shown in
the question figures below?
Question Shapes: Answer Figures:

Answer Figures: (a) (b) (c) (d)


7. Question Shapes:

(a) (b) (c) (d)


Answer Figures:
4. Question Shapes:

182
after it is folded, cut and opened as shown in
the question figures below?
Question Shapes:
(a) (b) (c) (d)
8. Question Shapes:

Answer Figures:

Answer Figures:

(a) (b) (c) (d)


12. Which answer figure will the paper look like
after it is folded, cut and opened as shown in
(a) (b) (c) (d)
the question figures below?
9. Which answer figure will the paper look like
Question Shapes:
after it is folded, cut and opened as shown in
the question figures below?
Question Shapes:

Answer Figures:

Answer Figures:

(a) (b) (c) (d)


13. Which answer figure will the paper look like
after it is folded, cut and opened as shown in
(a) (b) (c) (d)
the question figures below?
10. Which answer figure will the paper look like
after it is folded, cut and opened as shown in Question Shapes:
the question figures below?
Question Shapes:

Answer Figures:

Answer Figures:

(a) (b) (c) (d)


14. Which answer figure will the paper look like
(a) (b) (c) (d) after it is folded, cut and opened as shown in
11. Which answer figure will the paper look like the question figures below?

183
Question Shapes:
Answer Sheet

1. (c) 2. (b) 3. (c) 4. (d) 5. (c)


6. (b) 7. (d) 8. (c) 9. (b) 10. (d)
Answer Figures: 11. (b) 12. (b) 13. (d) 14. (a) 15. (d)

(a) (b) (c) (d)


15. Which of the following figures can be drawn
without lifting the pencil and without
repeating a line?
(CGPSC-2019)

(a) A and C (b) B and D


(c) A and B (d) A, C and D

184
Explanation
11. (b)

1. (c)

12. (b)

2. (b)

13. (d)

3. (a)

14. (a)

4. (d)

5. (c)

6. (b)

7. (d)

8. (c)

9. (b)

10. (d)

185
25
Directions (Questions 1 to 42): In each I. All water is temple.
question below are given two statements II. All temples are water.
6. Statements : All cars are cats. All fans are
followed by two conclusions numbered I and II.
cats.
You have to take the given two statements to be Conclusions :
true even if they seem to be at variance from I. All cars are fans.
commonly known facts. Read the conclusion and II. Some fans are cars.
then decide which of the given conclusions 7. Statements : All pens are chalks. All chairs
logically follows from the two given statements, are chalks.
Conclusions :
disregarding commonly known facts.
I. Some pens are chairs.
Give answer II. Some chalks are pens.
(a) if only conclusion I follows’, 8. Statements: All tubes are handles. All cups
(b) if only conclusion II follows’, are handles.
(c) if either conclusion I or II follows; Conclusions :
(d) if neither conclusion I nor II follows’, I. All cups are tubes.
(e) if both conclusions I and II follow. II. Some handles are not cups
9. Statements : All bags are cakes. All lamps
1. Statements: All men are dogs. All dogs are
are cakes.
cats. Conclusions :
Conclusions : I. Some lamps are bags.
I. All men are cats. II. No lamp is bag.
II. All cats are men. 10. Statements : All roads are waters. Some
2. Statements : All film stars are playback waters are boats.
singers. All film directors are film stars. Conclusions :
Conclusions : I. Some boats are roads.
I. All film directors are playback singers. II. All waters are boats.
II. Some film stars are film directors. 11. Statements : All jungles are tigers. Some
tigers are horses.
3. Statements : All pens are roads. All roads are
Conclusions :
houses.
I. Some horses are jungles.
Conclusions :
II. No horse is jungle.
I. All houses are penis.
12. Statements : All birds are tall. Some tall are
II. Some houses are pens.
hens.
4. Statements : All huts are mansions. All Conclusions :
mansions are temples. I. Some birds are hens.
Conclusions : II. Some hens are tall.
I. Some temples are huts. 13. Statements : All artists are smokers. Some
II. Some temples are mansions. smokers are drunkards.
5. Statements : All water is divine. All temples Conclusions :
are divine. I. All smokers are artists.
Conclusions : II. Some drunkards are not smokers.

186
14. Statements : Some hens are cows. All cows Conclusions :
are horses. I. No crocodile is a fish.
Conclusions : II. No fish is a crocodile.
I. Some horses are hens. 24. Statements : All windows are doors. No door
II. Some hens are horses. is wall.
15. Statements : Some pastries are toffees. All Conclusions :
toffees are chocolates. I. No window is wall.
Conclusions : II. No wall is door.
I. Some chocolates are toffees. 25. Statements : All roads are poles. No pole is a
II. Some toffees are not pastries. house.
16. Statements : Some kings are queens. All Conclusions :
queens are beautiful. I. Some roads are houses.
Conclusions : II. Some houses are poles.
I. All kings are beautiful. 26. Statements : All flowers are trees. No fruit is
II. All queens are kings. tree.
17. Statements : All men are married. Some men Conclusions :
are educated. I. No fruit is flower.
Conclusions : II. Some trees are flowers.
I. Some married are educated. 27. Statements : All mangoes are golden in
II. Some educated are married. colour. No golden-coloured things are cheap.
18. Statements : Some adults are boys. Some Conclusions :
boys are old. I. All mangoes are cheap.
Conclusions : II. Golden-coloured mangoes are not cheap.
I. Some adults are not old. 28. Statements : No gentleman is poor. All
II. Some boys are not old. gentlemen are rich.
19. Statements : Some books are tables. Some Conclusions :
tables are mirrors. I. No poor man is rich.
Conclusions : II. No rich man is poor.
I. Some mirrors are books. 29. Statements : No magazine is cap. All caps
II. No book is mirror. are cameras.
20. Statements : Some dreams are nights. Some Conclusions :
nights are days. I. No camera is magazine.
Conclusions : II. Some cameras are magazines.
I. All days are either nights or dreams: 30. Statements : Some desks are caps. No cap is
II. Some days are nights. red.
21. Statements : Some papers are pens. Some Conclusions :
pencils are pens. I. Some caps are desks.
Conclusions : II. No desk is red.
I. Some pens are pencils. 31. Statements : Some books are pens. No pen is
II. Some pens are papers. pencil.
22. Statements : Some doctors are fools. Some Conclusions :
fools are rich. I. Some books are pencils.
Conclusions : II. No book is pencil.
I. Some doctors are rich. 32. Statements : No women teacher can play.
II. Some rich are doctors. Some women teachers are athletes.
23. Statements : All fish are tortoise. No tortoise Conclusions :
is a crocodile.
187
I. Male athletes can play I, II, III and IV. You have to take the given
II. Some athletes can play. statements to be true even if they seem to be at
33. Statements : No bat is ball. No ball is wicket. variance from the commonly known facts and
Conclusions : then decide which of the given conclusions
I. No bat is wicket. logically follows from the given statements
II. All wickets are bats. disregarding commonly known facts.
34. Statements : Many scooters are trucks. All 41. Statements : All branches are flowers. All
trucks are trains. flowers are leaves.
Conclusions : Conclusions :
I. Some scooters are trains. I. All branches are leaves.
II. No truck is a scooter. II. All leaves are branches.
35. Statements : Most teachers are boys. Some III. All flowers are branches.
boys are students. IV. Some leaves are branches.
Conclusions : (a) None follows
I. Some students are boys. (b) Only I and IV follow
II. Some teachers are students. (c) Only II and III follow
(d) All follow
36. Statements : No man is a donkey. Rahul is a
42. Statements : All politicians are honest. All
man.
honest are fair.
Conclusions : Conclusions :
I. Rahul is not a donkey. I. Some honest are politicians.
II. All men are not Rahul. II. No honest is politician.
37. Statements : All boys are honest. Sachin is III. Some fair are politicians.
honest. IV. All fair are politicians.
Conclusions : (a) None follows
I. Sachin is a boy. (b) Only I follows
II. All honest persons are boys (c) Only I and II follow
38. Statements : Some engineers are fools. Anand (d) Only I and III follow
is an engineer. 43. Statements :All terrorists are guilty. All
Conclusions : terrorists are criminals.
I. Some fools are engineers. Conclusions :
I. Either all criminals are guilty or all guilty
II. Anand is a fool.
are criminals.
39. Statements : Some men are educated.
II. Some guilty persons are criminals.
Educated persons prefer small families. III. Generally criminals are guilty
Conclusions : IV. Crime and guilt go together.
I. All small families are educated. (a) Only I follows
II. Some men prefer small families. (b) Only I and III follow
40. Statements : All poles are guns. Some boats (c) Only II follows
are not poles (d) Only II and IV follow
Conclusions : 44. Statements: Some tables are TVs. Some TVs
I. All guns are boats. are radios.
II. Some boats are not guns. Conclusions :
Directions (Questions 41 to 45): In each of I. Some tables are radios.
the following questions, two statement are given II. Some radios are tables.
followed by three or four conclusions numbered III. All radios are TVs.

188
IV. All TVs are tables. D : Some creatures are mammals. Some
(a) None follows creatures are viviparous. Some mammals
(b) All follow are viviparous.
(c) Only I and III follow (a) A only (b) B only
(d) Only II and IV follow (c) C only (d) D only
45. Statements : Some bags are pockets. No 48. A : A few professors are scientists. Some
pocket is a pouch. scientists eat grass. A few professors eat
Conclusions :
grass. B : Ajay is tree. Some trees are
I. No bag is a pouch.
made of iron. Ajay is made of iron.
II. Some bags are not pouches.
C : No cows eat grass. All bulls eat grass. No
III. Some pockets are bags.
bulls are cows.
IV. No pocket is a bag.
D : Some squirrels are donkeys. All squirrels
(a) None follows
(b) Only I and III follow are wolves. Some wolves are donkeys.
(c) Only II and III follow (a) A and C (b) B only
(d) Only either I or IV follows (c) C only (d) C and D
(e) All follow 49. A : Giraffes are carnivores. Kangaroos are
Directions (Questions 46 to 50): Each not giraffes. Kangaroos are not
question given below has a set of three or four carnivores.
statements. Each set of statements is further B : All teachers are intelligent. No intelligent
divided into three segments. Choose the people are crooks. No crooks are teachers.
alternative where the third segment in the C : No television is radio. No car is radio.
statement can be logically deduced using both Televisions are cars:
the preceding two, but not just from one of D : All dogs like milk. Some cats like milk.
them. Some cats are dogs.
46. A : All beautiful things are sad. She is (a) A and D (b) B only
beautiful. She is sad. (c) C only (d) B and G
B : All nice things are flat. TVs are flat. TVs
50. A : Many poets are not readers. All singers
are nice things.’
are poets. Some singers are not readers.
C : Potatoes are stems. All stems are fruits.
B : Boys play cricket. Some girls do not play
Potatoes are fruits.
cricket. Some girls are not boys.
(a) A only (b) A and B
C : All Eskimos live in igloos. Some
(c) C only (d) A and C
penguins live in igloos. Some penguins
47. A : All mammals are viviparous. Some fish
are Eskimos.
are viviparous. Some fish are mammals.
(a) A only (b) B only
B : All birds are oviparous. Some fish are not
(c) C only (d) B and C
oviparous. Some fish are birds.
Directions (Questions 51 to 56): Each question
C : No mammal is oviparous. Some creatures
given below consists of five or six statements
are oviparous. Some creatures are not followed by options consisting of three
mammals. statements put together in a specific order.
Choose the option which indicates a valid
argument containing logically related statements

189
that is, where the third statement is a conclusion E : Poor girls want to marry rich girls.
drawn from the preceding two statements. F : Rich boys want to marry poor girls.
51. A : All synopses are poets. (a) ABC (b) ADE
B : Some synopses are mentors. (c) BCD (d) DEF
C : Some X are not mentors. Directions (Questions 57 to 59): In each of
D : All X are poets. the following questions, examine the given
E : All synopses are mentors. statements carefully and find out which two
F : All synopses are X. of the statements can not be true
(a) ACB (b) AEC simultaneously, but can both be false.
(c) FEC (d) DFA 57. 1. All animals are carnivorous.
52. A : No window is a door. 2. Some animals are not carnivorous.
B : All poles are doors. 3. Animals are not carnivorous.
C : No pole is a window. 4. Some animals are carnivorous.
D : Some poles are not windows. (a) 1 and 2 (b) 2 and 3
E : Some windows are poles. (c) 1 and 3 (d) 3 and 4
F : Some doors are not windows. 58. 1. All children are inquisitive.
(a) ABC (b) ACB 2. Some children are inquisitive.
(c) BDA (d) DFA 3. No children are inquisitive.
53. A : Some abra are dabra. 4. Some children are not inquisitive,
B : All abra are cabra. (a) 1 and 3 (b) 1 and 4
C : All dabra are abra. (c) 2 and 3 (d) 3 and 4
D : All dabra are not abra. 59. 1. Some nations wish for peaceful
E : Some cabra are abra. coexistence.
F : Some cabra are dabra. 2. All nations wish for peaceful coexistence.
(a) ABD (b) AEF 3. Some nations are not wishing for peaceful
(c) BCE (d) BCF coexistence.
54. A : No patriot is a criminal. 4. No nations are wishing for peaceful
B : Ramdas is not a criminal. coexistence.
C : Ramdas is a patriot. (a) 1 and 2 (b) 1 and 3
D : Mohandas is not a patriot. (c) 2 and 4 (d) 3 and 4
E : Mohandas is a criminal Directions (Questions 60 to 67) : In each of
(a) ABC (b) ACB the questions below are given three statements
(c) ABE (d) ADE followed by three conclusions numbered I, II
55. A : All vegetarians eat meat. and III. You have to take the given statements
B : All those who eat meat are not to be true even if they seem to be at variance
vegetarians. from the commonly known facts. Read all the
C : All those who eat meat are herbivorous. conclusions and then decide which of the
D : All vegetarians are carnivorous. given conclusions logically follows from the
E : All those who eat meat are carnivorous.
given statements disregarding commonly
F : Vegetarians are herbivorous.
known facts.
(a) ABE (b) ACD
60. Statements: All dolls are windows. All bottles
(c) ACF (d) BCE
are windows. All cars are bottles.
56. A : Poor girls want to marry rich boys.
B : Rich girls want to marry rich boys. Conclusions:
C : Poor girls want to marry poor boys. I. All cars are windows.
D : Rich boys want to marry rich girls. II. Some cars are dolls.

190
III. Some windows are cars. II. No camel is tiger.
(a) Only I and II follow III. Some tigers are cows.
(b) Only II and III follow (a) None follows
(c) Only I and III follow (b) Only I follows
(d) All follow (c) Only II follows
(e) None of these. (d) Only III follows
61. Statements : All fruits are vegetables. All pens (e) Either f or II follows
are vegetables. All vegetables are rains. 65. Statements: All flowers are toys. Some toys
Conclusions: are trees. Some angels are trees.
I. All fruits are rains. Conclusions :
II. All pens are rains. I. Some angels are toys.
III. Some rains are vegetables. II. Some trees are flowers.
(a) None follows III. Some flowers are angels.
(b) Only I and II follow (a) None follows
(c) Only II and III follow (b) Only I follows
(d) Only I and III follow (c) Only II follows
(d) Only III follows
(c) All follow
(e) Only I and III follow
66. Statements : Some boxes are hammers. Some
62. Statements: All snakes are trees. Some trees
hammers are beads. All beads are rings.
are roads. All roads are mountains.
Conclusions:
Conclusions:
I. Some rings are hammers.
I. Some mountains are snakes.
II. Some hammers are boxes.
II. Some roads are snakes.
III. Some rings are boxes.
III. Some mountains are trees.
(a) None follows
(a) Only I follows
(b) Only I follows
(b) Only II follows
(c) Only I and II follow
(c) Only III follows
(d) Only II and III follow
(d) Both I and II follow
(e) All follow
(e) None follows
67. Statements: Some rats are cats. Some cats are
63. Statements: Some blades are hammers. Some
dogs. No dog is cow.
hammers are knives. Some knives are axes.
Conclusions:
Conclusions:
I. No cow is cat.
I. Some axes are hammers.
II. No dog is rat.
II. Some knives are blades.
III. Some cats are rats.
III. Some axes are blades.
(a) None follows
(a) None follows
(b) Only I and II follow
(b) Only I follows
(c) Only II and III follow
(c) Only II follows
(d) Only II follows
(d) Only III follows
(e) All I, II and III follow
(e) None of these
Directions (Questions 68 to 73) : In each, of
64. Statements: All tigers are lions. No cow is
the following questions, three statement are
lion. Some camels are cows.
given followed by four conclusions numbered
Conclusions:
I, II, III and IV. You have to take if given
I. Some lions are camels.
statements to be true even if they seem to be at
191
variance with commonly known facts and then III. Some teachers are not houses.
decide which of the given conclusions logically IV. No college is house.
follows from the give statements disregarding (a) None follows
commonly known facts. (b) Only either I or IV follows
68. Statements: All rods are bricks. Some bricks (c) Only II, III and IV follow
are ropes. All ropes are doors (d) All follow
Conclusions: (e) Only either I or IV, and III follow
I. Some rods are doors. 72. Statements: No tree is fruit. All fruits are
II. Some doors are bricks. stones. All stones are rains.
III. Some rods are not doors. Conclusions:
IV. All doors are ropes. I. No stone is tree.
(a) Only I and II follow II. No rain is tree.
(b) Only I, II and III follow III. Some rains are fruits.
(c) Only either I or III, and II follow IV. Some rains are trees.
(d) Only either I or III, and IV follow (a) Only either II or III, and I follow
(e) None of these (b) None follows
69. Statements: Some bricks are trees. All trees (c) Only either II or IV, and III follow
are pens. All pens are boats. (d) All follow
Conclusions: (e) None of these
I. Some boats are bricks 73. Statements: Some papers are cats. All cats
II. Some pens are bricks are bats. No bat is horse.
III. Some trees are bricks. Conclusions:
IV. Some bricks are boats. I. Some papers are horses.
(a) Only I and II follow II. No horse is cat.
(b) Only III and IV follow III. Some bats are papers.
(c) None follows IV. All papers are bats.
(d) All follow (a) Only I and II follow
(e) None of these (b) Only II and III follow
70. Statements: All doors are buses. All buses are (c) Only III and IV follow
leaves. No leaf is a flower.
(d) Only I and IV follow
Conclusions :
(e) All follow
I. No flower is a door.
Directions (74-78): In each of the questions
II. No flower is a bus.
III. Some leaves are doors. below are given three statements followed by
IV. Some leaves are buses. two conclusions numbered I and II. You have
(a) None follows to take the given statements to be true even if
(b) Only I and II follow they seem to be at variance from commonly
(c) Only II and III follow known facts. Read the conclusions and then
(d) Only II, III and IV follow decide which of the given conclusions logi-
(e) All follow cally follows from the given statements
71. Statements: No house is school. All colleges disregareding commonly known facts.
are schools. All schools are teachers. Statements:
Conclusions: Give answer (1) if only conclusion I follows.
I. No house is teacher. Give answer (2) if only conclusion II follows.
II. All colleges are teachers. Give answer (3) if either conclusion I or II
192
follows. conclusions numbered. You have to take the
Give answer (4) if neither conclusion I nor II given statements to be true even if they seem
follow. to be at variance from commonly known facts.
Give answer (5) if both conclusions I and II Read all the conclusions and then decide
follow. which of the given conclusions logically fol-
74. Statements: lows from the given statements disregarding
All benches are cots. commonly known facts.
No cot is lamp. 79. Statements:
Some lamps are candels. Some plates are spoons.
Conclusions: All spoons are forks.
I. Some cots are benches. All forks are bowls.
II. Some candeles are cots. Some bowls are untensils.
75. Statements: Conclusions:
Some cats are dogs. I. Some plates are bowls.
All dogs are goats. II. All spoons are bowls.
All goats are walls. III. Some forks are utensils.
Conclusions: (a) Only I follows
I. Some walls are dogs. (b) Only II follows
II. Some walls are cats. (c) Only I and III follow
76. Statements: (d) Only I and II follow
Some buildings are sofas. 80. Statements:
Some sofas are benches. Some books are files.
Some benches are tables. All files are discs.
Conclusions: Some discs are boards.
I. Some tables are sofas. All boards are keys.
II. No table is sofa. Conclusions:
77. Statements: I. Some books are keys.
All rats are bats. II. No book is key.
Some bats are desks. III. Some discs are keys.
All desks are chairs. (a) Only III follows
Conclusions: (b) Only I and III follow
I. Some desks are rats. (c) Either I or II and III follow
II. Some chairs are rats. (d) All follow
78. Statements: 81. Statements:
Some roads are ponds All buses are trains.
All ponds are stores. Some trains are cars.
Some stores are bags. No car is scooter.
Conclusions: All scooter are jeeps.
I. Some bags are ponds Conclusions:
II. Some stores are roads. I. Some cars are buses.
Directions (79-113): In each of the questions II. All jeeps are scooters.
below are given statements followed by three III. No jeep is train.
(a) Only I follows
193
(b) Only II follows (b) Only II and III follow
(c) Only either I or III follows (c) Only either I or IV follow
(d) None follows (d) Only III follows
82. Statements: 85. Statements:
All curtains are pillows. All suns are stars.
Non pillow is mattress. All moons are stars.
Some mattresses are beds. Some planets are suns.
All beds are sofas. Some stars are gases.
Conclusions: Conclusions:
I. No bed is pillow. I. Some stars are planets.
II. Some mattresses are sofas. II. Some suns are gases.
III. Some beds are pillows. III. No moon is a planet.
(a) Only either I or III follows IV. Some gases are moons.
(b) Only II follows (a) None follows
(c) Only II and either I or II forllow (b) Only I follows
(d) Only I and II follow (c) Only II and III follow
(e) All follow (d) Only II and IV follow
83. Statements: 86. Statements:
Some pulses are grains. All books are diaries.
Some grains are sprouts. Some diaries are pens.
All sprouts are nuts. Some pens are drawers.
No fruit is nut. All drawers are chairs.
Conclusions: Conclusions:
I. Some nuts are pulses. I. Some drawers are diaries.
II. Some nuts are grains. II. Some chairs are pens.
III. Non fruit is sprout. III. Some pens are books.
(a) Only II and III follow. IV. Some diaries are books.
(b) Only I and II follow (a) None follows
(c) Only either I or II follows (b) Only II follows
(d) None follows (c) Only II and III follow
84. Statements: (d) Only II and IV follow
Some robots are machines. 87. Statements:
Some computers are both robots and ma- Some buildings are rivers.
chines. Some mountains are both buildings and riv-
Some animals are machines. ers.
Some toys are animals. Some roads are buildings.
Conclusions: All roads are trucks.
I. Some toys are robots Conclusions:
II. Some toys are machines. I. Some mountains are roads.
III. Some animals are computers. II. Somr buildings are trucks.
IV. Some robots are not toys. III. Some rivers are roads.
(a) None follows IV. Some trucks are rivers.

194
(a) None follows No shirt is a blazer.
(b) Only I follows Some blazers are suits.
(c) Only II follows Conclusions:
(d) Only III follows I. Some shirts are umbrellas.
88. Statements: II. Some suits are raincoats.
All tables are round. 92. Statements:
Some hills are round. Some computers are boards.
Some rivers are hills Some boards are chalks.
All riveres are conical. All chalks are bulbs.
Conclusions: No bulb is a tube-light.
I. Some rivers are round. Conclusion:
II. Some hills are conical . I. Some bulbs are computers.
III. Some rivers are both hills and round. II. No chalk is a tube light.
IV. Some tables are conical. 93. Statements:
(a) None follows All doors ae floors.
(b) Only II follows Some floors are tiles.
(c) Only I and III follows All tiles are paints.
(d) Only II and III follow Some paints are stones.
89. Statements: Conclusions:
All sharks are fishes. I. Some floors are paints.
Some fishes are birds. II. Some doors are tiles.
All birds are trees. 94. Statements:
All trees are insects. Some leaves are petals.
Conclusions: Some petals are flowers.
I. Some insects are sharks. All flowers are fruits.
II. Some sharks are trees. Some fruits are nuts.
III. All insects are birds. Conclusions:
IV. Some birds are sharks. I. Some nuts are flowers.
(a) None follows II. No nut is flower.
(b) Only II follows 95. Statements:
(c) Only I and IV follow All pictures are paintings.
(d) Only II and III follow All paintings are photographs.
90. Statements: Some photographs are designs.
Some schools are colleges. Some desigtns are movies.
Some colleges are universities. Conclusions:
All universities are Institutes. I. Some paintings are designs.
All Institutes are classes. II. Some photographs are movies.
Conclusions: 96. Statements:
I. Some colleges are classes . Some tablets are capsules.
II. All universities are clasess All capsules are syrups.
91. Statements: Some syrups are medicines.
Some umbrellas are raincoats. All medicines are powders.
All raincoats are shirts. Conclusions:
195
I. Some syrups are powders. (b) Only I and III follow
II. Some syrups are tablests. (c) Only II and III follow
97. Statements: (d) All follow
Some rooms are flats. 101. Statements:
All flats are building. All pins are bags.
Some buildings are bunglows. All chalks are bags.
All bunglows are apartments. All needles are bags.
Conclusions: Conclusions:
Some flats are bunlows. I. Some needles are pins.
Some apartments are buildings. II. Some chalks are needles.
98. Statements: III. No needle is pin.
All pins are rods. (a) Only I follows
Some rods are chains. (b) Only III follows
All chains are hammers. (c) Only either II or III follows
Conclusions: (d) Only either I or II and III follow
I. Some pins are hammers. 102. Statements:
II. Some hammers are rods. Some buses are trucks.
III. No pin is hammer. Some trucks are boats.
(a) Only I follows No boat is jeep.
(b) Only II follows Conclusions:
(c) Only III follows I. Some jeeps are bus.
(d) Only either I or III and II follow II. Some boats are buses.
99. Statements: III. Some jeeps are trucks.
Some books are papers. (a) None follows
Some papwers are d3esks. (b) Only I follows
some desks are chairs. (c) Only II follows
Conclusions: (d) Only III follows
I. Some books are papers. 103. Statements:
II. Some papers are Desks. All flowers are trees.
III. Some books are chairs. All trees are jungles.
(a) None follows No jungles is hill.
(b) Only I follows Conclusions:
(c) Only II follows I. No flower is hill.
(d) Only III follows II. No tree is hill.
100. Statements: III. Some jungles are flowers.
Some pots are buckets. (a) None flollows
All buckets are tubs. (b) Only I and II follow
All tubs are drums. (c) Only I and III follow
Conclusions: (d) All follow
I. Some drums are pots. 104. Statements:
II. All tubs are buckets. All tables are sofas.
III. Some drums are buckets. All sofas are beds.
(a) Only I and II follow All beds are mats.
196
Conclusions: (d) All I, II and III folloow
I. Some mats are sofas. 108. Statements
II. Some beds are tables. Some rings are bangles.
III. Some mats are tables. Some bangles are beads.
(a) Only I and II follow Some beads are flowers
(b) Only II follows Conclusions
(c) Only II and III follow I. Some flowers are rings.
(d) All follow II. Somr flowers are banglses.
105. Statements III. No ring is flower.
All houses are window. (a) Only I follows
All roads are windows. (b) Only II follows
All toys are windows. (c) Only III follows
Conclusions (d) Only either I or III follows
I. Some toys are houses. 109. Statemnets
II. Some roads are houses. Some bats are dogs.
III. Some roads are toys. All dogs are tigers.
(a) All follow Some tigers are rats.
(b) Only I and II follow Conclusions
(c) Only II and III follow I. Some rats are bats.
(d) None of these II. Some tigers are bates.
106. Statements III. Some rats are dogs
All chairs are fruits. (a) None follows
No fruits is nail. (b) Only I and II follow
Some nails are hammers. (c) Only II and III follow
Conclusions (d) Only I and III follow
I. Some hammers are fruits. 110. Statements
II. Some nails are fruits. some chairs are pens.
III. Some fruits are chairs. All pens are books.
(a) Only I follow Some books are walls
(b) Only III follow Conclusions
(c) Only II follow I. Some walls are pens.
(d) Only II and III follow II. Some books are chairs.
107. Statements III. Some walll are chairs.
All benches are rivers. (a) None folows
All rivers are ponds. (b) Only I follow
Some ponds are hills. (c) Only II follow
Conclusions (d) Only I and II follow
I. Some ponds are benches. 111. Statements:
II. Some hills are benches. Some books are trees.
III. Some rivers are benches. All trees are roads.
(a) Only I and II follow All roads are wheels.
(b) Only I anad III follow Conclusions:
(c) Only II and III follow I. Some wheels are books.
197
II. Some roads are books. 115. Statements:
III. Some wheels are trees. (UPSC-2020)
(a) Only I and II follow 1. All cats are dogs.
(b) Only II and III follow 2. All cats are black.
(c) Only I and III follow Conclusions:
(d) All follow I. All dogs are black.
112. Statements: II. Some dogs are not black.
All stones are rivers. Which of the above conclusion(s) logically
All rivers are cars. follows/follow from the two given statements,
Some cars are trains. commonly known facts?
Conclusions: (a) Only Conclusion I follows.
I. Some trains are stones. (b) Only Conclusion I follows.
II. Some cars are stones. (c) Neither Conclusions I nor II follow.
III. Some trains are rivers. (d) Both Conclusion I and II follow.
(a) None follows 116. Statements:
(b) Only I follows (UPPSC-2021)
(c) Only II follows 1. Some fruits are flowers.
(d) Only III follows 2. No flower is a boat.
113. Statements: 3. All boats are rivers.
All desks are rooms. Conclusions:
Some rooms are halls. I. Some fruits are rivers.
All halls are leaves. II. Some rivers are boats.
Conclusions: III. Some rivers are fruits.
I. Some leaves are desks. IV. Some flowers are fruits.
II. Some halls are desks. (a) Both I and III follows.
III. Some leaves are rooms. (b) Both II and III follows.
(a) None follows (c) Both II and IV follows.
(b) Only I follows (d) None of these
(c) Only II follows 117. Statements:
(d) Only III follows (CGPSC-2022)
114. Statements: 1. All cities are districts.
(UPSC-2021) 2. No city is state.
1. Some radios are mobiles. 3. Only country is state.
2. All mobiles are computers. Conclusions:
3. Some computers are watches. I. All cities can be state.
Conclusions: II. Some states can be district.
I. Certainly some radios are watches. III. All countries can be district.
II. Certainly some mobiles are watches. IV. No country is city.
Which one of the following is correct? (a) Both conclusions II and III follows
(a) Only Conclusion I follows (b) All conclusions follows
(b) Only Conclusion I follows (c) None of the conclusion follows
(c) Both Conclusions I and II follow (d) Only conclusion II follows
(d) Neither Conclusion I nor II follow
198
118. Statements:
(UPSC-2022)
Answer Sheet
1. All pens are books. 1. (a) 31. (c) 61. (c) 91. (a)
2. No chair is a pen. 2. (e) 32. (d) 62. (e) 92. (b)
Conclusions: 3. (b) 33. (d) 63. (c) 93. (a)
I. All chairs are books. 4. (e) 34. (a) 64. (a) 94. (c)
II. Some chairs are pens. 5. (d) 35. (a) 65. (a) 95. (d)
III. All books are chairs.
6. (d) 36. (a) 66. (a) 96. (d)
IV. No chair is a book.
7. (b) 37. (d) 67. (c) 97. (b)
Which one of the following is correct?
8. (d) 38. (d) 68. (d) 98. (d)
(a) Only Conclusion I follows
(b) Only Conclusion II follows 9. (c) 39. (a) 69. (e) 99. (a)
(c) Both Conclusions III and IV follow 10. (d) 40. (b) 70. (d) 100. (b)
(d) None of the Conclusions follows 11. (c) 41. (b) 71. (e) 101. (c)
119. Statements: 12. (b) 42. (d) 72. (c) 102. (a)
(UPSC-2021) 13. (d) 43. (c) 73. (c) 103. (d)
1. Some cats are almirahs. 14. (e) 44. (a) 74. (a) 104. (d)
2. Some almirahs are chairs. 15. (a) 45. (c) 75. (d) 105. (d)
3. All chairs are tables. 16. (d) 46. (d) 76. (c) 106. (b)
Conclusions: 17. (e) 47. (c) 77. (d) 107. (b)
I. Certainly some almirahs are tables. 108. (d)
18. (d) 48. (d) 78. (b)
II. Some cats may not be chairs. 109. (a)
19. (c) 49. (b) 79. (d)
Which one of the following is correct?
20. (b) 50. (b) 80. (c) 110. (c)
(a) Only Conclusion I follows
21. (e) 51. (d) 81. (d) 111. (d)
(b) Only Conclusion II follows
22. (d) 52. (a) 82. (c) 112. (c)
(c) Both Conclusions I and II follow
23. (e) 53. (a) 83. (a) 113. (d)
(d) Neither Conclusions I nor II follows
120. Statements: 24. (e) 54. (d) 84. (c) 114. (d)
(UPSC-2021) 25. (d) 55. (b) 85. (b) 115. (c)
1. Some greens are blues. 26. (e) 56. (c) 86. (d) 116. (c)
2. Some blues are blacks. 27. (b) 57. (b) 87. (c) 117. (c)
Conclusions: 28. (d) 58. (c) 88. (b) 118. (d)
I. Some greens are blacks. 29. (c) 59. (a) 89. (a) 119. (c)
II. No green is black. 30. (a) 60. (c) 90. (d) 120. (d)
III. All greens are blacks.
IV. All blacks are greens.
Which one of the following is correct?
(a) Both Conclusions I and II follow
(b) Both Conclusions II and III follow
(c) Both Conclusions III and IV follow
(d) None follows

199
26
1. Consider the following three statements: (c) All those who take part in photo-contests
1. Only students can participate in the race. are members of the birdwatcher's club.
2. Some participants in the race are girls. (d) No conclusion can be drawn.
3. All girl participants in the race are invited 4. During the last summer vacation, Ankit went
for coaching. to a summer camp where he took part in hik-
Which one of the following conclusions can ing, swimining and boating. This summer, he
be drawn from the above statements? is looking forward to a music camp where he
(UPSC-2011) hopes to sing, dance and learn to play the gui-
(a) All participant in the race invited for tar.
coaching Based on the above information, four conclu-
(b) All students are invited for coaching. sions, as given below, have been made. Which
(c) All participant in the race are students. one of these logically follows from the infor-
(d) None of the statement (a), (b) and (c) mation given above?
given above is correct. (UPSC-2012)
2. Consider the following statements: (a) Ankit's parents want him to play. the gui-
1. All artists are whimsical. tar,
2. Some artists are drug addicts. (b) Ankit prefers music to outdoor activities.
3. Frustrated people are prone to become (c) Ankit goes to some type of camp every
drug addicts. summer.
From the above three statements it may be con- (d) Ankit likes to sing and dance.
cluded that 5. Ten new TV shows started in January-5
(UPSC-2012) sitcoms, 3 drama and 2 news magazines. By
(a) Artists are frustrated. April, only seven of the new shows were still
(b) Some drug addicts are whimsical. on, five of them being sitcoms
(c) All frustrated people are drug addicts. Based on the above information, four conclu-
(d) Whimsical people are generally frus- sions, as given below, have been made. Which
trated. one of these logically follows from the infor-
3. Examine the following statements. mation given above?
1. Only those who have a pair of binoculars (UPSC-2012)
can become the members of the (a) Only one news magazine show is still on.
birdwatcher's club. (b) Only one of the drama shows is still on.
2. Some members of the birdwatcher's club (c) At least one discontinued show was a
have cameras. drama.
3. Those members who have cameras can (d) Viewers prefer sitcoms over drama.
take part in photo-contests. 6. Consider the following statement: "Though
Which of the following conclusions can be quite expensive, television is a luxury item,
drawn from the above statements? as one can learn many things through televi-
(UPSC-2012) sion."
(a) All those who have a pair of binoculars Which one of the following is a valid infer-
are members of the birdwatcher's club. ence from the above statement?
(b) All members of the birdwatcher's club (UPSC-2012)
have a pair of binoculars. (a) All expensive things are regarded as

200
luxury. ros are parked here.
(b) All essential things for learning are not (c) Cars other than X-brand cannot have ra-
luxury. dial tyres.
(c) Television is essential for learning. (d) Most of the X-brand cars are manufac-
(d) Television is not a luxury item. tured before 1986.
7. Examine the following statements: 10. Consider the following statement:
1. I watch TV only if I am bored. The Third World War, if it ever starts, will end
2. I am never bored when I have my very quickly with the possible end of civiliza-
brother's company. tion. It is only the misuse of nuclear power
3. Whenever I go to the theatre I take my which will trigger it.
brother along. Based on the above statement, which one of
Which one of the following conclusions is the following inferences is correct?
valid in the context of the above statements? (UPSC-2012)
(UPSC-2012) (a) Nuclear power will be used in the Third
(a) If I am bored, I watch TV. World War.
(b) If I am bored, I seek my brother's com- (b) There will be no civilization left after the
pany. Third World War.
(c) If I am not with my brother, then I watch (c) The growth of nuclear power will destroy
TV, civilization in the long run.
(d) If I am not bored, I do not watch TV. (d) The Third World War will not take place.
8. Examine the following statements: 11. Consider the following statements:
1. None but students are members of the 1. All machines consume energy.
club. 2. Electricity provides energy.
2. Some members of the club are married 3. Electrically operated machines cheap to
persons. maintain.
3. All married persons are invited for dance. 4. Electrically operated machines do not
Which one of the following conclusions can cause pollution.
be drawn from the above statements? Which one of the following inferences can be
(UPSC-2012) drawn from the above statements?
(a) All students are invited for dance. (UPSC-2012)
(b) All married students of the club are in- (a) All machines are run by electric energy.
vited for dance. (b) There is no form of energy other than
(c) All-members of the club are married per- electricity.
sons. (c) Most machines are operated on electric
(d) None of the above conclusions can be energy.
drawn: (d) Electrically operated machines are pref-
9. Consider the following statements: erable to use.
1. All X-brand cars parked here are white. 12. Examine the following statements:
2. Some of them have radial tyres. 1. Apart from the rich, no one can afford
3. All X-brand cars manufactured after 1986 air travel.
have radial tyres. 2. Some of those who travel by air fall ill.
4. All cars are not X-brand. 3. Some of those who fall ill require treat-
Which one of the following conclusions can ment.
be drawn from the above statements? Which one of the following conclusions can
(UPSC-2012) be drawn from the above statements? Are
(a) Only white cars are parked here. (UPSC-2012)
(b) Some white X-brand cars with radial ty- (a) All rich people travel by air.

201
(b) Those who travel by air fall ill. except one would not."
(c) All rich people fall ill. The writer wants to say that if prices of all
(d) All people who travel by air are rich. things were doubled
13. Consider the following statements: (UPSC-2014)
(i) A primary group is relatively smaller in (a) the values of all things would remain con-
size. stant.
(ii) Primacy is an essential characteristic of (b) the values of the things sold would be
a primary group. doubled.
(iii) A family may be an example of a primary (c) the values of the things bought would be
group. halved.
In the light of the above statements, which one (d) the value of money only would be halved.
of the following is true? 17. "Liberty, therefore, is never real unless the
(a) All families are primary groups. Government can be called to account when it
(b) All primary groups are families. invades rights."
(c) A group of smaller size is always a pri- Which one of the following is the best justifi-
mary group. cation of the above statement?
(d) Members of a primary group know each (UPSC-2014)
other immediately. (a) In the realisation that the government can
14. The number of deaths among the army per- be brought to book in a court of law
sonnel is 8 in 1000, but among the civilian (b) In identifying a man as a political unit in
population it is 20 per 1000. Which one of the a way which distinguishes him from other
following inferences can be drawn from this citizens
statement? (c) In a decentralized society wherein the ba-
(UPSC-2014) sic needs of men can find satisfaction
(a) It is better to join the army. (d) In the understanding that liberty and re-
(b) The relationship is fortuitous. straints are complementary
(c) Quality of Life Index is very high within 18. Consider the following statements:
the armed forces. 1. Some claim to have seen UFOs (Uniden-
(d) The groups cannot be compared due to tified Flying Objects).
their heterogeneity. 2. Life on other heavenly bodies is consid-
15. Given the statement: "Buses are the cause of ered to be a possibility.
more accidents than cars, and trucks cause 3. Voyage to space is now an established
fewer accidents than buses", which of the fol- fact.
lowing conclusions can we draw ? From the above statements, it may be con-
(UPSC-2014) cluded that
(a) There are more buses on the road than (UPSC-2015)
trucks. (a) UFOs are heavenly bodies
(b) Car drivers are more careful than bus (b) UFOs are sent from other heavenly bod-
drivers. ies
(c) Truck drivers are more skilled than ei- (c) Some living species in other heavenly
ther car or bus drivers. bodies are more intelligent than man
(d) None of the above (d) Nothing definite can be said about the
16. "Price is not the same thing as value. Suppose UFOs
that on a day the price of everything viz., coal, 19. Examine the following statements:
bread, postage stamps, a day's labour, the rent 1. Lady's finger is tastier than cabbage.
of houses, etc. were to double. Prices then 2. Cauliflower is tastier than lady's finger.
would certainly rise, but values of all things 3. Cabbage is not tastier than peas.

202
The conclusion that can be drawn from these (c) every athlete who eats a well-balanced
statements is that diet is a good athlete.
(UPSC-2015) (d) all athletes who want to win are good ath-
(a) peas are as tasty as lady's finger. letes.
(b) peas are as tasty as cauliflower and lady's 23. There were 50 faculty members comprising
finger. 30 males and the rest females.
(c) cabbage is the least tasty of the four veg- No male faculty member knew music, but
etables. many of the female faculty members did. The
(d) cauliflower is tastier than cabbage. Head of the institution invited six faculty
20. Consider the following statements: members to a tea party by draw of lots. At the
1. A man had a wife, two sons and two party it was discovered that no member knew
daughters in his family. music. The conclusion is that:
2. The daughters were invited to a feast and (UPSC-2016)
the male members of the family went out (a) the party comprised male faculty mem-
to take part in a picnic. bers only
3. The man's father did not return from his (b) the party comprised only those female
work. faculty members who could not give ren-
Which of the following statements is true? derings in music
(UPSC-2015) (c) the party comprised both male and female
(a) Only the man's wife was left at home. faculty members
(b) It is likely that the man's wife was left at (d) nothing can be said about the gender com-
home. position of the party
(c) None was left at home. 24. Consider the following statements:
(d) More than one person was left at home. 1. The rate of population growth is increas-
21. Geeta : Naresh has become a better boxer since ing in the country
he started meditation. 2. The death rate is declining faster in the
Radha: Impossible. A boxer's most important country compared to birth rate
asset is his aggressiveness. 3. The birth rate is declining faster in the
Radha's statement reflects her belief that country compared to death rate
(UPSC-2015) 4. Rural-urban migration is taking place
(a) meditation tends to make a person less regularly in the country
aggressive. Which one of the following conclusions may
(b) meditation has little or no effect on the be true in the light of the above facts?
person who practises it. (UPSC-2016)
(c) Naresh was a poor boxer earlier because (a) The rate of population growth is increas-
he was not aggressive enough. ing due to rural-urban migration
(d) Naresh would not have taken to medita- (b) The rate of population growth is increas-
tion as he was a boxer. ing due to decline in death rate only
22. All good athletes want to win and all athletes (c) The rate of population growth is increas-
who want to win eat a well-balanced diet; ing due to increase in birth rate only
therefore all athletes who do not eat a well- (d) The rate of population growth is increas-
balanced diet are bad athletes. ing due to faster decline in death rate than
The best conclusion from this statement is that in birth rate
(UPSC-2015) 25. Consider the following statement:
(a) no bad athlete wants to win. "We shall go either for a picnic or for trek-
(b) no athlete who does not eat a well-bal- king".
anced diet is a good athlete. Which of the following, if true, would falsify

203
this claim?. I. Natural gifts need nurturing and care.
(UPSC-2016) II. Even though one's voice is not good, one
(a) We go for a picnic but not for trekking can keep practising.
(b) Activities such as picnic and trekking are Which one of the following is correct in re-
encouraged by the health authorities spect of the above statement and conclusions?
(c) We go for trekking and not for picnic (UPSC-2017)
(d) We do not go either for picnic or for trek- (a) Only conclusion I follows from the state-
king ment.
26. Directions for the following 2 (two) items: (b) Only conclusion II follows from the state-
Consider the given information and answer ment.
the two items that follow. (c) Either conclusion I or conclusion II fol-
No supporters of 'party X', who knew Z and lows from the statement.
supported his campaign strategy, agreed for (d) Neither conclusion I conclusion II follows
the alliance with 'party Y'; but some of them from the statement.
had friends in 'party Y'. 29. Examine the following statements-
With reference to the above information, 1. All the members of Mohan's family are
which one among the following statements honest.
must be true? 2. Some members of Mohan's family are not
(UPSC-2017) in employment.
(a) Some supporters of party y' did not agree 3. The person engaged in some employment
for the alliance with the 'party X'. is not honest.
(b) There is at least one supporter of 'party 4. Some honest persons are not in employ-
Y' who knew some supporters of 'party ment.
X' as a friend. Which one of the following conclusions fol-
(c) No supporters of 'party supported Z's lows from the above statements?
campaign strategy. (UPSC-1998)
(d) No supporters of 'party X' knew Z. (a) All the members of Mohan's family are
27. With reference to the above information, con- employed.
sider the following statements: (b) The employed members of Mohan's fam-
1. Some supporters of 'party X' knew Z. ily are honest.
2. Some supporters of party X', who op- (c) The honest members of Mohan's family
posed Z's campaign strategy. knew Z. are not in employment.
3. No supporters of 'party X' supported Z's (d) The members of Mohan's family engaged
campaign strategy. in employment are not honest.
Which of the statements given above is/are 30. "Rights are some beneficial conditions of so-
not correct? cial welfare, indispensable for the real devel-
(UPSC-2017) opment of the citizen."
(a) 1 only In the light of this statement, which of the fol-
(b) 2 and 3 only lowing are the correct interpretations of rights?
(c) 3 only (UPSC-2017)
(d) 1, 2 and 3 (a) The object of rights is individual welfare
28. Consider the following: only.
Statement: (b) The object of rights is social welfare only.
Good voice is a natural gift but one has to keep (c) The purpose of rights is individual and
practising to improve and excel well in the social welfare.
field of music. (d) the object of rights without social
Conclusions: welfare“Personal welfare.

204
31. Consider the given statement and the two con-
clusions that can be expected from it-
Statements: Morning walk is good for health.
Conclusion: Answer Sheet
I. All healthy people take morning walk.
1. (c) 18. (d)
II. Morning walk is essential to maintain
good health. 2. (b) 19. (c)
Which is/are the valid conclusion(s)?
3. (b) 20. (b)
(UPSC-2017)
(a) I only 4. (d) 21. (a)
(b) II only
5. (c) 22. (d)
(c) Both I and II
(d) neither I nor II 6. (b) 23. (d)
32. In a school, 60% of the students play cricket.
7. (d) 24. (d)
The student who does not play cricket plays
football. Every football player has a two 8. (d) 25. (d)
wheeler.
Which of the following conclusions cannot be
9. (b) 26. (b)
drawn from the above data? 10. (a) 27. (b)
(UPSC-2019)
1. 60% The student does not have a two
11. (d) 28. (a)
wheeler. 12. (d) 29. (b)
2. No cricketer has a two wheeler.
3. Cricket players do not play football.
13. (d) 30. (c)
Select the correct answer using the code given 14. (d) 31. (b)
below -
(a) 1 and 2 only 15. (d) 32. (d)
(b) 2 and 3 only 16. (d) 33. (a)
(c) 1 and 3 only
(d) 1, 2 and 3 17. (a) *******
33. A statement is given followed by two Con-
clusions I and II. Decide which of the given
conclusions follows logically from the given
statement.
Statement Fortune favours the brave.
Conclusions
(UKPSC-2022)
I. Risks are necessary for success.
II. Cowards die many times before their
death.
(a) Only I
(b) Only II
(c) Both I and II
(d) Neither I nor II

205
27
Direction : Given the following questions II. No. Such polls are conducted all over the
Statement followed by two Arguments world.
numbered I and II. An Arguments is something 4. Statement: Should the vehicles older than
supposed or taken for granted. You have to 15 years be rejected in metros in India?
consider the statement and the following
Arguments:
Arguments and decide which of the
Arguments is true in the statement. I. Yes This is a significant step to lower
down the pollution level in metros.
Give (a) answer if only Arguments I is true.
II. No. It will be very difficult for vehicle
Give (b) answer if only Arguments II is owners to shift to other parts in conutry
true. because they will not get suitable job for
Give (c) answer if both Arguments I and II their very existence.
are true. 5. Statement: Should there be a restriction on
Give (d) answer if neither Arguments I true the migration of people from one state to
another state in India?
II and true.
Arguments:
1. Statement: Should there be more than one
I. No. Any Indian citizen has a basic right
High Courts in each state in India?
to stay at any place of his/her choice and
Arguments: hence they cannot be stopped.
I. No.This will be a sheer wastage of II. Yes. This is the way to effect an equitable
taxpayers' money. distribution of resources across the states
II. Yes, This will help to reduce the backlog in India.
of cases pending for a very long time. 6. Statement: Should the railways in India be
2. Statement: Should there be only one rate of privatized in a phased manner like other public
interest for term deposits of varying durations sector enterprises?
in banks? Arguments:
Arguments: I. Yes. This is the only way to bring in
competitiveness and provide better
I. No. People will refrain from keeping
services to the public.
money for longer duration resulting into
II. No. This will pose a threat to national
reduction of liquidity level of banks.
security of our country as multinationals
II. Yes. This will be much simple for the will enter into the fray.
common people and they may be
7. Statement: Should there be a complete ban
encouraged to keep more money in banks.
on use of all types of chemical pesticides in
3. Statement : Should the opinion polls India?
predicting outcome of elections before the
Arguments:
elections be banned in India?
I. No. The pests will destroy all the crops
Arguments:
and the farmers will have nothing to
I. Yes. This may affect the voters'mind and
harvest.
may affect the outcome.

206
II. Yes. The chemical pesticides used in I. No. This is not advisable as the jobs
agriculture pollute the water underground should be offered to only deserving
and this has become a serious health candidates without any reservation for a
hazared. particular group.
8. Statement : Should the prestigious people II. Yes. This will help to reduce the growing
who have committed crime unknowingly be population in India as the parents will be
met with special treatment? encouraged to adopt single child norm.
Arguments: 13. Statement: Should there be complete ban on
Indian professionals seeking jobs elsewhere
I. Yes. The prestingious people do not
after getting their education in India?
commit crime interntionally
Arguments:
II. No. It is our policy that everybody is equal
I. Yes. This is the only way to sustain
before the law.
present rate of technological development
9. Statement: Should all the unauthorized in India.
structures in the city be demolished?
II. No. The Indians settled abroad send huge
Arguments: amount of foreign exchange and this
I. No. Where will the people residing in constitutes a significant part of foreing
such houses live? exchange reserve.
II.Yes. This will give a clear message to III. No. The practical knowledge gained by
general public and they will refrain from Indians by working in other countries
construction unauthorized buildings. help Indian to develop its economy.
10. Statement : Should Education be brought (a) None is true
under the control of the Central Government (b) All are true
like defence? (c) Only I and II are true
Arguments: (d) Only III is true
I. No Education is a state subject and it (e) Only II and III are true
should remain with the state. 14. Statement: Should religion be taught in our
II. Yes. This is the only way to establish schools?
uniformity in growth of education across Arguments:
the states. I. No. we are a secular country.
11. Statement: II. Yes. Teaching religion helps inculcate
Should Agriculture in Rural India be moral values among children.
Mechanized? III. No. How can one dream of such a step
Arguments: when we want the young generation to
fulfill its role in the 21st century.
I. Yes. It would lead to higher Production.
(a) All are true
II. No. many Villagers would be left
(b) None is true
Unemployed.
(c) Only I is true
12. Statement: Should there be reservation in
Government jobs for candidates from single (d) Only II is true
child family? (e) Only I and III are true
Arguments:

207
15. Statement: Should there be reservation for (e) None of these
jobs in the organizations in the private sector17. Statement: Should mercy death be legalized,
also as in the public sector undertakings in i.e., all those who are suffering form terminal
India? diseases be allowed to end their lives if they
Arguments: so desire?
I. Yes. This would give more opportunities Arguments:
of development to the weaker sections of I. No. Nobody should be allowed to end his/
the society and thus help reduce the gape her life at his/her will as this goes against
the basic tenets of humanity.
between the affluent and the downt
rodden in India. II. Yes.Patients undergoing terrible suffering
and having absolutely no chance of
II. No. The private sector does not get any recovery should be liberated from
government assistance and therefore they suffering through mercy death.
should not be saddled with such policies. III. No Even mercy death is a sort of killing
III. No. Nowhere else in the world such a and killing can never be legalized.
practice is being followed. (a) None is true
IV. No. The management of the private sector (b) Only I and II are true
undertaken would not agree to such (c) Only II and III are true
compulsions. (d) Only I and III are true
(a) Only I is true (e) All ae true
(b) Only I and II are true 18. Statement: Should the public sector
(c) Only I, II and IV are true undertakings be allowed to adopt hire and fire
policy?
(d) Only I and IV are true
(e) All are true Arguments:
16. Statement: Should the income generated out I. Yes. This will help the public sector
of agricultural activities be taxed? undertakings to get rid of non-performing
Arguments: employees and reward the performing
employees.
I. No. Farmers are otherwise suffering form
natural calamities and low yield coupled II. No. This will give an unjust handle to the
with low procurement price and their management and they may use it
incomeshould not be taxed. indiscriminately.
II. Yes. Majority of the population is III. Yes. This will help increase the level of
dependent on agriculture and hence their efficiency of these organizations and
income should be taxed to augment the these will become profitable
resources. establishments.
III. Yes. Many big farmers earn much more (a) None is true
than the majority of the service earners
(b) Only I and II are true
and they should be taxed to remove the
disparity. (c) Only II and III are true
(a) Only I is true (d) Only I and III are true
(b) Only I and II are true (e) All are true
(c) Only II and III are true 19. Statement: Should there be a total ban on
(d) All are true tobacco products and smoking in India?

208
Arguments: (d) if neither I nor II is true and
I. Yes. It is wrong to smoke away millions 21. Statement : Should there be only a uniform
of money. rate of income tax irrespective of the level of
II. No. It will throw thousands of workers income?
in the tobacco industry out of Arguments :
employment.
I. Yes, this will substantially reduce the
III. No. The government will lose huge work of the officials of the income tax
amount of money as it will not earn by department.
way of taxes on these products.
II. No, this will reduce Govt, tax collection
(a) None is true
to a large extent.
(b) Only I and II are true
22. Statement : Should be opinion polls
(c) Only II is true predicting outcome of elections before the
(d) Only II and III are true elections be banned in India?
(e) All are true Arguments :
20. Statement: Should all the school teachers be I. Yes. This may affect the voter’s mind and
debarred form giving private tuitions? may affect the outcome.
Arguments: II. No. Such polls are conducted all over the
I. No. The needy students will be deprived world.
of the expertise of these teachers. 23. Statement : Should there be compulsory
II. Yes. This is an injustive to the medical examination of both the man and the
unemployed educated people who can woman before they marry each other ?
earn their liveing by giving tuitions. Arguments :
III. Yes. Only then the quality of teaching in I. No. This is an intrusion to the privacy of
schools will improve. an individual and hence cannot be
IV. Yes. Now salary of these teachers is tolerated.
reasonable. II. Yes. This will substantially reduce the
(a) Only I and III are true risk of giving birth to children with
serious ailments.
(b) Only I, II and III are true
24. Statement : Should computer education be
(c) Only III and IV are true
made a compulsory subject at school level by
(d) Only II, III and IV are true the Government?
(e) None of these Arguments :
Direction (21-42) : Each question given I. No, it may be difficult to teach computers
below consists of a statemesnt, followed by in the schools located in rural parts of the
two arguments numbered I and II. You have country.
to decide which of the arguments is a 'true'
II. Yes, it is difficult to function without
argument and which is a 'weak' argument. Give
computer in the era of modernisation.
answer
(a) if only argument I is true; 25. Statement : Will introduction of quota for
(b) if only argument II is true; backward Classes in the institutes for higher
(c) if both I and II are true; studies be the right move to do social justice?

209
Arguments : logging and hence it is necessary to ban
I. No, with this very few seats will be left it.
for unreserved classes which is unfair to 29. Statement : Should there be reservation in
the deserving candidates and particularly Government jobs for candidates from single
to those who are economically child family?
back-ward. Arguments :
II. Yes, it is necessary to give them I. No. This is not advisable as the jobs
opportunities to rise upto the level of should be offered to only deserving
general category candidates. candidates without any reservation for a
26. Statement : Should it be made compulsory particular group.
for all the private sector organisations to II. Yes. This will help reduce the growing
reserve quota for socially backward classes? population in India as the parents will be
encouraged to adopt single child norm.
Arguments :
30. Statement : Should India create a huge oil
I. No, the private sector should not be
reserve like some Western countries to face
governed by the Government rules.
difficult situations in future?
II. Yes, private sector organisations should
Arguments :
also contribute in upliftment of socially
I. No. There is no need to block huge
backward classes.
amount of foreign exchange and keep
27. Statement : Should the women be advised not money idle.
to travel alone at night in view of the
II. Yes. This will help India withstand
increasing incidences of rape and sexual
shocks of sudden rise in oil prices due to
abuse?
unforeseen circumstances.
Arguments :
31. Statement : Should non-vegetarian food be
I. No, instead the Government should take totally banned in our country ?
measures to control such incidences. Arguments :
II. Yes, it is difficult even for the police I. Yes. It is expensive and therefore it is
department to control such cases. beyond the means of most people in our
28. Statement : Should there be a total ban on country.
use of plastic bags? II. No. Nothing should be banned in a
Arguments : democratic country like ours.
I. No, instead the thickness of plastic bags, 32. Statement : Should all the drugs patented and
which can be used without much damage manufactured in Western countries be first
to the environment, should be specified. tried out on sample basis before giving licence
for sale to general public in India?
II. Yes, use of plastic bags causes various
problems like water pollution and water- Arguments :
I. Yes. Many such drugs require different
210
doses and duration for Indian population Arguments :
and hence it is necessary. I. Yes, this is the only way to save people
II. No. This is just not feasible and hence from suffering from cancer.
cannot be implemented. II. No, this will adversely affect the sale of
33. Statement : Should ‘education’ be brought tobacco products.
under the control of the Central Government 38. Statement : Should all refugees, who make
like defence? unauthorized entry into a country, be forced
Arguments : to go back to their homeland?
I. No. Education is a state subject and it Arguments :
should remain with the state.
I. Yes, they make their colonies and occupy
II. Yes. This is the only way to establish a lot of land.
uniformity in growth of education across
II. No, they leave their homes because of
the states.
hunger or some terror and on human
34. Statement : Should military service be made grounds, should not be forced to go back.
compulsory in our country ?
39. Statement : Should there be only one rate of
Arguments : interest for term deposits of varying durations
I. No. It is against the policy of non- in banks?
violence. Arguments :
II. Yes. Every citizen should protect his I. No, people will refrain money for longer
country. duration resulting into reduction of
35. Statement : Should cricket replace hockey as liquidity level of banks.
the national sport of India?
II. Yes, this will be much simpler for the
Arguments : common people and they may be
I. Yes, the performance of the hockey team encouraged to keep more money in
has been dismal for the last few years. banks.
II. No, cricket is the national sport of 40. Statement : Should the fees of all the private
Australia and no two countries must have professional colleges be made equal to those
the same national sport. of the government professional colleges?
36. Statement : Should the sex determination test Arguments :
during pregnancy be completely banned? I. No, the private colleges need additional
Arguments : funds to maintain quality of education.
I. Yes, this leads to indiscriminate female II. Yes, otherwise a large number of
foeticide and eventually will lead to social meritorious students will not be able to
imbalance. study in these colleges for exorbitantly
II. No, people have a right to know about high fees.
their unborn child. 41. Statement : Should there be a restriction on
37. Statement : Should there be a complete ban number of ministers in each cabinet in India?
on advertising of tobacco products in all Arguments :
media?

211
I. Yes, as a result of this a lot, of money Arguments :
will be saves and the same can be used I. No. Farmers are otherwise suffering from
in developmental programmes. natural calamities and low yield coupled
II. No, there should not be such restrictions with low procurement price and their
on democratically elected representatives income should not be taxed.
and it should be left to the judgement of II. Yes. Majority of the population is
the leader of the council of ministers. dependent on agriculture and hence their
42. Statement : Should the press in India be given income should be taxed to augment the
full freedom? resources.

Arguments : III. Yes. Many big farmers earn much more


than the majority of the service earners
I. Yes, because only then people will
and they should be taxed to remove the
become politically enlightened.
disparity.
II. No, because full freedom to press will (a) Only I is true
create problems.
(b) Only I and II are true
Direction (43-47): Each question below is
(c) Only II and III are true
followed by three Arguments numbered I, IIand
(d) All are true
III. You have to decide which of the arguments is a
‘true’ argument and whichisa ‘weak’ argument. (e) None of these
45. Statement : Should smoking cigarettes and
43. Statement : Should there be complete ban on
drinking alcohol by the actors be completely
setting up of thermal power plants in India ?
banned in the movies in India?
Arguments :
Arguments :
I. Yes, this is the only way to arrest further
I. Yes, this will significantly reduce the
addition to environmental pollution.
trend of smoking cigarettes and drinking
II. No, there is a huge shortage of electricity alcohol among the youth in India.
in most parts of the country and hence II. No, there should be no such ban on the
generation of electricity needs to be creative pursuits of the filmmaker.
augmented.
III. No, the films portray the society and
III. No, many developed countries continue hence such scenes should be an integral
to set up thermal power plants in their part of the movie if the storyline demands
countries. so.
(a) None is true (a) None is true
(b) Only I is true (b) Only I and II are true
(c) Only II is true (c) Only II and III are true
(d) Only III is true (d) Only I and III are true
(e) Only either I or II is true (e) All are true Oriental Bank of
44. Statement : Should the income generated out 46. Statement : Should the rule of wearing helmet
of agricultural activities be taxed ? for both driver and ; pillion-rider while riding
a motorbike be enforced strictly?
212
Arguments : (UPPSC-2021)
I. ‘ Yes, it is rule and rules should be Arguments :
followed strictly by all. I. Yes, this will put an put an end to
II. No, each individual knows how to protect unhealthy competition among different
his own life and it should be left to his companies.
discretion. II. No the travellers should be given a wider
III. No, it does not ensure safety as only the choice of transport system.
head is protected and the rest of the body 49. Statement : Our country should enhance
is not. exports even when there is a shortage for
IV. Yes, it is necessary as head, being the internal consumption.
most sensitive organ, is protected by the (MPPSC-2021)
helmet. Arguments :
(a) Onfy I and IV are true I. Yes, we need foreign exchange to import
(b) Only II and IV are true things like oil.
(c) Only I and III are true II. No, it will harm the internal consumers.
(d) All are true 50. Statement : In crowded public places like
(e) Only I, II and III are true market, buses, trains, one should cover his/
47. Statement : National Human Rights her mouth while coughing and sneezing.
Commission (NHRC) of India is responsible (MPPSC-2021)
for the protection and promotion of Human Arguments :
Rights, defined by the act as rights relating to I. Germs of deadly diseases such as
life, liberty, equality and dignity of the tuberculosis spread through droplets of
individual guaranted by the Constitution or cough/sneeze.
embodied in the International Covenants.
II. Chances of getting infected from deadly
(MPPSC-2021) diseases such as tuberculosis is high, if
Arguments : you visit crowded public places
I. On the violation of Human Rights, frequently.
NHRC can initiate an inquiry on their
own or against a petition filed by a victim.
II. NHRC promotes and undertakes research
about the Human Right issues.
(a) Argument I is strong
(b) Argument II is strong
(c) Arguments I and II are strong
(d) Neither Argument I nor II is strong
48. Statement : Should only one company be
allowed to run the transport system in a big
city?

213
Answer Sheet
1. (b) 26. (b)
2. (d) 27. (d)
3. (a) 28. (e)
4. (a) 29. (d)
5. (a) 30. (b)
6. (d) 31. (b)
7. (c) 32. (a)
8. (b) 33. (a)
9. (c) 34. (b)
10. (d) 35. (a)
11. (a) 36. (a)
12. (b) 37. (d)
13. (e) 38. (e)
14. (d) 39. (d)
15. (a) 40. (a)
16. (e) 41. (a)
17. (e) 42. (d)
18. (d) 43. (c)
19. (d) 44. (c)
20. (e) 45. (d)
21. (b) 46. (a)
22. (a) 47. (a)
23. (b) 48. (b)
24. (b) 49. (d)
25. (e) 50. (d)

214
28
Direction : Given the following questions II. There is an awareness to a great extent
Statement followed by two Assumption numbered about Yoga exercises among people.
I and II. An Assumption is something supposed or 4. Statement : As a measure to avoid occurrence
taken for granted. You have to consider the of the epidemics due to monsoon the civic
statement and the following Assumption and decide authorities have organised free vaccination
which of the Assumption is implict in the statement. camps all over the city.
Give (a) answer if only Assumption I is Assumptions :
implict. I. There may be a good response to the
Give (b) answer if only Assumption II is camps organized by civic authorities.
implict. II. People are generally aware about the need
Give (c) answer if both Assumption I and II for vaccination.
are implict. 5. Statement : The govt, has recently announced
Give (d) answer if neither Assumption I an incentive package for setting up new
implict II and implict. business ventures in the rural areas and
promised uninterrupted power supply to all
1. Statement : A major retail store announced
the units.
thirty percent reduction on all food items
during the weekend. Assumptions :
Assumptions : I. The govt, may be able to supply adequate
power to all such units.
I. People may still prefer buying food items
II. People living in the rural areas may
from other stores.
welcome the govt, decision.
II. A large number of customers may visit
6. Statement : The college administration has
the retail store and buy food items.
instructed all the students to stop using cell
2. Statement : The railway authority has phones within the college premises.
rescheduled the departure time of many long- Assumptions :
distance trains and put up the revised timing
I. The students may stop using cell phones
on its website.
in the college premises.
Assumptions :
II. The students may continue to use cell
I. The passengers may note the change in phones in the college premises.
departure times from the website.
7. Statement : The government has decided to
II. The passengers may be able to notice the provide monetary relief to the farmers in the
change and board their respective trains drought-hit areas.
before departure. Assumptions :
3. Statement : “Join our Yoga institute to keep I. The farmers of the affected areas may
yourself completely fit.” — An advertisement accept the government relief.
Assumptions : II. The government machinery may be able
I. People may prefer exercise to medication. to reach the affected farmers to provide
relief.
215
8. Statement : Mohan requested his mother to Assumptions :
arrange food for about thirty persons as he I. The prices of crude oil in the international
invited all his friends to celebrate his birthday.
market may again increase in the near
Assumptions : future.
I. Most of Mohan’s friends may come to II. The present price difference of petroleum
his house on his birthday.
products will help the government to
II. There may not be more than thirty withstand any possible price rise in
Persons who may attend Mohan’s future.
birthday party.
13. Statement : The govt, has made an appeal to
9. Statement : The teachers of all the degree col all the citizens to honestly pay income tax and
leges went on an indefinite strike in protest file returns reflecting the true income level to
against the Govt’s decision to postpone the help the Government to carry out
pay revision to next year. developmental activities.
Assumptions : Assumptions :
I. The govt, may suspend all the striking I. People may now start paying more taxes
teachers. in response to the appeal.
II. The Govt, may revise the pay of the II. The total income tax collection may
college teachers in the current year. considerably increase in the near future.
10. Statement : The government announced a 14. Statement : The state government has decided
heavy compensation package for all the to appoint four thousand primary school
victims of the terrorist attacks. teachers during the next financial year.

Assumptions : Assumptions :
I. There are enough schools in the state to
I. Such incidents of terror may not occur in
accommodate four thousand additional
near future.
primary school teachers.
II. Compensation may mitigate the anger II. The eligible candidates may not be
among the citizens against the current interested to apply as the Government
government. may not finally appoint such a large
11. Statement : Gambling through lotteries is number of primary school teachers.
banned by the Central Government in all the 15. Statement : The retail vegetable vendors
states with immediate effect. increased the prices of vegetables by about 20
Assumptions : per cent due to non-availability of vegetables
at lower prices at the wholesale market.
I. This may save innocent citizens from
Assumptions :
getting cheated of their hard-earned
money. I. The customers may totally stop buying
vegetables at higher prices.
II. The citizens may not gamble in any other
II. The customers may still buy vegetables
way if the lotteries are banned.
from the retail vendors.
12. Statement : The Government has decided
16. Statement : Provide mid-day meals to the
against reduction of prices of petroleum children in primary schools to increase the
products though there is a significant drop in number of students attending schools.
the crude oil prices in the international market.

216
Assumptions : gone up in recent times.
I. Mid-day meals will attract the children II. Many luxury goods are available in plenty
to the schools.
in the country.
II. Those children who are otherwise
deprived of good food will attend the 22. Statement : Unemployment allowance should
schools. be given to all unemployed Indian youth above
17. Statement : Imprisonment for 27 years made 18 years of age.
Nelson Mandela, the President. Assumptions :
Assumptions :
I. There are unemployed youth in India who
I. Only who will be imprisoned for 27 years need monetary support.
will become the president.
II. The government has sufficient funds to
II. To become the President, imprisonment
is a qualification. provide allowance to all unemployed
youth.
18. Statement : The government has decided to
23. Statement : “Do not lean out of the moving
reduce its subsidy on LPG however the
train.” — A warning in the railway
subsidy on kerosene remains unchanged.
compartment.
Assumptions :
Assumptions :
I. Those people who buy LPG can afford I. Such warnings will have some effect.
to purchase LPG for a higher price.
II. Leaning out of a moving train is
II. Many people may stop buying LPG and dangerous.
instead use kerosene. III. It is the duty of railway authorities to take
19. Statement : It is faster to travel by air to Delhi care of passengers’ safety.
from Bangalore. (a) Only I and II are implicit
Assumptions : (b) Only II and III are implicit
I. Bangalore and Delhi are connected by air. (c) Only II is implicit

II. There are no other means of transport (d) Only I and III are implicit
available to Delhi from Bangalore. (e) All are implicit

20. Statement : If it is easy to become an engineer, 24. Statement : There is big boom in drug
business and a number of Jhuggi-Jhopari
I don’t want to be an engineer.
dwellers in Delhi can be seen pedaling with
Assumptions : small pouches of smack and brown sugar.
I. An individual aspires to be professional. Assumptions :
II. One desires to achieve a thing which is I. Drug addiction is increasing in the
hard earned. country, specially in the capital.
21. Statement : The cost of living has gone up in II. All the big dons involved in the
India. smuggling of drugs live in jhuggi-jhopari
areas.
Assumptions :
III. Most of the jhuggi-jhopari dwellers
I. The price of essential commodities has

217
would do anything for money. packet.
(a) Only I is implicit II. People take careful note of a warning.
(b) Only II is implicit III. Non-smoking promotes health.
(c) Only III is implicit (a) Only I is implicit
(d) Only I and III are implicit (b) Only I and II are implicit
(e) Either I or III is implicit (c) Only II is implicit
25. Statement : The address of the Principal to (d) All are implicit
the students, “Dear students, if you want a (e) None of these
healthy mind, listen to music.
28. Statement : The telephone company informed
Assumptions : the subscribers through a notification that
I. Normally students like to follow good those who do not pay their bills by the due
advice. date will be charged penalty for every
II. It is desirable to develop a healthy mind. defaulting day.

III. It is the duty of the Principal to advise Assumptions :


the students. I. Majority of the people may pay their bills
(a) Only I and II are implicit by the due date to avoid penalty.
II. The amount collected as penalty may set
(b) Only II and III are implicit
off the losses due to delayed payment.
(c) Only I and III are implicit
III. People generally pay heed to such
(d) None is implicit notices.
(e) None of these (a) All are implicit
26. Statement : Inspire of the heavy rains the (b) Only I and II are implcit
traffic has not been disrupted this year.
(c) None is implicit
Assumptions :
(d) Only II and III are implicit
I. The traffic is disrupted in rainy seasons
(e) None of these
only.
29. Statement : "Use Craft colours. They add
II. Rains do not affect traffic movement.
colour to our life", said an advertisement.
III. Adequate precautions were taken for (MPPSC-2020)
traffic management during rainy season. Assumptions :
(a) Only I is implicit I. Catchy slogans do not attract people.
(b) Only I and II are implicit II. People like dark colours.
30. Statement : Anger is an emotion; it needs
(c) Only III is implicit
special skills as to control it.
(d) Only II and III are implicit (MPPSC-2020)
(e) None is implicit. Assumptions :
27. Statement : ‘Smoking is injurious to health’ I. Anger need to be controlled.
— A warning printed on the cigarette packets. II. Only skillful people can control it.
Assumptions :
I. People read printed matter on a cigarette
218
Answer Sheet

1. (b) 13. (e) 25. (c)


2. (e) 14. (a) 26. (c)
3. (b) 15. (b) 27. (c)
4. (e) 16. (a) 28. (e)
5. (e) 17. (d) 29. (c)
6. (a) 18. (a) 30. (b)
7. (e) 19. (a) *****
8. (a) 20. (b)
9. (b) 21. (e)
10. (b) 22. (a)
11. (a) 23. (e)
12. (d) 24. (d)

219
29
Direction : Given the following questions II. The government should announce an
Statement followed by two course of action attractive package to ensure regular
numbered I and II. An Course of action is supply of raw material for food
something supposed or taken for granted. You have processing industry.
to consider the statement and the following course
3. Statement : A very large number of students
of action and decide which of the Course of action
have failed in the final high school
is implict in the statement.
examination due to faulty questions in one of
Give (a) answer if only course of action I is the subjects.
implict.
Courses of action :
Give (b) answer if only course of action II is
implict. I. All the students who have failed in that
subject should be allowed to take
Give (c) answer if both course of action I and
II are implict. supplementary examination.
Give (d) answer if neither course of action I II. All those who are responsible for the error
implict II and implict. should be suspended and an enquiry
should be initiated to find out the facts.
1. Statement : The Minister said that the teachers
are still not familiar is with the need, 4. Statement : The prices of food-grains and
importance and meaning of populatic vegetables have substantially increased due to
education in the higher education system. a prolonged strike call given by the truck
They are not even clearly aware about their owners’ association.
rol and responsibilities in the population Courses of action :
education programme. I. The government should immediately
Courses of action : make alternative arrangement to ensure
adequate supply of food-grains and
I. Population education programme should
vegetables in the market.
be include in the college curriculum.
II. The government should take steps to
II. Orientation programme should be
cancel the licenses of all the
conducted teachers on population
vehiclesbelonging to the association.
education.
5. Statement : There have been a large number
2. Statement : One of the problems facing the of cases of Internet hacking in the recent
food processing industry is the irregular supply months, creating panic among the Internet
of raw material. The producers of raw material users.
are not getting a reasonable price.
Courses of action :
Courses of action :
I. The government machinery should make
I. The government should regulate the an all effort to nab those who are
supply of raw material to other industries responsible and put the behind bars.
also. II. The Internet users should be advised to

220
stay away from using Internet till the Courses of action :
culprits are caught. I. The municipal authority should advise
people living in the area not to use water
6. Statement: There is an alarming increase in
supplied through pipeline for drinking
the number of people suffering from malaria purpose.
in many parts of the city.
II. The local hospitals should be put on high
Courses of action : alert to tackle the emerging crisis
I. The municipal corporation has advised situation.
all the Govt, hospitals to store adequate 11. Statement : Misuse of subsidies offered to
the farmer was observed and brought to the
supply of malaria drugs.
notice of the concerns authorities.
II. The municipal corporation has urged Courses of action :
people to use mosquito repellents and I. Government should issue orders to the
keep their premises clean. concerns officials to be stricter and more
careful while verifying the required
7. Statement : Many motorists driving on the documents while granting subsidy.
highway within the city are found to be driving II. Government should take stringent action
much beyond the permissible speed limit. agains those making false claim of
Courses of action : subsidy.
I. The traffic police officials should 12. Statement : The daytime temperatures in this
personally monitor the movement of
summer have been four to five degrees Celsius
vehicles on the highway within the city.
above the normal temperature across the
II. The Govt, should immediately put in
place a mechanism to identify and punish country.
erring drivers. Courses of action :
8. Statement : Increasing levels of air pollution I. The government machinery should be put
is creating health hazards for people living in
on high alert and provided with necessary
the cities.
equipments to prevent any untoward
Courses of action :
incident.
I. All industries should be shifted to the
outskirts of the cities. II. The government should make necessary
II. Transport authorities should take steps for arrangements to provide water in all the
converting all public transport vehicles areas affected due to extreme heat waves.
to run on CNG
13. Statement : A large number of engineering
9. Statement : Money has become more
graduates in the country are not in a position
important than the game itself in the case of
Indian cricket. to have gainful employment at project and the
Courses of action : number of such engineers is likely to grow in
I. Govt, should put a cap on the earnings the future.
from different sources of the Indian Courses of action :
cricket board.
I. The government should launch attractive
II. Govt, should put a cap on the earnings
employment generation schemes and
from different sources of Indian cricket
players. encourage these graduate to opt for such
10. Statement: Very large numbers of people from schemes to use their expertise and
northern part of the city are suffering from knowledge effectively.
water-borne diseases. II. This happened due to proliferation of
221
engineering colleges in the country and incentive for those who are sharing a
thereby lowered the quality of the private vehicle.
engineering graduates. Those colleges II. City development authorities should
which are not equipped to impart quality impose heavy taxes on use of individual
education should be closed down. private vehicle.
14. Statement : Many medical and engineering 18. Statement : Cases of robbery while travelling
in public transport have increased substantially
graduates are taking up jobs in administrative
in the recent past.
services and in banks.
Courses of action :
Courses of action :
I. Adequate number of security guards
I. All the professionals should be advised should be deployed in all public transport
to refrain from taking up such jobs. vehicles immediately.
II. The government should appoint a II. People should be advised to refrain from
committee to find out the reasons for carrying highly valuable articles while
these professionals taking up jobs and to travelling by public transport system.
suggest remedial measures. 19. Statement : A sudden cloud burst over the
15. Statement : The apex court has directed that island city resulted into unpredicted rainfall
there is a need to bring in a mechanism in the causing a flood-like situation in the entire area.
Govt, functions to make them transparent. A large number of people were caught
Courses of action : unaware and were stranded on the road.
Courses of action:
I. The Govt, should immediately appoint a
Courses of action :
task force to work out the modalities.
I. The local administration should
II. The Government should appeal to the immediately put in place an action plan
apex court to reconsider its directive. for avoiding such a situation in future.
16. Statement : The prices of crude oil in the II. The local administration should
international market have risen by about 40 immediately deploy personnel to help the
per cent in a month’s time and show no stranded people to move to safer places.
downward trend. III. The local administration should advise
Courses of action : all the citizens not to venture out on the
road till the situation improves.
I.The petrol prices in the domestic retail
(a) Only I follows
market should immediately be increased
(b) Only II follows
by about 30 per cent to absorb the cost
(c) Only III follows
escalation.
(d) Only II and III follow
II. The Govt, should provide subsidy to the (e) All I, II and III follow
oil marketing companies to absorb the 20. Statement : A large number of employees
cost escalation. have gone on a mass casual leave in protest
17. Statement : Air pollution level in the city is against the company's new recruitment policy.
observed to be rising up day by day. Courses of Action
Courses of action : I. The company should immediately
I. City development authorities should offer withdraw the new recruitment policy.

222
II. All these employees should immediately
be suspended from their service.

Answer Sheet
1. (b) 11. (e)
2. (b) 12. (a)
3. (e) 13. (d)
4. (a) 14. (b)
5. (a) 15. (a)
6. (e) 16. (c)
7. (b) 17. (e)
8. (d) 18. (d)
9. (b) 19. (a)
10. (b) 20. (a)

223

You might also like